Item: 1 of 24 ~ 1 • M k -<:J 1>- Jil ~· !:';-~ QIO: 4749 ..L a r Previous Next Labfli!llues Not es Calculat o r

• 1 & & A 67-year-old man admitted for postoperative recovery is found to be oliguric. Laboratory studies show a blood urea nitrogen level of 200 • 2 mg/dl and a serum level of 6 mg/dl. Urinalysis shows: • 3 Specific gravity: 1.050 · 4 osmolality: 670 mOsm/kg • 5 Sodium: 14 mEq/L BUN/Creatinine ratio: 56 • 6 Fractional excretion of Na: 0.54% • 7 Protein: negative Casts: negative · 8 . 9

• 10 Which of the following is the most likely cause of this patient's oliguria? • 11 : • 12 A. Acute interstitial nephritis

• 13 B. • 14 C. Bladder calculus • 15 • 16 D. Heart failure

• 17 E. • 18 • 19 • 20 • 21 • a s 8 Lock Suspend End Block Item: 1of24 ~ . , . M k <:] t> al ~· ~ QIO: 4749 .l. ar Previous Next Lab'lifllues Notes Calculator

1 • The correct a nswer is 0. 4 80/o c hose t his. • 2 This patient's laboratory tests confirm the classic criteria for d iagnosing prerenal azotemia. Prerenal azotemia is caused by a reduction of the • 3 g lomerular filtration rate (GFR} provoked by an insult to the vascular supply to the . Causes of prerenal azotemia include heart failure, sepsis, and renal artery stenosis. The reduction in GFR increases the accumulation of both blood urea nitrogen (BUN} and creatinine (Cr} in the • 4 blood, but because the BUN concentration in blood is determined by both g lomerular filtration and reabsorption (in contrast to Cr, which is • 5 limited to filtration and not reabsorbed}, the BUN level rises out of proportion to the Cr leveL This therefore elevates the BUN :Cr ratio. In • 6 addition, this patient's fractional excretion of sodium (FeNa) is less than 1%, ind icative of a prerenalI cause. f the FeNa is g reater than 2%, an intrinsic renal orig in, such as acute tubular necrosis, is the cause. Of the causes listed here, heart failure is the most likely option for a prerenal • 7 cause. • 8 Renal artery stenosis Blood urea nitrogen Azotemia Renal function Acute tubular necrosis Urea Creatinine Sepsis Fractional sodium excretion Kidney Renal artery Glomerulus Sodium Glomerulus (kidney) Stenosis Necrosis Blood vessel Nitrogen Heart failure Excretion • 9 A is not correct. 1 50/o c hose t his . • 10 Acute interstitial nephritis (AIN} is an intrarenal cause of azotemia caused by due to a number of factors, includ ing toxins and · 11 d rugs. AIN usually manifests with eosinophilic casts in the urine and with findings similar to acute tubular necrosis (ATN}, includ ing an elevated • 12 fractional excretion of sodium . Interstitial nephritis Acute tubular necrosis Nephritis Azotemia Fractional sodium excretion Urine Interstitial fluid Sodium Necrosis Inflammation Excretion • 13 B is not correct. 200/o c hose t his . • 14 Acute tubular necrosis (ATN} can mimic prerenal azotemia in many ways, although it is essentially an intrarenal azotemia. However, the maj or • 15 d istinguishing features that make this patient unlikely to have ATN are the absence of casts in the urine, the high urine-specific g ravity (ATN • 16 usually causes nonoliguric azotemia), and the very high BUN :Cr ratio. In addition, in ATN the fractional excretion of sodium is usually >2%, which is not the case in this patient. • 17 Acute tubular necrosis Azotemia Fractional sodium excretion Necrosis Sodium Urine Excretion • 18 C is not correct. 130/o c hose t his . • 19 Bladder calculi are a maj or cause of postrenal azotemia, which is d istinguished from prerenal azotemia by the relatively normal BUN :Cr ratio and, • 20 more importantly, the lack of highly concentrated urine. This patient's BUN :Cr ratio, urine specific g ravity, and fractional excretion of sodium suggest the presence of prerenal azotemia . • 21 • Specific gravity Azotemia Fractional sodium excretion Urine Urinary bladder Sodium Excretion 6 s 0 lock Suspend End Block Item: 1of24 ~ . , . M k <:] t> al ~· ~ QIO: 4749 .l. ar Previous Next Lab'lifllues Notes Calculator

1 A is not correct. 1 50/o c hose t his. . 2 Acute interstitial nephritis (AIN} is an intrarenal cause of azotemia caused by inflammation due to a number of factors, includ ing toxins and d rugs. AIN usually manifests with eosinophilic casts in the urine and with findings similar to acute tubular necrosis (ATN}, includ ing an elevated • 3 fractional excretion of sodium. . 4 Interstitial nephritis Acute tubular necrosis Nephritis Azotemia Fractional sodium excretion Urine Interstitial fluid Sodium Necrosis Inflammation Excretion

• 5 B is not correct. 20 0/o c hose t his. • 6 Acute tubular necrosis (ATN} can mimic prerenal azotemia in many ways, although it is essentially an intrarenal azotemia. However, the maj or d istinguishing features that make this patient unlikely to have ATN are the absence of casts in the urine, the high urine-specific g ravity (ATN . 7 usually causes nonoliguric azotemia), and the very high BUN :Cr ratio. In addition, in ATN the fractional excretion of sodium is usually >2%, • 8 which is not the case in this patient. Acute tubular necrosis Azotemia Fractional sodium excretion Necrosis Sodium Urine Excretion • 9 • 10 C is not correct. 130/o c hose t his . Bladder calculi are a maj or cause of postrenal azotemia, which is d istinguished from prerenal azotemia by the relatively normal BUN :Cr ratio and, · 11 more importantly, the lack of highly concentrated urine. This patient's BUN :Cr ratio, urine specific g ravity, and fractional excretion of sodium • 12 suggest the presence of prerenal azotemia . Specific gravity Azotemia Fractional sodium excretion Urine Urinary bladder Sodium Excretion • 13 E 4 0/o • 14 is not correct. c hose t his . Nephrotic syndrome is caused by excessive g lomerular filtration of protein due to loss of the g lomerular filtration barrier. The characteristic • 15 clinical feature of nephrotic syndrome is the presence of , which is not seen in this patient. Patients with nephrotic syndrome also • 16 commonly have hypertension, edema, and hypercholesterolemia . Nephrotic syndrome Proteinuria Hypercholesterolemia Hypertension Protein Glomerulus Edema Glomerulus (kidney) Renal function • 17

• 18 • 19 Bottom Line : • 20 Prerenal azotemia is caused by a reduction of the g lomerular filtration rate (heart failure, sepsis, and renal artery stenosis) . Renal artery stenosis Azotemia Renal function Sepsis Renal artery Glomerulus Stenosis Glomerulus (kidney) Heart failure • 21 6 s 0 lock Suspend End Block Item: 1 of 24 ~ 1 • M k -<:J 1>- Jil ~· !:';-~ QIO: 4749 ..L a r Previous Next Labfli!llues Not es Calculat o r

1 & & FA17 p 571 .2 • 2 Acute kidney injury • 3 Acute kidney injury is de fin ed as an abrupt decline in renal functi on as measured by t creatinine (acute renal failure) and t BU! or by oliguria/anuria. · 4 Prerenal azotemia Due to l RBF (eg, hypotemion) - l CFR. la•/ 11 0 and BUt retained by kidn ey in an attempt to • 5 1 consen e volume - t BU '/creatinine ratio (BUN is reabsorbed, crea ti nine is not) and l F'E'Ia· • 6 Intrinsic renal failure Cenerallr due to acute tubular necrosis or ischemia/toxins; less commonly due to acute • 7 (eg, RPCN, hcmol) Lie uremic S) nd rome) or ac ute interstitial nephritis. · 8 In ATN, patchy necrosis - debr is obstructing tubule and Auid backAow across necrotic tubule . 9 - l C FR. Urine has epithel ial/granular c~1 s t s. BU!': reabsorption is impaired - l BUI':/crcatininc • 10 rati o and t FENa· • 11 Postrenal azot emia Due to outfl ow obstructi on (stones, BPI I, neoplasia, congenital anomalies). De,·elops onl y with • 12 bilateral obstruction . • 13 Pre renal Intrinsic renal Postrenal • 14 > 500 < 350 < 350 • 15 (mOsm/kg) • 16 Urine Na+ (mEq/l) <20 > 40 > 40 • 17 FENa < I% >2% < 1% (mild) • 18 > 2% (se,·ere) • 19 Serum BUN/Cr > 20 < 15 Varies • 20 • 21 • FA17 o 562.2 a s 8 Lock Suspend End Block Item: 1 of 24 ~ 1 • M k -<:J 1>- Jil ~· !:';-~ QIO: 4749 ..L a r Previous Next Labfli!llues Not es Calculat o r

1 & & • 2 FA17 p 562.2 • 3 Casts in urine Presence of casts indicates that /pyuria is of glomerular or renal tubular origin. · 4 Bladder cancer, kidney stones - hematuria, no casls. • 5 Acute cystitis - pyuria, no casts. • 6 RBC casts Glomerulonephritis, malignant hypertension. • 7 WBC casts Tubulointerstitial inAammation, acute , transplant rejection. · 8 Fatty casts ("oval fat 'ephrotic syndrome. Associated with ·· i\ laltese cross" sign. . 9 bodies") • 10 Granular ("muddy Acute tubular necrosis. • 11 brown") casts • 12 Waxy casts I!] End-stage renal disease/chronic renal failure. • 13 Hyaline casts 0 1'\onspecific, can be a normal finding, often seen in concentrated urine samples. • 14 • 15 • 16 ' • 17 • 18 • 19 • 20 • 21 • a s 8 Lock Suspend End Block Item: 2 of 24 ~ 1 • M k -<:J 1>- Jil ~· !:';-~ QIO: 4162 ..L a r Previous Next Labfli!l tues Not es Calculat o r

1 & & On a routine physical examination, a previously healthy 26-year-old woman has a blood pressure of 150/85 mm Hg . She has had no other • 2 physical symptoms. She begins following a diet and exercise program, but on her next visit, her blood pressure is 170/95 mm Hg. A blood • 3 test demonstrates elevated plasma renin. · 4 Which is the most likely explanation for this patient's condition? • 5 • 6 A. A thromboembolus lodged in the renal artery • 7 of the renal artery · 8 B. Atherosclerosis

. 9 C. Conn syndrome • 10 D. Fibromuscular dysplasia • 11 E. Pheochromocytoma • 12

• 13 • 14

• 15

• 16

• 17

• 18 • 19

• 20

• 21 • a s 8 Lock Suspend End Block Item: 2 of 24 ~ . I • M k <:] t> al ~· ~ QIO: 4162 .l. ar Previous Next lab 'lifllues Notes Calculator • 1 The correct a nswer is 0. 4 50/o c hose t his. 2 In young, healthy women with sudden-onset hypertension, it is im portant to consider inherent structural defects of the renal artery wall that may • 3 impair perfusion of one or both kid neys. These defects are termed fibromuscular dysplasias. Decreased perfusion leads to stimulation of the renin-ang iotensin-aldosterone ax is, resulting in systemic vasoconstriction and volume expansion. If the cond ition is unilateral, ultrasound may • 4 demonstrate one atrophic kid ney and if advanced, contralateral hypertrophy of the unaffected kid ney . Re nal artery stenosis of this type is best • 5 treated with surgery, which can then help relieve the secondary hypertension. Renal artery stenosis Vasoconstriction Secondary hypertension Renal artery Ultrasound Stenosis Hypertension Medical ultrasound Kidney Hypertrophy Perfusion • 6 Anatomical terms of location Contralateral Atrophy • 7 A is not correct. 130/o c hose t his . • 8 A thromboembolus in the renal artery may cause new -onset hypertension; however, it is unlikely to be the cause in this young, otherwise healthy • 9 patient. Thromboemboli to the renal artery often orig inate in the left heart, either after myocard ial or resulting from bacterial vegetations seen in bacterial endocard itis. An acute embolus would likely cause a spectrum of other symptoms and could result in infarction of • 10 the affected kid ney . · 11 Myocardial infarction Embolism Embolus Endocarditis Hypertension Renal artery Kidney Infarction Heart • 12 B is not correct. 180/o c hose t his. • 13 Atherosclerosis of the renal artery is a more common cause of sudden-onset hypertension but would more likely be suspected in ind ivid uals over 50 years old or with pred isposing cond itions (eg, d iabetes). Similarly to fibromuscular dysplasia, decreased perfusion of one or both kid neys • 14 leads to stimulation of the renin-ang iotensin-aldosterone ax is, resulting in systemic vasoconstriction and volume expansion. Re nal artery stenosis • 15 of this type is best treated with conventional balloon ang ioplasty . Fibromuscular dysplasia Renal artery stenosis Atherosclerosis Vasoconstriction Renal artery Angioplasty Stenosis Hypertension Diabetes mellitus Perfusion Kidney • 16 • 17 C is not correct. 1 60/o c hose t his . Conn synd rome is caused by an aldosterone-secreting adenoma of the ad renal cortex and is a form of hyperaldosteronism. It may result in new ­ • 18 onset hypertension and is often associated with other systemic symptoms (alkalosis and hypokalemia). If the patient had an aldosterone­ • 19 secreting tumor, she would be expected to have low plasma renin . Hypokalemia Primary aldosteronism Adrenal cortex Hyperaldosteronism Hypertension Renin Blood plasma Alkalosis Adenoma Neoplasm Adrenal gland • 20 E is not correct. 80fo c hose t his . • 21 r oo ~ · ' . ' " ' 0 6 s 0 lock Suspend End Block Item: 2 of 24 ~ . I • M k <:] t> al ~· ~ QIO: 4162 .l. ar Previous Next lab 'lifllues Notes Calculator

1 B is not correct. 180/o c hose t his. 2 Atherosclerosis of the renal artery is a more common cause of sudden-onset hypertension but would more likely be suspected in ind ividuals over • 3 50 years old or with pred isposing conditions (eg, d iabetes). Sim ilarly to fibromuscular dysplasia, decreased perfusion of one or both kidneys leads to stimulation of the ren in -angiotensin-aldosterone axis, resu lting in systemic vasoconstriction and volume expansion . Renal artery stenosis . 4 of this type is best treated with conventional balloon angioplasty. • 5 Fibromuscular dysplasia Renal artery stenosis Atherosclerosis Vasoconstriction Renal artery Angioplasty Stenosis Hypertension Diabetes mellitus Perfusion Kidney

• 6 C is not correct. 1 60/o c hose t his . . 7 Conn syndrome is caused by an aldosterone-secreting adenoma of the adrenal cortex and is a form of hyperaldosteronism. It may resu lt in new­ onset hypertension and is often associated with other systemic symptoms (alkalosis and hypokalemia). If the patient had an aldosterone­ • 8 secreting tumor, she would be expected to have low plasma ren in . • 9 Hypokalemia Primary aldosteronism Adrenal cortex Hyperaldosteronism Hypertension Renin Blood plasma Alkalosis Adenoma Neoplasm Adrenal gland

• 10 E is not correct. 80/o c hose t his . · 11 Pheoch romocytoma is a catecholamine-secreting tumor of the adrenal medulla. Increases in plasma epinephrine and norepinephrine cause relapsing and remitting symptoms such as hypertension, d iaphoresis, headaches, and heart palpitations. However, plasma ren in levels would not • 12 be increased in the setting of pheoch romocytoma. Approximately 50% of patients have hypertension at baseline, not j ust paroxysm ally, an • 13 interesting physical exam finding in these patients is orthostatic hypotension. Because the tumor still secretes low levels of catecholamines at baseline the vasculature in these patient's is already maximally constricted, upon standing they are unable to constrict their vessels further, • 14 resu lting in orthostatic hypotension. When systemic vasculature is maximally constricted, ren in will not be released and is low in these patients. • 15 Pheochromocytoma Orthostatic hypotension Epinephrine Adrenal medulla Norepinephrine Perspiration Renin Hypotension Hypertension Catecholamine • 16 Blood plasma Circulatory system Palpitations Vascular resistance Neoplasm Medulla oblongata Adrenal gland Physical examination • 17

• 18 Bottom Line : • 19 Sudden-onset hypertension can be associated with renal artery stenosis, in which fibromuscular changes to the renal artery impair perfusion of the kidney . • 20 Renal artery stenosis Renal artery Hypertension Stenosis Perfusion Kidney • 21 6 s 0 lock Suspend End Block Item: 2 of 24 ~ 1 • M k -<:J 1>- Jil ~· !:';-~ QIO: 4162 ..L a r Previous Next Labfli!l tues Not es Calculat o r

1 & & FA17 p 290.2 2 Hypertension Defined as persistent systolic BP ~ 140 mm Hg and/or diastolic BP ~ 90 mm Hg • 3 RISK FACTORS f age, obesity, diabetes, physical inactivity, excess salt intake, excess alcohol intake, family histor); · 4 African American > Caucasian> sian. • 5 FEATURES 90% of hypertension is 1° (essential) and related to f CO or f TPR. Remaining 10% mostly zoto • 6 renal/renovascular disease (eg, fibromuscular dysplasia (which has characteristic "string of beads" . 7 appearance of renal artery ], atherosclerosis) and 1° hyperaldosteronism. · 8 Hypertensive urgency-se,ere ( ~ ISOP- 120 mm llg) h~ pertension without acute end-organ . 9 damage. Hypertensive emergency-severe h) pertension with e\'idence of acute end-organ damage (eg, • 10 encephalopathy, stroke, retinal hemorrhages and e-..udates, papilledema, MI, HF, aortic dissection, • 11 kidney injury, microangiopathic hemolytic anemia, eclampsia). • 12 • 13 PREDISPOSES TO CAD, LVH, HF, atrial fibrillation; aortic dissection, aortic aneurysm; stroke; (hypertensive nephropathy); retinopathy. • 14 • 15 FA17 p 292.1 • 16 Atherosclerosis and medium-sized muscular arteries; a form of • 17 Very common. Disease of clastic arteries ami large- arteriosclerosis caused by buildup of cholesterol plaques . . 18 LOCATION Abdominal aorta > coronary artery > popliteal arlcr) >carotid artery t:J. • 19 • 20 • 21 • a s 8 Lock Suspend End Block Item: 3 of 24 ~ 1 • M k -<:J 1>- Jil ~· !:';-~ QIO: 1806 ..L a r Previous Next Labfli!l tues Notes Calculat o r

1 & & A 50-year-old man is admitted to the hospital. His drug regimen includes furosemide, a loop diuretic that acts on the sodium-potassium­ 2 chloride symporter in the ascending limb of the loop of Henle. The patient's treating physician must decide which medications to continue • 3 during the patient's hospital stay.

· 4 The presence of which of the following conditions in the patient would prompt the discontinuation of furosemide? • 5 • 6 A. Exacerbation of congestive heart failure • 7

· 8 B. Hyperkalemia

. 9 C. Hypocalcemia • 10 D. Nephrotic syndrome • 11 E. Worsening hypertension • 12 • 13 • 14 • 15 • 16 • 17 • 18 • 19 • 20 • 21 • a s 8 Lock Suspend End Block Item:3of24 ~ . , . M k <:] t> al ~· ~ QIO: 1806 .l. ar Previous Next lab 'lifllues Notes Calculator

1 The correct answer is C. 680/o chose this. 2 Furosemide is a loop d iuretic that inhibits the sodium-potassium-chloride co-transporter, and at maximal dosage, can excrete about 25% of 3 filtered sodium. The reabsorption of calcium in the loop of Henle is primarily passive, being d riven by the g rad ient created by sodium and chloride . 4 transport. With furosemide, this g rad ient is destroyed. Because it promotes the excretion of calcium, furosemide is used to treat hypercalcemia, but it is contraindicated in a hypocalcemic state. • 5 loop diuretic Furosemide Diuretic Hypercalcaemia loop of Henle Calcium Sodium Hypocalcaemia Reabsorption

• 6 A is not correct. ]Ofo chose this. . 7 A patient with an exacerbation of congestive heart failure (CH F} is a good candidate for furosemide. In decompensated CH F, patients are volume • 8 overloaded, and as a resu lt, may have pulmonary edema and experience d ifficulty breathing . Furosemide will encourage d iuresis and help the body mobilize the excess fluid, clearing the lungs and helping that patient breathe better. • 9 Furosemide Pulmonary edema Heart failure Congestive heart failure Diuretic Edema Forced diuresis Polyuria lung

• 10 B is not correct. 11 Ofo chose this. · 11 Hyperkalemia is a serious condition because of the possibility of developing an arrhythmia. Patients should be treated when ECG changes are present (spiking of the T wave) or if serum potassium reaches > 7.0 m Eq/ L. Initial treatment may include calcium, which will antagonize the • 12 membrane actions of hyperkalemia to help prevent arrhythmia, along with insulin and g lucose, which help to pull the potassium from outside to • 13 inside the ceiL The patient also may receive sodium polystyrene (Kayexalate), a gastrointestinal syrup that works more slowly by ridding the body of potassium through the stooL Loop d iuretics such as furosemide actually may be used to help with hyperkalemia as well, as they increase • 14 excretion of potassium . • 15 Furosemide Hyperkalemia Cardiac arrhythmia Potassium Diuretic Electrocardiography T wave Sodium Polystyrene sulfonate Insulin Polystyrene loop diuretic • 16 Calcium Blood plasma Serum (blood) Equivalent (chemistry) Human gastrointestinal tract Gastrointestinal tract

• 17 0 is not correct. 90fo chose this . • 18 Nephrotic syndrome is characterized by massive proteinuria, hypoalbuminemia (because of loss of albumin in urine), edema, and hypercholesterolemia. Whereas the treatment of nephrotic syndrome is targeted at the underlying d isease (which can be quite varied}, d iuretics • 19 like furosemide also are used to treat the edema associated with this syndrome. • 20 Furosemide Nephrotic syndrome Hypoalbuminemia Proteinuria Hypercholesterolemia Edema Albumin Urine Diuretic

• 21 E is not correct. 50fo chose this . 6 s 0 lock Suspend End Block Item:3of24 ~ . , . M k <:] t> al ~· ~ QIO: 1806 .l. ar Previous Next lab 'lifllues Notes Calculator y g y g 1 • • • • •

2 B is not correct. 11 Ofo c hose t his. Hyperkalemia is a serious condition because of the possibility of developing an arrhythmia. Patients should be treated when ECG changes are 3 present (spiking of the T wave) or if serum potassiu m reaches > 7.0 m Eq/ L. Initial treatment may include calcium, which will antagonize the • 4 membrane actions of hyperkalemia to help prevent arrhythmia, along with insulin and g lucose, which help to pull the potassium from outside to inside the ceiL The patient also may receive sodium polystyrene (Kayexalate), a gastrointestinal syrup that works more slowly by ridding the • 5 body of potassium through the stooL Loop d iuretics such as furosemide actually may be used to help with hyperkalemia as well, as they increase • 6 excretion of potassium. • 7 Furosemide Hyperkalemia Cardiac arrhythmia Potassium Diuretic Electrocardiography T wave Sodium Polystyrene sulfonate Insulin Polystyrene loop diuretic Glucose Calcium Blood plasma Serum (blood) Equivalent (chemistry) Human gastrointestinal tract Gastrointestinal tract • 8 0 is not correct. 90/o c hose t his . • 9 Nephrotic syndrome is characterized by massive proteinuria, hypoalbuminemia (because of loss of albumin in urine), edema, and • 10 hypercholesterolemia. Whereas the treatment of nephrotic syndrome is targeted at the underlying d isease (which can be quite varied}, d iuretics · 11 like furosemide also are used to treat the edema associated with this syndrome. Furosemide Nephrotic syndrome Hypoalbuminemia Proteinuria Human serum albumin Hypercholesterolemia Edema Albumin Urine Diuretic • 12 E is not correct. 50/o c hose t his . • 13 Hypertension is caused most often by a combination of too much intravascular fluid volume and excessive constriction of blood vessels. Loop • 14 d iuretics can be used to help lower blood pressure, but many d rugs that would be used before them, such as angiotensin-converting enzyme • 15 inhibitors and thiazide d iuretics . Thiazide Angiotensin-converting enzyme Enzyme ACE inhibitor Hypertension loop diuretic Diuretic Blood pressure Blood vessel Vasoconstriction Enzyme inhibitor • 16 • 17

• 18 Bottom Line : • 19 Clinical ind ications of furosemide include congestive heart failure, acute pulmonary edema, and hypercalcemia. Furosemide is contraindicated when hypocalcemia is present. • 20 Furosemide Hypocalcaemia Hypercalcaemia Heart failure Pulmonary edema Congestive heart failure Edema Contraindication • 21 • 6 s 0 lock Suspend End Block Item: 3 of 24 ~ 1 • M k -<:J 1>- Jil ~· !:';-~ QIO: 1806 ..L a r Previous Next Labfli!l tues Notes Calculat o r

1 & & FA17 p 575.3 2 loop diuretics 3 Furosemide, bumetanide, torsemide · 4 • 5 MECHANISM Sulfonamide loop diuretics. Inhibit cotransport system (Na+fK+f2CI-) of thid. ascending limb • 6 of loop of llcnlc. bolish hyperton icit} of . 7 medulla, preventing concentration of urine. · 8 Stimulate PCE release (\'asodilatory effect . 9 on afferent arteriole); inhibited b) NSAIDs. 2 2 • 10 f Ca + excretion. Loops Lose Ca • .

• 11 CliNICAl USE Edematous states (HF, cirrhosis, nephrotic syndrome, pulmonar} edema), h) perlension, • 12 hypercalcem ia . • 13 ADVERSE EFFECTS Ototoxicity, I lypokalemia, I Jypomagnesemia, O llll D.\Al':C! • 14 Dehydration, Allergy (sulfa), metabolic • 15 Alkalosis, Nephritis (interstit ia l), Gout . • 16 Ethacrynic acid • 17 MECHANISM ~onsulfon amide inhibitor of cotransport system . 18 ( a•fK+f2CI -) of thick ascending limb of loop • 19 of llenle . • 20 CLINICALUSE Diuresis in patients allergic to sulfa drugs . • 21 ADVERSE EFFECTS • Similar to furosemide, but more ototo,ic . Loop earrings hurt rour ears. a s 8 Lock Suspend End Block Item: 3 of 24 ~ 1 • M k -<:J 1>- Jil ~· !:';-~ QIO: 1806 ..L a r Previous Next Labfli!l tues Notes Calculat o r

1 & ( la+J K+fZCl -) of thick ascending limb of loop & 2 of llcnle. 3 CLINICAL USE Diuresis in patients allergic to sulfa dmgs. · 4 ADVERSE EFFECTS Similar to furosemide, but more ototo,ic. Loop earrings hurt your ears. • 5

• 6 FA17 p 576.3 • 7 Diuretics: electrolyte changes · 8 Urine NaCI t with all diuretics (strength varies based on potency of diuretic effect). Serum _ aCl may decrease . 9 as a result. • 10 Urine K• t especially with loop and thiazide diuretics. Serum K+ may decrease as a result. • 11 Blood pH l (acidem ia): carbonic anhydrase inhibitors: l IIC03- reabsorption. K+ sparing: aldosterone • 12 blockade prevents K ~ secretion and II+ secretion. Additionally, hyperkalemia leads to K+ entering • 13 all cells (,·ia H+fK+ exchanger) in exchange for H .. exiting cells . t (alkalemia): loop diuretics - Jil ~· !:';-~ QIO: 4694 ..L a r Previous Next Labfli!l tues Not es Calculat o r

1 & & A 65-year-old man presents to the office with complaints regarding his urine. He states that he has recently had bloody urine, but does not 2 have any urinary pain, hesitation, dribbling, or increased frequency. He also says he has lost 4.5 kg (10 lb) over the past 2 months. A biopsy reports that he has never traveled outside the United States. 3 of the patient's bladder wall is shown in the image. The patient . 4 • 5 • 6 . 7 · 8 . 9 • 10

• 11 • 12 • 13 • 14 • 15 • 16 • 17 Which of the following risk factors has the strongest association with this patient's disease? . 18 • 19 : A. A mutation in the VHL gene on chromosome 3 • 20 B. A mutation of the WT1 gene on chromosome 11 • 21 • a s 8 Lock Suspend End Block 2 3 ' 4 • 5 • 6 . 7 · 8 . 9

• 10

• 11 • 12 • 13 Which of the following risk factors has the strongest association with this patient's disease?

• 14 : • 15 A. A mutation in the VHL gene on chromosome 3

• 16 B. A mutation of the WT1 gene on chromosome 11 • 17 C. History of another primary tumor . 18 D. History of heavy cigarette smoking • 19 • 20 E. History of schistosomiasis • 21 • a s 8 Lock Suspend End Block Item:4of24 ~ . , . M k <:] t> al ~· ~ QIO: 4694 .l. ar Previous Next lab 'lifllues Notes Calculator

1 • The correct answer is 0. 690/o chose t his. 2 This patient has transitional cell carcinoma of the bladder, as evidenced by his painless hematuria and absent signs of prostate cancer. Histology 3 confirms the d iagnosis with the presence of invasive transitional cells and prominent, atypical nuclei. Numerous factors increase the risk of developing transitional cell carcinoma, the most significant of which is an extensive history of cigarette smoking. Cigarette smoking increases a 4 patient's risk by three- to sevenfold. Treatment of transitional cell carcinoma largely depends on the stage of d isease, although these tumors have • 5 a tendency to recu r regard less of d isease stage. Low-grade lesions may be treated with excision, whereas higher-grade lesions require a combination of excision and chemotherapy . • 6 Transitional cell carcinoma Hematuria Histology Prostate cancer Chemotherapy Urinary bladder Prostate Carcinoma Cancer Transitional epithelium

• 7 Tobacco smoking Neoplasm Epithelium Cell nucleus Cigarette • 8 A is not correct. 100/o chose t his . • 9 A mutation in the VHL gene of chromosome 3, which causes von Hippei-Lindau syndrome, is associated with a higher risk of renal cell carcinoma, • 10 rather than transitional cell carcinoma of the bladder. Von Hippel-lindau syndrome Transitional cell carcinoma Renal cell carcinoma Gene Mutation Chromosome 3 (human) Urinary bladder Carcinoma VHl · 11 Chromosome Von Hippel-lindau disease Kidney • 12 B is not correct. 50fo chose t his . • 13 A mutation in the WT1 gene on chromosome 11 would place this patient at higher risk of Wilms' tumor, a malignant neoplasm of renal • 14 mesenchymal cells. Wilms tumor is uncommon in adults . Wilms' tumor WTl Neoplasm Gene Chromosome 11 (human) Mutation Chromosome Cancer Malignancy Mesenchyme Mesenchymal stem cell Kidney • 15 C is not correct. 50fo chose t his . • 16 Transitional cell carcinomas are primary tumors, rather than metastases from other locations. Transitional cell carcinomas themselves often • 17 metastasize to other locations, especially the spinal column and other bones. • 18 Metastasis Vertebral column Neoplasm Transitional cell carcinoma Primary tumor Carcinoma Epithelium • 19 E is not correct. 11 Ofo chose t his . • 20 Schistosomiasis infection (eg, in a patient with travel to an endemic area, includ ing the Middle East) is a risk factor for the development of squamous cell carcinoma; however, smoking is the most significant risk factor. Furthermore, a history of schistosomiasis in a patient who has • 21 never traveled outside of the United States would be unlikely . • 6 s 0 lock Suspend End Block Item:4of24 ~ . , . M k <:] t> al ~· ~ QIO: 4694 .l. ar Previous Next lab 'lifllues Notes Calculator

1 Tobacco smoking Neoplasm Epithelium Cell nucleus Cigarette 2 A is not correct. 100/o c hose t his. 3 A mutation in the VHL gene of chromosome 3, which causes von Hippei-Lindau syndrome, is associated with a higher risk of renal cell carcinoma, 4 rather than transitional cell carcinoma of the bladder. Von Hippel-lindau syndrome Transitional cell carcinoma Renal cell carcinoma Gene Mutation Chromosome 3 (human) Urinary bladder Carcinoma VHl • 5 Chromosome Von Hippel-lindau disease Kidney • 6 B is not correct. 50/o c hose t his. • 7 A mutation in the WT1 gene on chromosome 11 would place this patient at higher risk of Wilms' tumor, a malignant neoplasm of renal • 8 mesenchymal cells. Wilms tumor is uncommon in adults. • 9 Wilms' tumor WTl Neoplasm Gene Chromosome 11 (human) Mutation Chromosome Cancer Malignancy Mesenchyme Mesenchymal stem cell Kidney • 10 C is not correct. 50/o c hose t his .

· 11 Transitional cell carcinomas are primary tumors, rather than metastases from other locations. Transitional cell carcinomas themselves often metastasize to other locations, especially the spinal column and other bones . • 12 Metastasis Vertebral column Neoplasm Transitional cell carcinoma Primary tumor Carcinoma Epithelium • 13 E is not correct. 11 Ofo c hose t his . • 14 Schistosomiasis infection (eg, in a patient with travel to an endemic area, includ ing the Middle East) is a risk factor for the development of • 15 squamous cell carcinoma; however, smoking is the most significant risk factor. Furthermore, a history of schistosomiasis in a patient who has never traveled outside of the United States would be unlikely . • 16 Schistosomiasis Squamous-cell carcinoma Endemism Risk factor Infection United States Squamous epithelial cell Middle East Carcinoma • 17 • 18 Bottom Line : • 19 Transitional cell carcinoma often presents with painless hematuria and constitutional symptoms. Incidence is influenced by a number of risk • 20 factors, includ ing smoking, aniline dye exposu re, and cyclophosphamide use. Hematuria Transitional cell carcinoma Cyclophosphamide Aniline Carcinoma Aniline dye Epithelium Dye • 21 • 6 s 0 lock Suspend End Block Item: 4 of 24 ~ 1 • M k -<:J 1>- Jil ~· !:';-~ QIO: 4694 ..L a r Previous Next Lab fli!ltues Notes Calculator

1 & Schistosomiasis Squamous-cell carcinoma Endemism Risk factor Infection United States Squamous epithelial cell Middle East Carcinoma & 2

3 Bo tto m line :

4 Transitional cell carcinoma often presents with painless hematuria and constitutional symptoms. Incidence is influenced by a number of risk • 5 factors, including smoking, aniline dye exposure, and cyclophosphamide use. HematUI T a1 sitional eel carcinoma Cyclophosphamide Aniline carcinoma An 1ne dye Epithel urn Dye • 6 • 7 · 8 I@ I ;J'.i 1;1I•J for y e ar: 20 17 ... . 9 FIRST AI D FA CTS • 10 FA17 p 569.2 • 11 Transitional cell mmon tumor of urinary tracl system • 12 lost co carcinoma (can occur in renal calyces, renal pelvis, • 13 ureters, and bladder) fJ IE). Can be suggested • 14 by painless hematuria (no casts). • 15 Associated wit h problems in yo ur Pee SAC: • 16 Phenacetin, Smoking, Ani li ne dyes, and • 17 Cyclophosphamide . • 18 • 19 • 20 • 21 • a s 8 Lock Suspend End Block Item: 5 of 24 ~ 1 • M k -<:J 1>- Jil ~· !:';-~ QIO: 4741 ..L a r Previous Next Labfli!llues Not es Calculat o r

& 1 A 30-year-old woman presents to the em er gency department with nausea, vomiting, dysuria, urina•·y urgency, and g radually worsening has had mild flank pain and discomfort for months that waxes and wanes in intensity. On further 2 flank pain. She indicates that she questioning she suspects she might have another urinary tract infection (UTI) because she has had recurrent UTis over the past few years 3 that have been treated with antibiotics. Urinalysis reveals no bacteria, but shows the presence of microscopic hematuria. X-ray of the abdomen is 4 shown in the image. The physician informs the patient that her present symptoms are probably the result of her past UTis. • 5 • 6 • 7

· 8 . 9 • 10 • 11 • 12 • 13 • 14 • 15 • 16 • 17 • 18 • 19 • 2 0 • 21 • a s 8 Lock Suspend End Block 5 24 • M k ~ £!1}>' !!":-~ Item: of ~ 1 ar -<:J I> • QIO: 4741 ..L Previous Next Lab lues Notes Calculator

.. - -- - • • .. .. - -- - F - - • - ---- 1 & &

2 3 4 • 5 • 6 . 7 · 8 . 9 • 10

• 11 • 12 • 13 • 14 • 15 • 16 • 17 . 18 • 19 • 20 • 21 • a s 8 Lock Suspend End Block • 10 • 11 • 12 Which of the following organisms most likely caused her previous UTis? • 13 : • 14 A. Enterococcus faecalis • 15 B. Escherichia coli • 16 • 17 C. Proteus mirabilis

• 18 D. Pseudomonas aeruginosa • 19 E. Serratia marcescens • 20 • 21 • a s 8 Lock Suspend End Block Item: 5 of 24 ~ 1 • M k -<:J 1>- Jil ~· !:';-~ QIO: 4741 ..L a r Previous Next Labfli!llues Not es Calculat o r

1 & & Th e correct an swer is C. 6 30/o ch ose this. 2 As indicated by the circle, the abdominal film in this patient shows the presence of a stag horn 3 calculus, most commonly a struvite-based stone. These stones are usually formed under heavily 4 alkalinizing conditions in the urine. Recurrent UTis do not typically cause alkalinization of urine, except for UTis caused by organisms that produce urease, an enzyme that cleaves urea to produce 5 alkaline ammonia, thereby increasing the pH of the urine. Urease-producing bacteria most commonly • 6 include bacteria of the Proteus and Klebsiella genera, and recurrent UT!s with these organisms are strongly associated with a preponderance of struvite stones. • 7 Urease Ammonoa Struvite Urea Enzyme PH Proteus (bacterium) Klebsiella Urine Alkalinity Bacteria · 8 Urinary tract infection . 9 • 10 • 11 • 12 • 13 • 14 • 15 A i s not correct. 30/o c hose this • • 16 Enterococcus faecalis is a g ram-negative bacteria that is found in the gastrointestinal flora and is known to be highly resistant to antibiotics . However, E. faecalis is unlikely to cause struvite stones or outpatient UTis . • 17 Enterococcus faecalis Gut flora Gram-negative bacteria Struvite Antibiotics Enterococcus Bacteria Gastrointestinal tract Human gastrointestinal tract • 18 Urinary tract infection

• 19 B is n ot correct . 270/o ch ose this. • 20 Escherichia coli is the leading cause of community-acquired UTI and is a gram-negative bacillus. E. coli, however, is not a urease-producing organism, and E. coli-based UTis are not associated with struvite stones . • 21 • Escheo choa co Gram-neqative bacteria Struvite Bacillus Bacillus (shape) Urina v t oct tnfection Orqanism a s 8 Lock Suspend End Block Item:5of24 ~ . , . M k <:] t> al ~· ~ QIO: 4741 .l. ar Previous Next Lab'lifllues Notes Calculator -- ...... ,._ ... _.,. _ ...... -- _.,.- ... _.,..,._ - -- - ... _.,. -- • 1 However, E. faecalis is unlikely to cause struvite stones or outpatient UT!s. 2 Enterococcus faecalis Gut flora Gram-negative bacteria Struvite Antibiotics Enterococcus Bacteria Gastrointestinal tract Human gastrointestinal tract Urinary tract infection 3 4 B is not correct. 270/o c hose t his. Escherichia coli is the lead ing cause of community-acquired UTI and is a g ram-negative bacillus. E. coli, however, is not a urease-producing 5 organism, and E. coli- based UT!s are not associated with struvite stones. • 6 Escherichia coli Gram-negative bacteria Struvite Bacillus Bacillus (shape) Urinary tract infection Organism

• 7 0 is not correct. 4 0/o c hose t his. • 8 Pseudomonas aeruginosa is a g ram -negative, aerobic bacillus usually associated with hospital-acquired or d rug-resistant UT!s. It produces a blue-green pigment and fruity odor. Given the outpatient setting for this case, P. aeruginosa is much less likely to be the cause of this patient's • 9 UTI. • 10 Pseudomonas aeruginosa Gram-negative bacteria Pseudomonas Aerobic organism Bacillus (shape) Bacillus Pigment Drug resistance Antimicrobial resistance · 11 Hospital-acquired pneumonia • 12 E is not correct. 30/o c hose t his . Serratia marcescens is often associated with hospital-acquired and d rug-resistant , and certain Serratia organisms produce a red • 13 pigment that may cause d iscoloration of the urine. However, this organism is not associated with urease production or the presence of renal • 14 calculi. • 15 Urease Serratia marcescens Serratia Urine Pigment Kidney stone Drug resistance Organism • 16 • 17 Bottom Line : • 18 Recu rrent UT!s with urease-producing bacteria are strongly associated with a preponderance of struvite stones because the urease alkalinizes the urine . • 19 Urease Struvite Urine Alkalinity Bacteria Urinary tract infection • 20 • 21 • 6 s 0 lock Suspend End Block Item: 5 of 24 ~ 1 • M k -<:J 1>- Jil ~· !:';-~ QIO: 4741 ..L a r Previous Next Labfli!llues Not es Calculat o r

& & 1 FA17 p 177.2 2 UTI bugs 3 SPECIES FEATURES COMMENTS 4 Escherichia coli Leading cause of UT I. Colonies shO\\ green Diagnostic markers: 5 metallic sheen on E~IB aga r. Ef> Leukoc)t e esterase = e\'idence of \V BC aCti\'itY. • 6 Staphylococcus 2nd leading cause of UT I in sexually acli\e saprophyticus women. Ef> 'il ril e lest = reduction of urinar) nil rates . 7 by bacterial species (eg, E coli). Klebsiella pneumoniae 3rd leading cause of UTI. Large mucoid capsule · 8 Ef> Urease test = urease-producing bugs (eg, and \'iscous colonies. . 9 S saprophyticus, Proteus, Klebsiella). Serratia marcescens Some strains produce a red pigment; often • 10 nosocomial and drug resistant. • 11 Enterococcus Often nosocomial and drttg resistant. • 12 Proteus mirabilis Moti lity causes '·swarming" on aga r; produces • 13 urease; associated with struvitc stones . • 14 Pseudomonas Blue-green pigment and frttity odor; usually • 15 aeruginosa nosocomial and drug resistant. • 16 • 17 FA17p567.1 . 18 Kidney Can lead to se\·ere complications such as hydronephrosis, pyelonephritis. Presents with unilateral Aank • 19 stones tenderness, co licky pain radiating to groin , hematuria. Treat and pre\'ent by encouraging Au id int·ake. • 20 \!1 ost common kidney stone presentation: calcium oxalate stone in patient with hypercalciuria and • 21 normocalcemia . • a s 8 Lock Suspend End Block Item: 6 of 24 ~ 1 • M k -<:J 1>- Jil ~· !:';-~ QIO: 4745 ..L a r Previous Next Labfli!llues Notes Calculat o r

1 & & A 75-year-old male with end-stage presents to his primary care physician for right leg pain. He states the pain began 2 yesterday after he hit his right leg on his kitchen table. He states that he has had bone pain, joint pain, and multiple fractures since becoming is able to pinpoint the leg pain to the middle of his right femur. Suspecting a 3 dialysis-dependent 3 years ago. On examination, the patient fracture, the physician orders right femur films. 4

5 The loss of which renal function is involved in this condition? • 6 : • 7 A. Ability to concentrate urine at t he d istal tubule · 8 B. Ability to excrete phosphate . 9 • 10 C. Production of erythropoiet in • 11 0. Reabsorption of calcium at the proximal tubule

• 12 E. Secretion of renin • 13 • 14 • 15 • 16 • 17 • 18 • 19 • 20 • 21 • a s 8 Lock Suspend End Block Item: 6 of 24 ~ . I • M k <:] t> al ~· ~ QIO: 4745 .l. ar Previous Next Lab'lifllues Notes Calculator • 1 The correct answer is B. 500/o chose this. 2 The red uced g lomerular filtration rate in chronic renal d isease leads to the accu m ulation of phosphate, and retained phosphate d irectly stim ulates 3 parathyroid hormone {PTH} prod uction (secondary hyperparathyroid ism). In addition, increased serum phosphate d irectly suppresses the renal prod uction of vitam in D3, lead ing to red uced serum calciu m levels. Chronic renal d isease also affects a 1-hyd roxylation of vitamin D2, the second 4 hyd roxylation step necessary to prod uce active vitamin D3. These changes can lead to alterations in bone metabolism, a cond ition known as 5 chronic kid ney d isease/ m ineral bone d isease {CKD-MBD). The patient's symptoms of j oint pain, bone pain, and m ultiple bone fractures in the setting of chronic kid ney d isease are characteristic of CKD-MBD. These symptoms will manifest after a few years on d ialysis. Furthermore, 6 increased seru m phosphate binds free calcium, which can lead to hypocalcemia, which in turn leads to increased PTH prod uction (secondary • 7 hyperparathyroid ism ). Hypocalcaemia Parathyroid hormone Renal function Chronic kidney disease Vitamin D Kidney disease Hyperparathyroidism Parathyroid gland Dialysis Calcium • 8 Kidney Bone remodeling Glomerulus Hormone Arthralgia Secondary hyperparathyroidism Blood plasma Phosphate Bone Hydroxylation Bone pain Metabolism • 9 Chronic kidney disease-mineral and bone disorder Vitamin Serum (blood) Bone disease Serum calcium • 10 A is not correct. 20/o chose this. · 11 Ind irectly, the ability to excrete phosphate and the conseq uent seru m hyperphosphatemia that g ives rise to secondary hyperparathyroid ism may • 12 be related to the inability of the kid ney to concentrate urine in patients with chronic renal failure. However, parathyroid hormone prod uction is not d irectly related to the ability to concentrate urine . • 13 Hyperphosphatemia Parathyroid hormone Hyperparathyroidism Chronic kidney disease Secondary hyperparathyroidism Phosphate Kidney Urine Serum (blood) • 14 Hormone Parathyroid gland Blood plasma

• 15 C is not correct. 60/o chose this. • 16 Prod uction of erythropoietin by the kid ney is im portant for maintaining prod uction of RBCs. In chronic renal failuren, a ormocytic normochromic anemia (anemia of chronic d isease) is caused by the loss of erythropoietin. However, loss of erythropoietin has no effect on PTH prod uction or • 17 serum calciu m homeostasis . • 18 Anemia of chronic disease Erythropoietin Chronic kidney disease Kidney Normochromic anemia Anemia Blood plasma Homeostasis Chronic condition Calcium • 19 Normocytic anemia • 20 0 is not correct. 400/o chose this . • 21 Calcium is normally passively reabsorbed at the proximal convoluted tubule. Theoretically a deficiency in reabsorption could lead to decreased • 6 s 0 lock Suspend End Block Item: 6 of 24 ~ . I • M k <:] t> al ~· ~ QIO: 4745 .l. ar Previous Next Lab'lifllues Notes Calculator . ~,. ~.~ . ~ ... ,. .. ~ .. ~ ~ 1 Hyperphosphatemia Parathyroid hormone Hyperparathyroidism Chronic kidney disease Secondary hyperparathyroidism Phosphate Kidney Urine Serum (blood) 2 Hormone Parathyroid gland Blood plasma

3 C is no t correct. 6 0/o c hose this. 4 Production of erythropoietin by the kidney is im portant for maintaining production of RBCs. In chronic renal failure, a normocytic normochromic 5 anemia (anemia of chronic d isease) is caused by the loss of erythropoietin . However, loss of erythropoietin has no effect on PTH production or serum calcium homeostasis. 6 Anemia of chronic disease Erythropoietin Chronic kidney disease Kidney Normochromic anemia Anemia Blood plasma Homeostasis Chronic condition Calcium • 7 Normocytic anemia • 8 0 is no t correct. 400/o c hose this . • 9 Calcium is normally passively reabsorbed at the proximal convoluted tubule. Theoretically a deficiency in reabsorption could lead to decreased serum calcium levels and parathyroid hormone secretion. However, this is not what is described commonly in renal failure, as the ability of the • 10 to reabsorb calcium is not documented as one of the maj or contributors to hypocalcemia in chronic renal failure. · 11 Nephron Hypocalcaemia Proximal convoluted tubule Parathyroid hormone Chronic kidney disease Calcium Hormone Blood plasma Parathyroid gland • 12 Serum calcium Serum (blood) Anatomical terms of location Kidney Secretion Reabsorption • 13 E is no t correct. 20/o c hose this . Secretion of ren in by the kidney is responsible for renal regulation of blood pressu re. Ren in facilitates the conversion of angiotensinogen to • 14 angiotensin, which regulates vascular tone and is important in the control of blood pressu re. The ren in -angiotensin system has no role in calcium • 15 metabolism or parathyroid hormone production . Angiotensinogen Renin-angiotensin system Parathyroid hormone Renin Blood pressure Hormone Kidney Parathyroid gland Metabolism Calcium Angiotensin • 16 Calcium metabolism Secretion Vascular resistance Blood vessel • 17 • 18 • 19 Bo tto m Line : • 20 Secondary increase in parathyroid hormone production is caused primarily by the reduction in phosphate excretion . Parathyroid hormone Parathyroid gland Phosphate Hormone Excretion • 21 • 6 s 0 lock Suspend End Block Item: 6 of 24 ~ 1 • M k -<:J 1>- Jil ~· !:';-~ QIO: 4745 ..L a r Previous Next Labfli!llues Notes Calculat o r

& & 1 FA17 p 332.1 2 Hyperparathyroidism 3 Primary Usually due to parathyroid adenoma or Osteitis fibrosa cystica-cystic bone spaces 4 hyperparathyroidism hyperplasia. Hypercalcemia, hypercalciuria fill ed with brown fibrous tissue · ("br0\\11 5 (renal stones), polyuria (thron e~) , tumor'' consisting of osteoclas ts and deposited hypophosphatemia, t PTI I, t ALP, t cA ~ I P in hemosiderin from hemorrhages; causes 6 urine. \ lost often asymptomatic. \ lay present bone pain). Due to t PTH, classicall}' 0 7 with weakness and consti pa tion (''groan<,"), associated \\ ith 1° (but also seen \\ it h 2°) o8 abdominaJ/flank pain (kidne) stones, acute h) perparathyroidism). . 9 pancreatitis), depression ("psychiatric "Stones, thrones, bones, groans, and • 10 O\Crtoncs"). p~ychiatric O\erlones."

• 11 Secondary zoh yperplasia due to ! Ca2+ absorption Renal osteodystrophy- renal disease ..... zoa nd • 12 hyperparathyroidism and/o r t P043- , most oft en in chronic 3° h) perparathyroidism ..... bone lesions. • 13 renal disease (causes hypovitaminosis 0 2 0 14 and hyperphosphatemia ..... ! Ca +). • 15 Hypocalcemia, hyperphosphatcmia in chronic renal failure (vs hypophosphatemia 0 16 with most other causes), t ALP, t PTH. 0 17 Tertiary Refractory (autonomous) hyperparalh) roidism . 18 hyperparathyroidism resulting from chronic renal disease. t t PTH, • 19 t Ca2+ . • 20

• 21 FA17 p 571 .3 • a s 8 Lock Suspend End Block Item: 7 of 24 ~ 1 • M k -<:J 1>- Jil ~· !:';-~ QIO: 4750 ..L a r Previous Next Labfli!l tues Not es Calculat o r

1 & & A 55-year-old man admitted to the hospital because of complications of alcoholic cirrhosis and concurrent C infection progressively 2 develops an increase in serum blood urea nitrogen and creatinine levels, followed closely with urine output of <300 mL/day. Urinalysis of 5 mEq/L and the presence of benign urinary sediment. No proteinuria or hematuria is present. 3 reveals a sodium concentration 4 Imaging of this patient's kidneys is likely to reveal which of the following? 5

6 A. Enlarged cystic kidneys 0 7 o8 B. Enlarged kidneys with severe hydronephrosis

. 9 C. Horseshoe kidneys • 10 D. Normal kidneys with "flea-bitten" infarcted appearance • 11 E. Normal size and shape • 12

• 13

0 14

• 15

0 16

0 17 • 18 • 19

• 20

• 21 • a s 8 Lock Suspend End Block Item: 7 of 24 ~ . I • M k <:] t> al ~· ~ QIO: 4750 .l. ar Previous Next lab 'lifllues Notes Calculator

1 The correct answer is E. 390/o chose this. 2 The patient described in this vignette has , which is a progressive functional renal failure caused by a reduction in the 3 g lomerular filtration rate due to declining hepatic function. It is characterized by splanchnic vasodilation and concomitant vasoconstriction in the 4 renal vascular beds due to the production of mediators and to the activation of the ren in -angiotensin system. The combination of these two factors causes a prerenal type azotemia that develops, most commonly, without severe oliguria. One of the features of hepatorenal syndrome is 5 that kidney anatomy is completely unaffected, and thus visualization of the kidneys by most modalities would reveal normal size and shape. 6 Hepatorenal syndrome Oliguria Renin-angiotensin system Renal function Vasodilation Vasoconstriction Azotemia Kidney Glomerulus liver Blood vessel 7 A is not correct. 11 Ofo chose this. • 8 Enlarged cystic kidneys can be a feature of polycystic kidney d isease, but usually do not present with prerenal azotemia . Azotemia Polycystic kidney disease Kidney Kidney disease • 9 B is not correct. 200/o chose this . • 10 Enlarged kidneys with hydronephrosis can be caused by obstruction from stones in the renal calyx or ureters. However, the other clinical findings · 11 in this patient implicate a hepatorenal (prerenal} cause of renal failure. • 12 Hydronephrosis Ureter Kidney • 13 C is not correct. 4 0fo chose this . Horseshoe kidney is a ped iatric abnormality that is generally asymptomatic if not associated with other abnormalities. Horse-shoe kidneys are • 14 caused by the embryon ic blastemas (embryonic kidneys) partially fusing at their inferior poles and becoming trapped against the inferior • 15 mesenteric artery during their migration. In children it is sometimes associated with Wilms' tumor. • 16 Horseshoe kidney Inferior mesenteric artery Wilms' tumor Kidney Pediatrics Horseshoe Asymptomatic Neoplasm • 17 0 is not correct. 260/o chose this . A flea-bitten appearance in kidneys is usually seen in patients with malignant hypertension . • 18 Hypertensive emergency Hypertension Kidney Cancer Malignancy • 19 • 20 Bottom Line: • 21 r , r ·• . ' ,.. ' ' . ~ ·· 6 s 0 lock Suspend End Block Item: 7 of 24 ~ . I • M k <:] t> al ~· ~ QIO: 4750 .l. ar Previous Next lab 'lifllues Notes Calculator

~ • 'f... • ,~ ~ • ~' • • • ~ • • • • ~. 'f.. • • ~ ~ .... ~ ,. ~ ~ • ' • • • ~ ~ "• 1 factors causes a prerenal type azotemia that develops, most commonly, without severe oliguria. One of the features of hepatorenal syndrome is 2 that kidney anatomy is completely unaffected, and thus visualization of the kidneys by most modalities would reveal normal size and shape. Hepatorenal syndrome Oliguria Renin-angiotensin system Renal function Vasodilation Vasoconstriction Azotemia Kidney Glomerulus liver Blood vessel 3 4 A is not correct. 11 Ofo c hose t his. Enlarged cystic kidneys can be a feature of polycystic kidney d isease, but usually do not present with prerenal azotemia. 5 Azotemia Polycystic kidney disease Kidney Kidney disease 6 B is not correct. 200/o c hose t his. 7 Enlarged kidneys with hydronephrosis can be caused by obstruction from stones in the renal calyx or ureters. However, the other clinical findings • 8 in this patient implicate a hepatorenal (prerenal} cause of renal failure . Hydronephrosis Ureter Kidney • 9 C is not correct. 4 0/o c hose t his • • 10 Horseshoe kidney is a ped iatric abnormality that is generally asymptomatic if not associated with other abnormalities. Horse-shoe kidneys are · 11 caused by the embryon ic blastemas (embryonic kidneys) partially fusing at their inferior poles and becoming trapped against the inferior • 12 mesenteric artery during their migration. In children it is sometimes associated with Wilms' tumor. Horseshoe kidney Inferior mesenteric artery Wilms' tumor Kidney Pediatrics Horseshoe Asymptomatic Neoplasm • 13 0 is not correct. 260/o c hose t his • • 14 A flea-bitten appearance in kidneys is usually seen in patients with malignant hypertension . • 15 Hypertensive emergency Hypertension Kidney Cancer Malignancy • 16 • 17 Bottom Line : • 18 Hepatorenal syndrome is a prog ressive functional renal failure caused by a reduction in the g lomerular filtration rate due to declining hepatic • 19 function. The kidney anatomy is completely unaffected, and thus visualization of the kidneys by use of most modalities would reveal normal size • 20 and shape . Hepatorenal syndrome Renal function Glomerulus Kidney liver • 21 • 6 s 0 lock Suspend End Block Item: 7 of 24 ~ 1 • M k -<:J 1>- Jil ~· !:';-~ QIO: 4750 ..L a r Previous Next Labfli!l tues Not es Calculat o r

1 & &

2 FA17 p 571 .2 3 Acute kidney injury Acute kidney injury is de fin ed ns an abrupt decl ine in rennl functi on as measured by t creatinine 4 (acute renal failure) and t BU1 or by oliguria/anuria. 5 Prerenal azotemia Due to l RBF (eg, hypotemion) - l CFR. i':a+/ 1110 and BU1 retained by kidn ey in an attempt to 6 consen e ,-olume - t BU1 /creatinine ratio (BUN is reabsorbed, crea ti nine is not) and l FE\Ia· 7 Intrinsic renal failure Cenernllr due to acute tubular necrosis or ischemia/toxins; less commonlr due to acute · 8 glomerulonephritis (eg, RPCN, hcmol) Lie uremic S) nd rome) or ac ute interstitial nephritis. . 9 In ATN, patchy necrosis - debr is obstructing tubule and Auid backAow across necrotic tubule - l C FR. Urine has epithel ial/granular casts. BUi': re 500 < 350 < 350 (mOsm/kg) • 16 Urine Na+ (mEq/L) < 20 > 40 > 40 • 17 < 1% >2% < 1% (mild) • 18 FENa > 2% (severe) • 19 Serum BUN/Cr > 20 < 15 Varies • 20 • 21 • a s 8 Lock Suspend End Block Item: 8 of 24 ~ 1 • M k -<:J 1>- Jil ~· !:';-~ QIO: 4696 ..L a r Previous Next Labfli!l tues Notes Calculat o r

1 & & A 22-year-old woman presents to the clinic with symptoms of left-sided back pain and blood visible at the end of the urine stream. Urinalysis 2 shows a specific gravity of 1.010 and a pH of 7 .9; results are positive for nitrite and WBC esterase, but negative for ketones and glucose. shows many RBCs and many bacteria. Culture of urine sample produced swarming colonies on agar plate. 3 Microscopic examination of urine 4 Which of the following is the most likely underlying cause of the patient's symptoms? 5

6 A. Ammonium magnesium phosphate stone(s) 7 · 8 B. Calcium oxalate stone(s)

. 9 C. Calcium phosphate stone(s) • 10 D. Cystine stone(s) • 11 E. Uric acid stone(s) • 12

• 13 • 14

• 15

• 16

• 17

• 18 • 19

• 20

• 21 • a s 8 Lock Suspend End Block Item:8of24 ~ . , . M k <:] t> al ~· ~ QIO: 4696 .l. ar Previous Next lab 'lifllues Notes Calculator

1 • The correct answer is A. 680/o chose this. 2 This patient has symptoms and signs consistent with nephrolithiasis. Her urinalysis resu lts are consistent with a concomitant urinary tract 3 infection with an organism that is urease positive (eg, Proteus or Klebsiella}, and the swarming appearance on the agar suggests a Proteus 4 infection . The resu ltant increase in urinary pH provides a suitable environment for the formation of ammonium magnesium phosphate (also known as struvite) stones. Struvite stones are rad iopaque and often located in the renal calyces; they may g row into large stones, sometimes 5 extending throughout the entire collecting system (known as a stag horn calculi). Treatment includes hydration, pain control, and appropriate 6 antibacterial therapy and prophylax is, but larger stones frequently require either surgical removal or lithotripsy (using shock waves to break up the stones). Chemical d issolution with a urease-inhibiting d rug is frequently ineffective. These urine crystals have a characteristic coffin lid 7 appearance. 8 Urinary tract infection Renal calyx Urease Urinalysis Struvite PH Kidney stone Urinary system Magnesium Extracorporeal shock wave lithotripsy Klebsiella Urine • 9 Radiodensity Antibiotics Phosphate Ammonium Preventive healthcare Proteus (bacterium) • 10 B is not correct. 120/o chose this . · 11 Calcium oxalate stones are the most common type of kidney stone and resu lt from hypercalciuria. These stones are rad iopaque and occu r in the setting of renal tubular acidosis and hyperparathyroidism. Therapy consists of hydration, pain control, and treatment of the underlying cause of • 12 hypercalciuria. The urine crystal for calcium oxalate looks like an envelope or dumbbelL • 13 Kidney stone Hypercalciuria Calcium oxalate Hyperparathyroidism Renal tubular acidosis Oxalate Calcium Urine Radiodensity Kidney Acidosis Analgesic • 14 C is not correct. 60/o chose this . • 15 Calcium phosphate stones resu lt from hypercalciuria. These stones are rad iopaque and occu r in the setting of renal tubular acidosis or hyperparathyroidism. Therapy consists of hydration, pain control, and treatment of the underlying cause of hypercalciuria. Calcium phosphate • 16 crystals are wedge-shaped prisms. • 17 Hypercalciuria Calcium phosphate Hyperparathyroidism Renal tubular acidosis Calcium Phosphate Radiodensity Acidosis Kidney • 18 0 is not correct. 60/o chose this . Cystine stones resu lt from inappropriate secretion of cystine into the urine due to an autosomal recessive defect in amino acid transport. Cystine • 19 stones are rad iolucent. Treatment consists of hydration, pain control, alkalinization of the urine, and therapy with a cystine-binding medication . • 20 Fo r a picture of a cystine stone please refer to the first aid reference below. Cystine stones have a characteristic hexagonal shape. Amino acid Cystine Urine Autosomal recessive Dominance (genetics) Autosome Radiodensity Secretion Recessive • 21 • 6 s 0 lock Suspend End Block Item:8of24 ~ . , . M k <:] t> al ~· ~ QIO: 4696 .l. ar Previous Next lab 'lifllues Notes Calculator

1 B is no t correct . 1 20/o c hose t his.

2 Calcium oxalate stones are the most common type of kidney stone and resu lt from hypercalciuria. These stones are rad iopaque and occu r in the setting of renal tubular acidosis and hyperparathyroidism. Therapy consists of hyd ration, pain control, and treatment of the underlying cause of 3 hypercalciuria. The urine crystal for calcium oxalate looks like an envelope or dumbbelL 4 Kidney stone Hypercalciuria Calcium oxalate Hyperparathyroidism Renal tubular acidosis Oxalate Calcium Urine Radiodensity Kidney Acidosis Analgesic 5 C is no t correct . 60/o c hose t his. 6 Calcium phosphate stones resu lt from hypercalciuria. These stones are rad iopaque and occu r in the setting of renal tubular acidosis or hyperparathyroidism. Therapy consists of hydration, pain control, and treatment of the underlying cause of hypercalciuria. Calciu m phosphate 7 crystals are wedge-shaped prisms. 8 Hypercalciuria Calcium phosphate Hyperparathyroidism Renal tubular acidosis Calcium Phosphate Radiodensity Acidosis Kidney • 9 0 is no t correct . 60/o c hose t his . • 10 Cystine stones resu lt from inappropriate secretion of cystine into the urine due to an autosomal recessive defect in amino acid transport. Cystine stones are rad iolucent. Treatment consists of hydration, pain control, alkalinization of the urine, and therapy with a cystine-binding medication. · 11 For a picture of a cystine stone please refer to the first aid reference below. Cystine stones have a characteristic hexagonal shape. • 12 Amino acid Cystine Urine Autosomal recessive Dominance (genetics) Autosome Radiodensity Secretion Recessive • 13 E is no t correct . 80/o c hose t his . • 14 Uric acid stones resu lt from hyperuricemia associated with increased cellular turnover (eg, gout, leu kemia). These stones are rad iolucent. Treatment consists of hydration, pain control, and alkalinization of the urine. For a picture of uric acid stone, refer to the First Aid reference • 15 below. The urine crystal will show the characteristic rhomboid shape. • 16 Hyperuricemia Uric acid Gout Urine Crystal Radiodensity Analgesic leukemia Management of dehydration • 17 • 18 Bo tto m Line : • 19 Organisms that are urease positive (eg, Proteus or Klebsiella} increase urinary pH and promote the formation of ammonium magnesium phosphate stones . • 20 Urease PH Magnesium Klebsiella Phosphate Ammonium Proteus (bacterium) Proteus Urine • 21 6 s 0 lock Suspend End Block Item:8of24 ~ . , . M k <:] t> al ~· ~ QIO: 4696 .l. ar Previous Next lab 'lifllues Notes Calculator

1 • • 2 FA17 p 567.1 3 Kidney Can lead to se,·ere compl ications such as hyd roneph rosis, pyelonephritis. Presents with unilateral Aank 4 stones tenderness, colicky pain radiating to groin, hematuria. Treat and prevent by encouraging Auid intake. Most common kidney stone presentation: calcium oxa late stone in patient with hypercalciuria and 5 normoca lcem ia. 6 CONTENT PRECIPITATES WITH X-RAY FINDINGS CT FINDINGS URINEC RYSTAL NOTES 7 Calcium Calcium Radiopaque Radiopaque Shaped like Calcium stones most common (80%); calcium 8 oxalate: envelope rJ oxalate more common than calcium • 9 hypocitraturia or dumbbell phosphate stones. Hypocitraturia often associated with ! urine pH . • 10 Can result from ethylene glycol (antifreeze) · 11 ingestion, vitamin C abuse, hypocitraturia, • 12 malabsorption (eg, Crohn disease). • 13 Treatment: th iazides, ci trate, low-sodium diet. • 14 Calcium Radiopaque Radiopaque Wedge- Treatment: low-sodium diet, thiazides. • 15 phosphate: shaped t pH pnsm • 16 Ammonium t pH Radiopaque Radiopaque Coffin lid I] Also known as struvite; account for 15% of • 17 magnesium stones. Caused by infection with urease® • 18 phosphate bugs (eg, Proteus mirabilis, Staphylococcus • 19 saprophyticus, Klebsiella) that hydrolyze • 20 urea to ammonia ... urine alkalinization . Commonly form staghorn ca l cul i ~ . • 21 • T ..o.,.hYu '>l"''._· Ol"•:.rl ; ,... ..,,.;l'\1"'1 nf ••nrli"',.J\Iinrr inf,...,...tinn • 6 s 0 lock Suspend End Block Item: 9 of 24 ~ 1 • M k -<:J 1>- Jil ~· !:';-~ QIO: 5 054 ..L a r Previous Next Labfli!l tues Not es Calculat o r

1 & & A 23-year-old African-American woman with an unknown past medical history presents to the emergency department with a 3-day history of 2 gross hematuria. She denies any recent illness or any history of urinary tract infections. Upon further questioning, she reveals that she has fatigue and malaise over the past few months with intermittent joint pain. She also notes that she gets a 3 been having gradually increasing rash on the bridge of her nose that occurs spontaneously and resolves on its own. On examination she is hypertensive with significant proteinuria 4 and RBC casts on urinalysis. laboratory tests reveal a creatinine level of 2.8 mg/ dl. 5 6 What is the most likely underlying pathophysiology of her condition?

7 : 8 A. Aggregation of light chains within t he renal mesangium

. 9 B. Deposition of immune complexes wit hin the renal glomeruli • 10 C. Effacement of foot processes supporting t he renal epithelial podocytes • 11 D. Loss of heparan sulfate moieties that form the renal tubule filtration barrier • 12 • 13 E. Neutrophilic infiltration of the tubules • 14 • 15 • 16 • 17 • 18 • 19 • 20 • 21 • a s 8 Lock Suspend End Block Item: 9 of 24 ~ 1 • M k -<:J 1>- Jil ~· !:';-~ QIO: 5 054 ..L a r Previous Next Labfli!l tues Not es Calculat o r

1 & & Th e c o rrect a n sw er i s B. 820/o ch ose this. 2 nephritis is a common and serious complication of systemic lupus erythematosus (SLE). It is 3 more common and severe in African-American female adolescents. This condition often manifests clinically with proteinuria; hypertension; varying degrees of renal failure; arthritic pain; and active 4 urine sediment with RBCs, WBCs, and cellular casts. Typically, patients will present with more I 5 gradual deterioration over several months with common symptoms, including fever, weight loss, and malaise. On physical exam, one of the classic skin findings is the malar rash, though this is only 6 found in half of patients with SLE. , however, is necessary to confirm the diagnosis of 7 and classify the different glomerulopathies that have prognostic and management implications. Focal segmental glomerulosclerosis (FSGS) is an important differential for patients with 8 possible lupus nephritis because FSGS occurs more commonly in patients with SLE. Lupus nephritis 9 results from the deposition of glomerular immune complexes, which activate the complement cascade and lead to complement-mediated damage. In contrast, FSGS has appearances of • 10 segmental sclerosis and hyalinosis. Treat ment of lupus nephritis consists of high-dose steroids • 11 combined with either cyclophosphamide or mycophenolate mofetil. • 12

• 13 The image shows the glomerular "wire loop" lesions (thickened, pink, hyalinized capillary loops) caused by circumferential subendothelial immune • 14 complex deposition characteristic of SLE . • 15 Mycophenolic acid Systemic lupus erythematosus Lupus nephritis Cyclophosphamide Immune complex Proteinuria Focal segmental glomerulosclerosis Renal biopsy Nephritis Complement system Malaise Hypertension Biopsy Malar rash Glomerulus Glomerulus (kidney) Rash Urine Lupus erythematosus Capillary Arthritis • 16 Fever Steroid Kidney African Americans Glomerulosclerosis Physical examination • 17 A i s not co rrect. ]Ofo ch ose this • • 18 Multiple myeloma is the neoplastic proliferation of plasma cells that secrete a monoclonal • 19 immunoglobulin. Patients with multiple myeloma are usually older adults who present with anemia, • 2 0 bone pain, frequent infections, hypercalcemia, lytic lesions in bone that lead to pathologic fractures, and renal failure secondary to the production of nephrotoxic light chains that precipitate in the renal • 21 • tubules. Given this patient's age, race, and clinical findings, her presentation is most consistent with a s 8 Lock Suspend End Block Item: 9 of 24 ~ 1 • M k -<:J 1>- Jil ~· !:';-~ QIO: 5054 ..L a r Prev ious Next Lab fli!ltues Not es Calculat or

& & 1 Fever Steroid Kidney African Americans Glomerulosclerosis Physical examination

2 A i s n ot correct . ]Ofo ch ose this. 3 Multiple myeloma is the neoplastic proliferation of plasma cells that secrete a monoclonal multiple myeloma are usually older adults who present with anemia, 4 immunoglobulin. Patients with bone pain, frequent infections, hypercalcemia, lytic lesions in bone that lead to pathologic fractures, 5 and renal failure secondary to the production of nephrotoxic light chains that precipitate in the renal with 6 tubules. Given this patient's age, race, and clinical findings, her presentation is most consistent lupus nephritis. 7 The image shows the granular protein intratubular casts characteristic of myeloma kidney. 8 Multiple myeloma Hypercalcaemia Lupus nephritis Nephrotoxicity Anemia Proteon Ant•body Nephritis Neoplasm 9 Systemic lupus erythematosus Plasma cell Kidney Renal tubules Lysis Blood plasma Pathology Bone Bone pain • 10 Lytic cycle Immunoglobulin light chain Lupus erythematosus • 11 • 12 C is not correct. 4 0fo chose t his. • 13 Effacement of foot processes supporting the renal epithelial podocytes seen on electron microscopy (shown in this image) is the classic find in g in minimal change disease (MCD), which is the most • 14 common cause of nephrotic synd rome in children. Patients with MCD typically have nephrotic-range • 15 proteinuria, hypoalbuminemia, hyperlipidemia, and edema. They do not have RBCs in the urine or renal failure. This patient's clinical features are typical of acute nephritis. Hence, it is much more • 16 likely that this patient is suffering from lupus nephritis than from MCD. • 17 Nephrotic syndrome Lupus nephritis Hypoalbuminemia Minimal change disease Proteinuria Hyperlipidemia Nephritis Edema Urine Podocyte Electron microscope Systemic lupus erythematosus Epithelium Microscopy • 18 Kidney • 19 • 20 • 21 • a s 8 Lock Suspend End Block Item: 9 of 24 ~ 1 • M k -<:J 1>- Jil ~· !:';-~ QIO: 5054 ..L a r Prev ious Next Lab fli!ltues Not es Calculat or

& 1 D i s not correct. 4 0/o ch ose this. & 2 Diabetic nephropathy is the most common cause of chronic renal failure in the United States, accounting for 45% of patients initiating renal replacement therapy every year. Within 1-2 years 3 after the onset of clinical diabetes, morphologic changes begin to appear in the kidney. Specifically, 4 the composition of the glomerular basement membrane is altered in this condition, notably with a loss of heparan sulfate moieties that form the negatively charged filtration barrier. This change 5 results in the increased filtration of negatively charged serum proteins into the urine, predominately 6 albumin. Some patients also develop eosinophilic, periodic acid-Schiff-positive nodules known as microscopy often reveals the 7 Kimmelstiei-Wilson nodules (shown in this image). Immunofluorescence nonspecific linear deposition of albumin, IgG, or complement, but electron microscopy shows no 8 immune deposits. 9 Diab<>tic nephropathy Glomerular basement membrane Immunofluorescence Heparan sulfate Immunoglobulin G Chronic kidney disease Basement membrane Diabetes mellitus Albumin Kidney disease Electron microscope • 10 Human serum albumin Glomerulus Urine Kidney Blood plasma Glomerulus (kidney) Serum (blood) Protein Renal replacement therapy united states Microscopy • 11 E is not correct. JO/o chose t his . . 12 Tubular infiltration of polymorphonuclear WBCs is a common renal finding seen in pyelonephritis . • 13 Pyelonephritis is an infection of the genitourinary tract that typically affects women. It is usually • 14 caused by Escherichia coli, and its pathogenesis is related to the short length of the female urethra . Pyelonephritis should be differentiated from cystitis. Pyelonephritis often involves more constitutional • 15 symptoms, flank pain, and fever. WBC casts on urinalysis are pathognomonic for pyelonephritis. This • 16 patient exhibits signs and symptoms of nephritic synd rome rather than an overtly infectious process . • 17 The image illustrates acute pyelonephritis .

. 18 Pyelonephritis Escherichia coli Urinalysis urethra Genitourinary system • 19 Urinary tract infection Nephron Abdominal pain Pathogenesis Fever Nephritis Kidney Infection • 20 . 21 • a s 8 Lock Suspend End Block caused by Escherichia coli, and its pathogenesis is related to the Pyelonephritis should be differentiated from cystitis. Pyelonephritis often involves more constitutional 2 symptoms, flank pain, and fever. WBC casts on urinalysis are pathognomonic for pyelonephritis. This 3 patient exhibits signs and symptoms of nephritic syndrome rather than an overtly infectious process. 4 The image illustrates acute pyelonephritis.

5 Pyeloneph itis Escheroch•a col Nephritic syndrome Urinalysis urethra Genitourinary system Kidney Infection 6 Urinary tract Infection Nephron Abdominal pain Pathogenesis Fever Nephritis 7 8 9 • 10 • 11 • 12 • 13 • 14 • 15 • 16 Image courtesy of Dr. Michael Bonert • 17 • 18 Bottom Line: • 19 Lupus nephritis results from the deposition of glomerular immune complexes, which activate the complem ent cascade and lead to complement­ • 2 0 mediated damage. • 21 Lupus nephr tis Nephr'tis Complement system Immune complex Glomerulus System•c lupus erythematosus Glomerulus (kidney) Lupus erythematosus • a s 8 Lock Suspend End Block Item: 9 of 24 ~ 1 • M k -<:J 1>- Jil ~· !:';-~ QIO: 5 054 ..L a r Previous Next Labfli!l tues Not es Calculat o r

1 & & FA17 p 443.1 2 Systemic lupus erythematosus 3 SYMPTOMS Classic presentation: rash, joint pain, and fe,er, R \ SII OR P\1'~ : 4 most commonly in a female of reproductive Rash (malar · or discoid : ) 5 age and African- merican descent. \ rt hritis (nonerosi,·e) 6 Libman-Sacks Endocarditis- nonbacterial, Serositis 7 verrucous thrombi usuall) on mitral or aortic l lematologic disorders (eg, cytopenias) 8 ,-ah-e and can be present on either surface of Oral/nasopharyngeal ulcers the ,-a h-e (but usually on undersurface) (LSE Renal disease 9 in SLE). Photosensitivity • 10 Lupus nephritis (glomerular deposition of anti- Antinuclear antibodies • 11 0 1 A immune complexes) can be nephritic Immunologic disorder (anti-dsDl A, anti- m, • 12 or nephrotic (hcmaturi- Jil ~· !:';-~ QIO: 5 054 ..L a r Previous Next Labfli!l tues Not es Calculat o r

1 & & FA17 p 564.1 2 Nephritic syndrome l\ephrltic syndrome= InAammatory process. When glomeruli are involved, leads to hematuria 3 and RBC casts in urine. Associated with a~otemia, oliguria, hypertension (due to salt retention), 4 proteinuria. 5 Acute L.M -glomeruli enlarged and h) percellular rJ. lost frequently seen in children. Occurs 6 poststreptococca I IF' - ( starry sky") granular appearance - 2-4 weeks after group A streptococcal 7 glomerulonephritis ("lumpy-bumpy") [lJ due to lgC, lg\ I, and C3 infection of pharynx or skin. Resolves deposition along GB I and mesangium. spontaneous!). Type Ill h) persensiti' it) 8 E~1-subep i thelial immune complex (IC) reaction. 9 humps. Presents with peripheral and periorbital edema, • 10 cola-colored urine, hypertension. • 11 Positi,·e strep titers/serologies, l complement • 12 b ·cls (C3) due to consumption . • 13 Rapidly progressive LM and IF- crescent moon shape [!I. Crescents Poor prognosis. Rapidly deteriorating renal • 14 (crescentic) consist of fibrin and plasma proteins (eg, C3b) function (days to weeks). glomerulonephritis with glomerular parietal cells, monocytes, • 15 macro phages . • 16 Several d isease processes may result in this • 17 pallern, in particular: . 18 • -type II llematuria/hemoptysis. • 19 hrpersensitivity reaction; antibodies to Treatment: emergent plasmapheresis. CBl\1 and alveolar basement membrane • 20 -+ linear IF • 21 • Granulomatosis with oolvanl!iitis (\Vel!ener) PR3-AJ CA/c-ANCA. Pauci-immune (no l!!/C3 a s 8 Lock Suspend End Block Item: 9 of 24 ~ 1 • M k -<:J 1>- Jil ~· !:';-~ QIO: 5 054 ..L a r Previous Next Labfli!l tues Not es Calculat o r

1 & &

2 FA17 p566.1 3 Nephrotic syndrome f'\ephrO tic syndrome-massi\e prO teinuria {> 3.5 glday) with hypoalbuminemia, resulting 4 edema, h) perlipidemia. Frothy urine " ith fatly casts. Due to podOC) te damage disrupting 5 glomerular filtration charge barrier. 'vlay be 1° {eg, direct sclerosis of podocytes) or 2° {systemic process [eg, diabetes] secondarily damages podocytes). Associated with hypercoagulable stale {eg, 6 thromboembolism) due to antithrombin ( T ) Ill loss in urine and t risk of infection {due to loss of 7 immunoglobulins in urine and soft tissue compromise by edema). 8 Se,·ere nephritic syndrome may present with nephrotic S}lldrome features {nephritic-nephrotic 9 syndrome) if damage to C Bl\1 is severe enough to damage charge barrier. • 10 Minimal change LM-normal glomeruli {lipid may be seen in \!lost common cause of nephrotic S)l1drome • 11 disease (lipoid PCT cells). in children. Often 1° (idiopathic) and ma) be • 12 nephrosis) IF 8 . triggered by recent infection, immunization, EM-effacement of foot processes fl. immune stimulus. Rarely, may be 2° to • 13 lymphoma (eg, cytokine-mediated damage). 1° • 14 disease has excellent response to corticosteroids . • 15 Focal segmental LM-segmental sclerosis and hya li nosis [lJ. Most common cause of nephrotic sy ndrome in • 16 glomerulosclerosis IF- often 8, but may be (f) for nonspecific focal African Americans and Hispanics. Can be l0 • 17 deposits of Ig r, C3, C l. {idiopathic) or zo to other conditions {eg, H IV . 18 E If-effacement of foot process sim ilar to infection, sickle cell disease, heroin abuse, minimal change disease. massive obesity, interferon treatment, chronic I • 19 kidney disease due to congenital malfom1ations) . • 20 1° disease has inconsistent response to steroids . • 21 • ~lay progress to chronic renal disease . a s 8 Lock Suspend End Block Item: 10 of 24 ~ 1 • M k -<:J 1>- Jil ~· !:';-~ QIO: 4746 ..L a r Previous Next Labfli!llues Not es Calculat o r

1 & & A 27-year-old male medical student visits his physician complaining of several intermittent episodes of passing reddish-orange urine. A urine 2 dipstick test confirms the presence of blood in the urine. Several years ago, the patient was diagnosed with celiac disease, and a few days illness that consisted of cough and nasal congestion. He denies any other m edical problems. Urinalysis 3 ago he recovered from a flu-like reveals proteinuria and gross hematuria with numerous RBC casts. A renal biopsy is performed. 4

5 This patient's glomeruli are likely to have abundant deposition of which of the following? 6 : 7 A. Anti-glomerular basement membrane antibodies 8 B. C3 9 • 10 C. IgA • 11 0. IgG • 12 E. IgM • 13 • 14 • 15 • 16 • 17 • 18 • 19 • 2 0 • 21 • a s 8 Lock Suspend End Block Item: 10 of 24 ~ . I • M k <:] t> al ~· ~ QIO: 4746 .l. ar Previous Next lab'lifllues Notes Calculator

1 • The correct answer is C. 660/o chose this. 2 The proteinuria and g ross hematuria with RBC casts strongly implicate a g lomerulonephritis (nephritic) d isease as the cause of this patient's 3 illness. The recu rrent episodes of interm ittent hematuria, especially soon after an upper respiratory tract infection, suggest IgA nephropathy, or Berger d isease, which is associated with celiac d isease or liver abnormalities due to defective IgA clearance. In celiac d isease, the IgA antibodies 4 formed against g liad in deposits in the g lomeruli, lead ing to IgA nephropathy. Thus, a microscopic examination of g lomeruli from this patient is 5 likely to reveal mesangial proliferation with IgA deposits. Berger d isease should not be confused with thromboangiitis obliterans (Buerger d isease), a d istinct vasculitis associated with tobacco use. 6 Coeliac disease Hematuria Proteinuria IgA nephropathy Upper respiratory tract infection Glomerulonephritis Vasculitis Immunoglobulin A Urinary cast Antibody 7 Kidney disease Nephritic syndrome Glomerulus (kidney) Mesangium Glomerulus Respiratory tract Nephritis Nephron Thromboangiitis obliterans 8 Infection 9 A is not correct. SO/o chose this. 10 Anti-glomerular basement membrane (GBM} antibodies are antibodies d irected against the GBM. These depositions are a common finding in anti­ GBM d isease (Goodpasture syndrome), in which the g lomerulonephritis is associated with pulmonary hemorrhage and the presence of linear · 11 depositions consisting of anti-GBM antibodies. This patient has no pulmonary symptoms suggestive of Goodpasture syndrome and is unlikely to • 12 have anti-GBM antibodies . Pulmonary hemorrhage Basement membrane Antibody Glomerulonephritis Goodpasture syndrome Anti-glomerular basement membrane antibody Bleeding • 13 Cell membrane • 14 B is not correct. 11 Ofo chose this . • 15 C3 (complement) deposition can be associated with many g lomerular d iseases, includ ing anti-glomerular basement membrane d isease • 16 (Goodpasture syndrome), postinfectious g lomerulonephritis, and lupus nephritis. However, this patient's clinical picture suggests IgA • 17 nephropathy. In IgA nephropathy, there is no C3 complex formed with IgA, and we would not see any C3 deposition in the g lomeruli. Goodpasture syndrome lupus nephritis IgA nephropathy Basement membrane Nephritis Glomerulonephritis Immunoglobulin A Glomerulus Kidney disease • 18 Glomerulus (kidney) lupus erythematosus Systemic lupus erythematosus • 19 0 is not correct. 11 Ofo chose this . • 20 Granular or linear IgG depositions can be associated with or without C3 (complement) deposition in numerous g lomerulonephritides, but none of • 21 these d iseases is consistent with the clinical history of this patient. Typically IgG complexes are seen in d isease states like poststreptococcal • I •o • I • I ol • . ' . ' .' ' .. ' ' r ' 6 s 0 lock Suspend End Block Item: 10 of 24 ~ . I • M k <:] t> al ~· ~ QIO: 4746 .l. ar Previous Next lab'lifllues Notes Calculator

. .. ~.~ •••• ~,.. ~ u ~ •• ~ .~, ..... ~ u' ••• ~ .... ~. ~-~~ ~. ~ .. 1 depositions consisting of anti-GBM antibodies. This patient has no pulmonary symptoms suggestive of Goodpasture syndrome and is unlikely to 2 have anti-GBM antibodies. Pulmonary hemorrhage Basement membrane Antibody Glomerulonephritis Goodpasture syndrome Anti-glomerular basement membrane antibody Bleeding 3 Cell membrane 4 B is no t correct . 11 Ofo c hose t his. 5 C3 (complement) deposition can be associated with many g lomerular d iseases, includ ing anti-glomerular basement membrane d isease 6 (Goodpasture syndrome), postinfectious g lomerulonephritis, and lupus nephritis. However, this patient's clinical picture suggests lgA 7 nephropathy. In lgA nephropathy, there is no C3 complex formed with lgA, and we would not see any C3 deposition in the g lomeruli. Goodpasture syndrome lupus nephritis IgA nephropathy Basement membrane Nephritis Glomerulonephritis Immunoglobulin A Glomerulus Kidney disease 8 Glomerulus (kidney) lupus erythematosus Systemic lupus erythematosus 9 0 is no t correct . 11 Ofo c hose t his. 10 Granular or linear lgG depositions can be associated with or without C3 (complement) deposition in numerous g lomerulonephritides, but none of · 11 these d iseases is consistent with the clinical history of this patient. Typically lgG complexes are seen in d isease states like poststreptococcal g lomerulonephritis, which this patient does not have due to the relatively acute (days) onset of hematuria, whereas PSGN occu rs about 10-14 • 12 days after infection. • 13 Hematuria Immunoglobulin G Glomerulonephritis Streptococcus Medical history

• 14 E is no t correct . 4 0/o c hose t his. • 15 Deposition of lgM in the mesangial matrix is an incidental nonspecific finding in renal biopsies and is not associated with any particular pathology . Immunoglobulin M Mesangium Biopsy Pathology Kidney • 16 • 17 • 18 Bo tto m Line : • 19 Glomerulonephritis after an upper respiratory tract infection suggests lgA nephropathy, or Berger d isease, which is associated with celiac d isease or liver abnormalities due to defective lgA clearance and resu lting in lgA deposition . • 20 Coeliac disease Upper respiratory tract infection IgA nephropathy Glomerulonephritis Immunoglobulin A Respiratory tract Kidney disease Respiratory tract infection liver Infection • 21 • 6 s 0 lock Suspend End Block Item: 10 of 24 ~ 1 • M k -<:J 1>- Jil ~· !:';-~ QIO: 4746 ..L a r Previous Next Labfli!llues Not es Calculat o r

1 & &

2 FA17p564.1 3 Nephritic syndrome l\ephrltic syndrome= Inflammatory process. When glomeruli are in\"01\'ed, leads to hematuria 4 and RBC casts in urine. Associated with azotemia, oliguria, hypertension (due to salt retention), proteinuria. 5 Acute LM-glomeruli enlarged and h) percellular ~ l os t frequently seen in children. Occurs 6 post streptococca I IF' - ('"starry sky") granular appearance - 2 -4 weeks after group A streptococcal 7 glomerulonephritis ("lumpy-bumpy") g due to lgC, lg\ I, and C3 infection of pharynx or skin. Resolves 8 deposition along GB I and mesangium. spontaneous!). Type Ill h) persensiti' it ~ 9 EM-subepithelial immune complex (IC) reaction. 10 humps. Presents with peripheral and periorbital edema, cola-colored urine, hypertension . • 11 Positi,·e strep titers/serologies, L complement • 12 b ·cls (C3) due to consumption . • 13 Rapidly progressive LM and IF- crescent moon shape Bl. C rescents Poor prognosis. Rapidly deteriorating renal • 14 (crescentic) consist of fibrin and plasma proteins (eg, C3b) function (days to weeks). • 15 glomerulonephritis with glomerular parietal cel ls, monocytes, • 16 macro phages. • 17 Several d isease processes may result in this pallern, in particular: . 18 • Goodpasture syndrome-type II llematuria /hemoptysis. • 19 hypersensiti\'ity reaction; antibodies to Treatment: emergent plasmapheresis. • 20 CB~I and alveolar basement membra ne • 21 -+ linear IF • a s 8 Lock Suspend End Block Item: 10 of 24 ~ 1 • M k -<:J 1>- Jil ~· !:';-~ QIO: 4746 ..L a r Previous Next Labfli!llues Not es Calculat o r

1 & & 2 FA17 p 563.2 3 Glomerular diseases 4 5 Nephritic syndrome-due to GBM disruptiOn Nephrotic syndrome-podocyte disruptiOn 6 Hypertension. i BUN and aeatJ01ne. obguna. -+ charge barner 1mpa1red Massive proteinuna 7 hematuna. RBC casts Ul unne Protem una often (> 3 5g/day) w1th hypoalbum•nem1a. 1n the subnephtotK range(< 3.5 g/day) but an hyperhpldemta. edema May be 1' leg. direct 8 severe cases may be in nephrotic range podocyte damage) orr (podocyte damage rrom systemiC process leg. diabetesJ) 9 • Acute poststreptococcal glomerulonephntiS • Rapadly progress1ve glomerulonephnus • Focal segmental glomerulosclerosiS (1' or 2') 10 • lgA nephropathy (Berger disease) • M1n1mal change d1sease (1' or 2') or 2') • 11 • Alport syndrome • Membranous nephropathy (1' • MembranoproiJrerabve glomerulonephnhs • AmylOidoSIS (2') • 12 • Diabetic glomerulonephropathy 12') • 13 • 14

• 15 Nephritic-nephrotic syndrome-severe nephritic syndrome • 16 with pro round GBM damage that damages the glomerular filtration charge barner -+ nephrot•c·range proteinuria(> 3.5 • 17 g/day) and concomitant features or nephrotic syndrome. Can occur with any room or nephritic syndrome. but IS most • 18 commonty seen with • 19 • D1ffuse prohrerallve gtomerutonephntis • Membranoprohrerahve glomerutonephntiS • 20 • 21 • a s 8 Lock Suspend End Block Item: 11 of 24 ~ 1 • M k -<:J 1>- Jil ~· !:';-~ QIO: 4164 ..L a r Previous Next Labfli!l tues Not es Calculat o r

1 & & A 21-year-old GlPO woman at 30 weeks of pregnancy experiences premature contractions and subsequently delivers a lifeless fetus. The 2 fetus has poorly developed limbs and facial deformities. The volume of amniotic Ruid present in the placenta is <0.5 L. 3 4 What structure did not form correctly in this developing fetus?

5 : 6 A. Gubernaculum

7 B. Median umbilical ligament 8 C. Mesangium 9 D. Paramesonephric duct 10 • 11 E. Ureteric bud

• 12

• 13 • 14

• 15

• 16

• 17

• 18 • 19

• 20

• 21 • a s 8 Lock Suspend End Block Item: llof24 ~ . , . M k <:] t> al ~· ~ QIO: 4164 .l. ar Previous Next lab 'lifllues Notes Calculator

1 The correct a nswer is E. 690/o c hose t his. The permanent metanephric kidneys found in humans orig inate from the ureteric bud . The ureteric bud is an outgrowth of mesodermal cells that 2 arises from the embryon ic mesonephric ducts. Failure of the ureteric bud to form altogether resu lts in Potter's syndrome, or bilateral renal 3 agenesis. This condition is fatal and is characterized by a lack of amniotic fluid because the fetus cannot produce urine. The oligohydramnios that 4 resu lts leads to other developmental anomalies, such as pulmonary hypoplasia, facial deformities, and lim b deform ities. Oligohydramnios Potter sequence Pulmonary hypoplasia Amniotic fluid Renal agenesis Fetus Urine Mesonephric duct Hypoplasia Ureteric bud Kidney Mesoderm 5 A is not correct. 30fo c hose t his. 6 The gubernaculum plays an im portant role in descent of the testes. This fetus has kidneys that failed to develop. 7 Gubernaculum Testicle Fetus Gubernaculum (nematode anatomy)

8 B is not correct. 50fo c hose t his. 9 The median umbilical ligament is a fibrous band that forms when the fetal urachus narrows. Failure of urachal reg ression can lead to a communication between the anterior abdominal wall and bladder. This can present as a d iverticulum, cyst, or sinus. A visible communication 10 between the bladder and the cystic mass is the sign of a patent urachus. 11 Urachus Median umbilical ligament Abdominal wall Urinary bladder Umbilical cord Cyst ligament Diverticulum Fetus

• 12 C is not correct. 60/o c hose t his. • 13 The mesangium describes a set of cells present in the developed g lomerulus. This fetus had a failure of kidney formation itself, which is more accurately described as orig inating from the ureteric bud . • 14 Ureteric bud Mesangium Glomerulus (kidney) Glomerulus Kidney Fetus • 15 0 is not correct. 1 70/o c hose t his . • 16 The paramesonephric duct (mullerian duct) develops into the uterus, fallopian tubes, and part of the vagina in females. In males, anti-mullerian • 17 hormone causes it to degenerate. A failure of these structures to develop would not necessarily cause the oligohydramnios depicted in this vignette . • 18 Oligohydramnios Paramesonephric duct Uterus Vagina Fallopian tube Hormone • 19 • 20 Bottom Line : • 21 "T" I-- · · ·- -a.- .. : _ 1• • . ...1 ; _ a.. l.- • c ... •-- ___ .__ ·-- ·- •- ·-: - 1.: ...1 .••. . • . · · ·•-- ·- ....__ •- · ·-' -' --- ...... -' - · · - • - ·- ·- ·-- ·-- ·-'· · - •: --•- · · -' ·---- ·- : -- - ·--· · -- •----· · -- ....__ c ...... 6 s 0 lock Suspend End Block Item: 11 of 24 ~ 1 • M k -<:J 1>- Jil ~· !:';-~ QIO: 4164 ..L a r Prev ious Next Lab fli!ltues Not es Calculat or .. .. - ...... ' ...... & & 1 Oligohydramnios Potter sequence Pulmonary hypoplasia Amniotic fluid Renal agenesis Fetus Urine Mesonephric duct Hypoplasia ureteric bud Kidney Mesoderm 2 A is n o t corre ct . JO/o ch ose this . 3 The gubernaculum plays an important role in descent of the testes. This fetus has kidneys that failed to develop. 4 Gubernaculum Testicle Fetus Gubernaculum (nematode anatomy) 5 B is n o t correct. soto ch ose t his. band that forms when the fetal urachus narrows. Failure of urachal regression can lead to a 6 The median umbilical ligament is a fibrous communication between the anterior abdominal wall and bladder. This can present as a diverticulum, cyst, or sinus. A visible communication 7 between the bladder and the cystic mass is the sign of a patent urachus. Fetus 8 Urach10s Mec can umbt tea tgament Abdominal wall Urinary bladder Umbi tea co d Cyst Ugament Diverticulum 9 C is not c orre ct. 6 0fo chose t his . set of cells present in the developed glomerulus. This fetus had a failure of kidney formation itself, which is more 10 The mesangium describes a accurately described as originat ing from t he ureteric bud. 11 Ureteric bud Mesangium Glomerulus (kidney) Glomerulus Kidney Fetus

• 12 D is not corre ct. 1 70/o c ho s e t h is. • 13 The paramesonephric duct (mullerian d uct} develops into the uterus, fallopian tubes, and part of the vag ina in females. In males, anti-mullerian e. A failure of these structures to develop would not necessarily cause the o ligohyd ramnios depicted in this • 14 hormone causes it to degenerat vignette . • 15 Oligohydramnios Paramesonephric duct Uterus Vagina Fallopian tube Hormone • 16

• 17 Bottom Line: . 18 The ureteric bud is the origin of the m etanephric kidneys; when the bud does not develop properly, oligohydramnios ensues because the fetus • 19 cannot produce urine . Oligohydramnios Ureteric bud Urine Fetus Kidney • 20 • 21 • a s 8 Lock Suspend End Block Item: 11 of 24 ~ 1 • M k -<:J 1>- Jil ~· !:';-~ QIO: 4164 ..L a r Previous Next Labfli!l tues Not es Calculat o r

1 & & FA17 p 548.2 2 Potter sequence Oligohydramnios ..... compression of Babies who can't "Pee" in utero de,·clop Potter 3 (syndrome) developing fetus ..... limb deformities, sequence. 4 facial anomalies {eg, lo"-set ears and POTTER sequence associated with: 5 retrognathia fJ, Aattened nose), compression Pulmonary h~ ·poplasia 6 of chest and lack of amniotic Auid aspiration Oligohrdramnios {trigger) 7 into fetal lungs ..... pulmonary hypoplasia 'Jwisted face {cause of death). 'h' isted skin 8 Causes include ARPKD, obstructi\'e uropathy I<.xlremity defects 9 {eg, posterior urethral val\'es), bilateral renal Renal failure {in utero) 10 agenesis, chronic placental insufficiency. 11 • 12 FA17 p 548.1 • 13 Kidney embryology Pronephros-week 4; then degenerates. • 14 Mesonephros-functions as interim kidney for } Degenerated lst trimester; later contributes to male genital • 15 pronephros system . • 16 Metanephros-permanent; first cl h\' 10th \\I'Pk a s 8 Lock Suspend End Block Item: 12 of 24 ~ 1 • M k -<:J 1>- Jil ~· !:';-~ QIO: 5158 ..L a r Previous Next Labfli!l tues Not es Calculat o r

1 & & A 67-year-old obese man complains of swelling in his hands, feet, and face. Physical examination is significant for 3+ edema in his lower 2 extremities and 1 + edema in his hands and around his eyes. Urine dipstick reveals 3 + protein and no blood. Renal biopsy results are shown in the image. 3 4 5 6 7 8 9 10

11 • 12 • 13 • 14 • 15 • 16 • 17 In addition to the underlying cause of his renal disease, which of the following comorbidities is most likely present in this patient? . 18 • 19 : A. Arthritis • 20 • 21 B. Bone pain • a s 8 Lock Suspend End Block 2 3 4 5 6 7 8 9

10

11 • 12 In addition to the underlying cause of his renal d isease, which of the following comorbidities is most likely present in this patient? • 13 : A . Arthritis • 14 • 15 B . Bone pain

• 16 c . Coronary artery disease • 17 D. Hearing loss . 18 • 19 E . Hemoptysis

• 20 F . Pharyngitis • 21 • a s 8 Lock Suspend End Block Item: 12 of 24 ~ 1 • M k -<:J 1>- Jil ~· !:';-~ QIO: 5158 ..L a r Prev ious Next Lab fli!ltues Not es Calculat or

1 & & Th e c o rrect a n sw er is C. 61 Ofo chose this. 2 This patient most likely has diabetic nephropathy caused by long-standing and often poorly managed 3 diabetes mellitus. The patient's presentation is typical for nephrotic syndrome, with massive proteinuria and peripheral and periorbital edema. Blood tests would likely have shown 4 hypoalbuminemia and hyperlipidemia, which are also associated with nephrotic syndrome. The 5 image shows changes typically associated with diabetic nephropathy, including basement membrane thickening and presence of hyaline deposits in the periphery of the glomerulus (known as 6 Kimmelstiei-Wilson nodular lesions, shown in blue circles in this image). The increased glucose levels 7 in diabetes can lead to vascular damage, and diabetes is strongly associated with coronary artery 8 disease. In addition to management of lipids and blood glucose, angiotensin-converting enzyme inhibitors and/ or angiotensin II receptor blockers are beneficial in the treatment of diabetic 9 nephropathy. 10 Neph otic synd orne Hypoalbuminemia Diabetic nephropathy Proteinuria Coronary artery disease Hyperlipidemia Image courtesy of CDC/ Dr. Edwin P. Basement membrane Angiotensin II Angiotensin-converting enzyme Angiotensin ACE inhibitor Kidney disease Ewing, Jr. 11 Enzyme Glomerulus Edema Diabetes mellitus Blood sugar Angiotensin II receptor antagonist 12 Glomerulus (kidney) Glucose Lipid Hyaline Blood vessel • 13 Enzyme inhibitor Receptor (biochemistry) • 14 A i s not co rrec t . 100/o chose this • presenting symptoms (proteinuria, peripheral and • 15 Although renal disease associated with systemic lupus erythematosus (SLE) may have similar periorbital edema, and hypoalbuminemia), the image shows Kimmelstiei-Wilson nodular lesions, w hich a re characterist ic of d iabetic nephropathy . • 16 With SLE, there are five different patterns of renal involvement. In the membranous glomerulonephritis pattern, biopsy reveals "spike and dome" • 17 lesions with subepithelial deposits. In addition to arthritis, symptoms of SLE include fatigue, malar rash, photosensitivity, pleuritis, pericarditis, and many more . • 18 Systemic lupus erythematosus Hypoalbuminemia Pleurisy Diabetic nephropathy Membranous glomerulonephritis Proteinuria Pericarditis Glomerulonephritis • 19 Photosensitivity Kidney disease Rash Lupus erythematosus Malar rash Biopsy Edema Fatigue (medical) Arthritis Diabetes mellitus Kidney

• 2 0 B i s n o t correct . 90/o ch ose this. • 21 Although renal disease associated with amyloidosis has similar presenting symptoms (proteinuria, peripheral and periorbital edema, and • " . " . ' f • • I '• ••t ' ' ' . ' r •· t , • . ' ' ' . a s 8 Lock Suspend End Block Item: 12 of 24 ~ . I • M k <:] t> al ~· ~ QIO: 5158 .l. ar Previous Next lab 'lifllues Notes Calculator

1 B is no t correct . 90fo c hose t his. 2 Although renal d isease associated with amyloidosis has similar presenting symptoms (proteinuria, peripheral and periorbital edema, and 3 hypoalbuminemia), the image shows Kimmelstiei-Wilson nodular lesions, which are characteristic of d iabetic nephropathy. Renal biopsy viewed under immunofluorescence with Congo red stain reveals apple-green birefringence in patients with amyloidosis. One excellent example of 4 amyloidosis is amyloid from immunog lobulin light chains, which is produced by cancerous plasma cells in multiple myeloma. In addition to bone 5 pain, signs and symptoms of multiple myeloma include renal failure, elevated calcium, anemia, and increased vulnerability to infection. Multiple myeloma Congo red Hypoalbuminemia Diabetic nephropathy Proteinuria Immunofluorescence Amyloidosis Amyloid Anemia Renal biopsy Biopsy Antibody 6 Immunoglobulin light chain Edema Blood plasma Diabetes mellitus Plasma cell Kidney disease Calcium Kidney Bone pain Infection 7 0 is no t correct . ]Ofo c hose t his. 8 Alport syndrome is an inherited g lomerular d isease caused by a mutation in type IV collagen. It can be inherited in an X-linked or an autosomal 9 recessive manner. It typically presents with recu rrent episodes of g ross hematuria during childhood. It is associated with sensorineural hearing I 10 loss and ocu lar d isorders. Renal biopsy would show a split basement membrane, not the Kimmelstiei-Wilson nodular lesions shown in the image. Alport syndrome Hematuria Basement membrane Collagen Type IV collagen Sensorineural hearing loss Autosomal recessive Dominance (genetics) Biopsy 11 Glomerulus Mutation Hearing loss Autosome Sex linkage Glomerulus (kidney) Renal biopsy 12 E is no t correct . SO/o c hose t his . • 13 Goodpasture d isease resu lts in a rapid ly progressive g lomerulonephritis, with proteinuria and hematuria, and alveolar hemorrhage causing • 14 shortness of breath and hemoptysis. It is caused by antibodies d irected against the g lomerular basement membrane. Renal biopsy with immunofluorescence would show linear deposition of lgG along the g lomerular membrane. Kimmelstiei-Wilson nodular lesions, as seen in the • 15 image, would not be observed . • 16 Hemoptysis Hematuria Rapidly progressive glomerulonephritis Proteinuria Glomerular basement membrane Immunofluorescence Basement membrane • 17 Renal biopsy Glomerulonephritis Immunoglobulin G Biopsy Antibody Glomerulus Pulmonary alveolus Dyspnea Glomerulus (kidney) Bleeding

• 18 F is no t correct . 50fo c hose t his . • 19 Acute poststreptococcal g lomerulonephritis {APSGN} is associated with recent Group A streptococcal infection, which would present as a history of skin (impetigo) or throat (pharyngitis) infection. It is seen most often in children, and can also present with peripheral and periorbital edema • 20 and proteinuria as seen in this patient. However, APSGN also usually presents with either g ross or microscopic hematuria, which is not found in • 21 this patient. Additionally, renal biopsy of a patient with APSGN would typically show d iffuse hypercellularity with a neutrophilic infiltrate on light ...... :.-.-...... :.1...... 1: .-. 1 ...... -._...... :.-.-...... 6 s 0 lock Suspend End Block Item: 12 of 24 ~ . I • M k <:] t> al ~· ~ QIO: 5158 .l. ar Previous Next lab 'lifllues Notes Calculator

• p y p ~ ll g 1 Alport syndrome Hematuria Basement membrane Collagen Type IV collagen Sensorineural hearing loss Autosomal recessive Dominance (genetics) Biopsy 2 Glomerulus Mutation Hearing loss Autosome Sex linkage Glomerulus (kidney) Renal biopsy 3 E is no t correct. S O/o c hos e this. 4 Goodpasture d isease resu lts in a rapid ly prog ressive g lomerulonephritis, with proteinuria and hematuria, and alveolar hemorrhage causing 5 shortness of breath and hemoptysis. It is caused by antibodies d irected against the g lomerular basement membrane. Renal biopsy with immunofluorescence would show linear deposition of lgG along the g lomerular membrane. Kimmelstiei-Wilson nodular lesions, as seen in the 6 image, would not be observed. 7 Hemoptysis Hematuria Rapidly progressive glomerulonephritis Proteinuria Glomerular basement membrane Immunofluorescence Basement membrane Renal biopsy Glomerulonephritis Immunoglobulin G Biopsy Antibody Glomerulus Pulmonary alveolus Dyspnea Glomerulus (kidney) Bleeding 8 9 F is no t correct. 50/o c hose this. Acute poststreptococcal g lomerulonephritis (APSGN} is associated with recent Group A streptococcal infection, which would present as a history 10 of skin (impetigo) or throat (pharyngitis) infection . It is seen most often in children, and can also present with peripheral and periorbital edema 11 and proteinuria as seen in this patient. However, APSGN also usually presents with either g ross or microscopic hematuria, which is not found in this patient. Additionally, renal biopsy of a patient with APSGN would typically show d iffuse hypercellularity with a neutrophilic infiltrate on light 12 microscopy, and subepithelial humps on electron microscopy . • 13 Hematuria Impetigo Proteinuria Pharyngitis Glomerulonephritis Renal biopsy Group A streptococcal infection Biopsy Streptococcus Edema light microscopy • 14 Electron microscope Optical microscope Kidney Infection Microscopy • 15 • 16 Bo tto m Line : • 17 Diabetic nephropathy typically presents with massive proteinuria, peripheral and periorbital edema, hypoalbuminemia, and hyperlipidemia, in a • 18 patient with long-standing poorly controlled d iabetes mellitus. Renal biopsy shows hyaline deposits in the periphery of the g lomerulus (Kimmelstiei-Wilson nodular lesions) and basement membrane thickening. It is very important to consider the increased risk of coronary artery • 19 d isease in managing patients with long-standing, poorly controlled d iabetes. • 20 Hypoalbuminemia Diabetic nephropathy Proteinuria Basement membrane Coronary artery disease Hyperlipidemia Diabetes mellitus Glomerulus (kidney) Glomerulus Kidney disease Edema Biopsy Renal biopsy Hyaline Kidney • 21 • 6 s 0 lock Suspend End Block Item: 12 of 24 ~ 1 • M k -<:J 1>- Jil ~· !:';-~ QIO: 5158 ..L a r Previous Next Labfli!l tues Not es Calculat o r

& & 1 FA17 p 566.1 2 Nephrotic syndrome 1'\ephrO tic syndrome-massi' e prO teinuria (> 3.5 g/day) with hypoalbuminemia, resulting 3 edema, hyperlipidemia. Frothy urine" ith fatty casts. Due to podocyte damage disrupting 4 glomerular filtration charge barrier. \lla)' be 1° (eg, direct sclerosis of podocytes) or 2° (systemic 5 process [eg, diabetes] secondarily damages podocytes). Associated with hypercoagulable stale (cg, thromboembolism) due to antithrombin ( T ) Ill loss in urine and f risk of infection (due to loss of 6 immunoglobulins in urine and soft tissue compromise by edema). 7 Se,·ere nephritic syndrome may present with nephrotic S}lldrome features (nephritic-nephrotic 8 syndrome) if damage to C Bt\ 1 is se,·cre enough to damage charge barrier. 9 Minimal change LVI-normal glomeruli (lipid may be seen in \!l ost common cause of nephrotic sp1drome 10 disease (lipoid PCT cells). in children. Often 1° (idiopathic) and ma) be 11 nephrosis) we. triggered by recent infection, immunization, 12 EM-effacement of fo ot processes Fl. immune stimulus. Rarely, may be zoto lymphoma (eg, cytokine-mediated damage). I0 • 13 disease has excellent response to corticosteroids . • 14 Focal segmental LM-segmental sclerosis and hyalinosis : . Most common cause of nephrotic syndrome in • 15 glomerulosclerosis IF -often 8, but may be (f) for nonspeci fie focal African Americans and Hispanics. Can be ]0 • 16 deposits of Ig r, C3, C l. (idiopathic) or zo to other conditions (eg, I-I IV • 17 EM -effacement of foot process similar to infection, sickle cell disease, heroin abuse, . 18 minimal change disease. massive obesity, interferon treatment, chronic • 19 kidney d isease due to congenital malfom1ations) . 1° disease has inconsistent response to steroids . • 20 ~ lay progress to chronic renal disease. • 21 • M<>m hr:onnooc a s 8 Lock Suspend End Block Item: 12 of 24 ~ 1 • M k -<:J 1>- Jil ~· !:';-~ QIO: 5158 ..L a r Previous Next Labfli!l tues Not es Calculat o r

1 & &

2 FA17 p336.1 Diabetes mellitus 3 ACUTE MANIFESTATIONS coma (type 2). 4 Polydipsia, polyuria, polyphagia, \\eight loss, DKA (type I), h) perosmolar Rarel y, can be caused by unopposed secretion of C ll and epinephrine. Also seen in patients on 5 glucocorticoid therapy (steroid diabetes). 6 CHRONIC COMPLICATIONS Nonenzymatic glycation: 7 mall \·esse! disease (diffuse thickening of basement membrane) - retinopathr (hemorrhage, 8 exudates, microaneurysms, \'esscl prol ifemt ion), glaucoma, neuropath\·, nephropathy (nodular 9 glomerulosclerosis, aka Kimmclst iel-Wilson nodules - progressi\·e proteinuria [initiallr 10 ; t\CE inhibitors are renoprotective) and arteriolosclerosis - hypertension; both lead to chronic renal failmc). 11 Large \'esse! atherosclerosis, CAD, periphera l \'a)cular occlusive disease, gangrene - limb loss, 12 cerebrovascular disease. \tl I most common cause of death. • 13 Osmotic damage (sorbitol accumulation in organs with aldose reductase and l or absent sorbitol • 14 dehyd rogenase): • 15 • europathy (motor, sensory !glove and stocking distribution], and autonomic degeneration) • Cataracts • 16 DIAGNOSIS IESI DIAGNOSTIC CUTOFF NOlES • 17 HbA1c ~ 6.5% Reflects average blood gl ucose . 18 O\·er prior 3 months I • 19 Fasting plasma glucose ~ 126 mg/d L Fasting for > 8 hours • 20 Z-hour oral glucose tolerance test ~ 200 mg/dL Z hours after consumption of 75 g • 21 of glucose in water • a s 8 Lock Suspend End Block Item: 13 of 24 ~ 1 • M k -<:J 1>- Jil ~· !:';-~ QIO: 4170 ..L a r Previous Next Labfli!llues Not es Calculat o r

1 & & A 68-year-old man with long-standing hypertension and d iabetes with secondary kidney disease presents to the clinic with gastrointestinal 2 symptoms and wate•-y diarrhea for 24 hours. His vital signs are consistent with his baseline documented in the m edical record. His serum pH is 19 mEq/L. His serum potassium is also elevated at 6. 7 mEq/L, and all other electrolytes are normal. 3 is 7.36 and bicarbonate 4 Which of the following therapies is most likely to help correct the hyperkalemia? 5

6 A. Administer eplerenone 7 8 B. Administer furosemide with normal saline

9 C. Administer hydrochloric acid 10 D. Administer lisinopril wit h normal saline 11 E. Large-volume fluid resuscitation with normal saline 12 • 13 • 14

• 15

• 16

• 17

• 18 • 19

• 20

• 21 • a s 8 Lock Suspend End Block Item: 13 of 24 ~ 1 • M k -<:J 1>- Jil ~· !:';-~ QIO: 4170 ..L a r Prev ious Next Labfli!llues Not es Calculat or

1 & & Th e correct an swer is B. 660/o chose this. 2 This patient is losing bicarbonate secondary to his diarrhea, causing a non-anion gap metabolic acidosis. Increased serum concentration of H+ in 3 acidosis leads to an exchange with cells via the H+-K+ exchanger, meaning that extracellular K+ increases to temporize the increase in H+. This patient has a relatively mild hyperkalemia. Treatment for hyperkalemia is step-wise: (1) give calcium gluconate to stabilize the cardiac cell 4 membranes and prevent arrhythmias; (2) determine the cause of the hyperkalemia; (3) utilize short-term pharmacotherapy (insulin + glucose, 5 ~-agonists, sodium bicarbonate) to shift K+ back into the intracellular environment; (4) increase potassium excretion by giving normal saline with a loop diuretic, via a cation-exchange resin like sodium-polystyrene or by hemodialysis. Normal saline + furosemide works by increasing the 6 excretion of sodium and potassium in the loop of Henle. 7 Loop d ehc Furosemide Hyperkalemia calcium gluconate Sodium bicarbonate Metabolic ac•dosis Diuretic Hemodialysis Diarrhea Loop of Henle Bicarbonate 8 Potassium Acidosis Cardiac arrhythmia Sodium Insulin Saline (medicine) Glucose calcium Serum (blood) Intracellular Cell membrane Metabolism Blood plasma Cardiac muscle ce · Pharmacotherapy 9 this. 10 A is not c orrect. g o;o chose Eplerenone is an aldosterone antagonist that acts as a diuretic: it is similar to spironolactone, but has a much higher affinity for the 11 mineralocorticoid receptor and thus has no effect on testosterone or estrogen levels. However, this diuretic would not be appropriate because it 12 does not enhance excretion of potassium. In fact, individuals who are prescribed eplerenone must be monitored for hyperkalemia. Antimineralocorticoid Hyperkalemia Eplerenone Mineralocorticoid receptor Spironolactone Aldosterone Diuretic Estrogen Testosterone Mineralocorticoid 13 Receptor antagonist Potassium Receptor (biochemistry) • 14 C is not correct. JO/o c hose this • • 15 Hydrochloric acid administration would be harmful to this patient, given his low pH. This would likely worsen the acidosis and might exacerbate • 16 the hyperkalemia through H+-K+ exchange across cell membranes. • 17 Hyperkalemia Hydrochloric acid Acidosis Cell membrane . 18 D is not correct. solo chose this • hyperkalemia, therefore it would • 19 Lisinopril is an ACE Inhibitor used to in hypertension, heart failure, and diabetes. Lisinopril can actually cause not be appropriate for acute use in this case . • 20 Hyperkalemia ACE nhibitor Usinopril Diabetes mellitus Hypertension Heart fa1lure Enzyme Inhibitor • 21 • E is n o t correct . 140/o ch ose this• a s 8 Lock Suspend End Block Item: 13 of 24 ~ . I • M k <:] t> al ~· ~ QIO: 4170 .l. ar Previous Next lab 'lifllues Notes Calculator . . . 1 Potassium Acidosis Cardiac arrhythmia Sodium Insulin Saline (medicine) Glucose Calcium Serum (blood) Intracellular Cell membrane Metabolism Blood plasma 2 Cardiac muscle cell Pharmacotherapy

3 A is not correct. 90fo c hose t his. 4 Eplerenone is an aldosterone antagonist that acts as a d iuretic: it is similar to spironolactone, but has a much higher affinity for the 5 mineralocorticoid receptor and thus has no effect on testosterone or estrogen levels. However, this d iuretic would not be appropriate because it does not enhance excretion of potassium. In fact, ind ividuals who are prescribed eplerenone must be monitored for hyperkalemia. 6 Antimineralocorticoid Hyperkalemia Eplerenone Mineralocorticoid receptor Spironolactone Aldosterone Diuretic Estrogen Testosterone Mineralocorticoid 7 Receptor antagonist Potassium Receptor (biochemistry)

8 C is not correct. 30/o c hose t his. 9 Hydroch loric acid administration would be harmful to this patient, g iven his low pH. This would likely worsen the acidosis and might exacerbate the hyperkalemia through H+-K+ exchange across cell membranes. 10 Hyperkalemia Hydrochloric acid Acidosis Cell membrane 11 0 is not correct. SO/o c hose t his. 12 Lisinopril is an ACE Inhibitor used to in hypertension, heart failure, and d iabetes. Lisinopril can actually cause hyperkalemia, therefore it would 13 not be appropriate for acute use in this case. Hyperkalemia ACE inhibitor lisinopril Diabetes mellitus Hypertension Heart failure Enzyme inhibitor • 14 E is not correct. 140/o c hose t his . • 15 The patient's vital signs are normal, therefore he does not require a fluid bolus. Maintenance fluids would be appropriate . • 16 Bolus (medicine) Bolus {digestion) Vital signs • 17

• 18 Bottom Line : • 19 Hyperkalemia of metabolic acidosis may be treated with a loop d iuretic such as furosemide with normal saline . • 20 loop diuretic Furosemide Hyperkalemia Metabolic acidosis Diuretic Saline (medicine) Acidosis Metabolism • 21 6 s 0 lock Suspend End Block Item: 13 of 24 ~ 1 • M k -<:J 1>- Jil ~· !:';-~ QIO: 4170 ..L a r Previous Next Labfli!llues Not es Calculat o r

1 & &

2 FA17 p 575.3 3 loop diuretics 4 Furosemide, bumetanide, torsemide 5 MECHANISM Sulfonamide loop diuretics. Inhibit cotransport 6 system (Na+fK•'fZCJ-) of thid. ascending limb of loop of llcnlc. bolish hypertonicit) of 7 medulla, preventing concentration of urine. 8 Stimulate PCE release (\'asodilatory effect 9 on afferent arteriole); inhibited b) NSA IDs. 10 f Ca2+ excretion. Loops Lose Ca1•. 11 CliNICAl USE Edematous states (HF, cirrhosis, nephrotic 12 syndrome, pulmonar} edema), h) pertension, ia. 13 hypercalcem • 14 ADVERSE EFFECTS Ototoxicity, llypokalemia, llypomagnesemia, Ollll D.\ANC! Dehydration, Allergy (sulfa), metabolic • 15 Alkalosis, Nephritis (intersti tia l), Gout . • 16 Ethacrynic acid • 17 MECHANISM 1'\onsulfonamide inhibitor of cotransport system . 18 ( a•fK+fZCJ -) of thick ascending limb of loop • 19 of l lenle . • 20 CliNICAl USE Diuresis in patients allergic to sulfa dmgs. • 21 • Simibr to fnrn~PmirlP hnt mnrP n tn ln' if' I .non P:>rri n a~ hnrt vnnr P:> r~ a s 8 Lock Suspend End Block 13 24 • M k ~ £!1}>' !!":-~ Item: of ~ 1 a r -<:J I> • QIO: 4170 ..L Previous Next Lab lues Not es Calculat o r

1 & & FA17 p 561 .2 2 Acidosis and alkalosis 3 Check arterial pH 4 5 pH< 7.35 pH> 745 6 Ac.idemia Alkalemia 7 8 9 RespiratOf}' Respiratory 10 Metabo~c acidos1s Metabolic alkalOSIS acidosis alkalosis 11 12 gap Hypowntilation Check anion Hyperventilation H• lossiHC01- excess 13 =Na ' - (CI- +HCO ; l Airway obstruction Hysteria loop diuretics • 14 Acute lung disease Hypoxemia (eg, high altitude) Vomiting Chronic lung disease Sallcylates (early) Antacid use • 15 Opioids, sedatives Tumor Hyperaldosteronism Weakening of respiratory Pulmonary embolism • 16 muscles • 17 . 18 > 12 mEq/L 8-12 mEq/L Pco . • • 19 I 45 40mmHg • 20 40 Resporatory 1 Anion gap Nonnal anion gap :::1 ac•dosls • 21 35 Mixed • MUDPILES: HARDASS: l alkalosrs Oft ----- a s 8 Lock Suspend End Block • M k ~ £!1}>' !!":-~ Item: 13 of 24 ~ 1 a r -<:J I> • QIO: 4170 ..L Previous Next Lab lues Not es Calculat o r

1 & &

2 FA17 p 559.2 3 Hormones acting on kidney 4 ~ 1\:l c •c "'e" 1e 5 Secreted in response toi atrial pressure Causes i GFR and i Na· filtration with no compensatory Na· reabsorpbon 6 in distal nephron Net effect Na· loss and volll!le toss Distal 7 convoluted Glomerulus tubule cr· 8 0 Alferent ""' 9 10 Efferent • \..__ K 11 H Secreted 1n response to 12 Angiotensin II .!. blood volume (vJa AT II) and Synthesized in response to J. BP Causes efferent arteriole i plasma (K"I. causes iNa 13 constnction-+ i GFR and i FF but with compensatory Na· reabsorption. iK• secretion. reabsotpbon in proximal and distal nephron. Net effect: • 14 Cortex i H• secretion. preservation of renal function li FFl in low· volume state - - • 15 with simultaneous Na' reabsorption {both proximal Medulla AOH (v sopr s n) and distal) to maintain circulating volume. Na' K Secreted in response to • 16 20 i plasma osmolarity and • 17 Parathyrotd hormone J. blood volume Binds to Secreted •n response to Ascending timb, loop of Henle receptors on principal cells. . 18 .!. plasma [u2'1. i plasma IP04 >-), {por,.ablt to ..11 11 causing i number of or J. plasma 1.25-(0H)z 03. • 19 aquaporins and i H~ Causes i (u2'1reabsorption IOCn. reabsorption • 20 .!. IP04' I reabsorpbon (PCTJ. and i 1.25-{0Hb 03 productiOn Collecting duct • 21 (i u·· and P04 absorption from gut • a s 8 Lock Suspend End Block Item: 14 of 24 ~ 1 • M k -<:J 1>- Jil ~· !:';-~ QIO: 4161 ..L a r Previous Next Labfli!l tues Not es Calculat o r

1 & & The kidneys of a 65-year-old patient with long-standing diabetes mellitus are examined at autopsy. One kidney is shown in the image. In the 2 later stage of his disease, this patient frequently required a Foley catheter to adequately drain his bladder. 3 4 5 6 7 8 9 10

11 12 13 • 14 • 15 • 16 • 17 What condition most likely caused the anomalous appearance of his kidneys? . 18 : • 19 A. Acute pyelonephritis

• 20 B. Chronic reflux-associated pyelonephritis • 21 • ,... t"\ .... _ : ... .J •• ~-.J = -~---~:~=-· ...... &...-:~: ..... a s 8 Lock Suspend End Block Item: 14 of 24 ~ 1 • M k -<:J 1>- Jil ~· !:';-~ QIO: 4161 ..L a r Previous Next Labfli!l tues Not es Calculat o r ...... ~ 1

2 3 4 5 6 7 8 9 10

11 12 13 • 14 • 15 What condition most likely caused the anomalous appearance of his kidneys? • 16 • 17 : A . Acute pyelonephritis . 18 • 19 B. Chronic reflux-associated pyelonephritis

• 20 C. Drug-induced interstitial nephritis • 21 • n H\tnortonc:inn a s 8 Lock Suspend End Block 12 13 What condition most likely caused the anomalous appearance of his kidneys?

• 14 : • 15 A. Acute pyelonephritis

• 16 B. Chronic reflux-associated pyelonephritis • 1 7 C. Drug-induced interstitial nephritis . 18 D. Hypertension • 19 • 20 E. Minimal change disease • 21 • a s 8 Lock Suspend End Block Item: 14 of 24 ~ 1 • M k -<:J 1>- Jil ~· !:';-~ QIO: 4161 ..L a r Prev ious Next Lab fli!ltues Not es Calculat or

1 & & The c o rrect a ns w e r i s B. 650/o chose this . 2 Long-time diabetes in this patient most likely caused a neurogenic bladder. Overfilling of the bladder 3 could have caused vesicoureteral reflux to the kidneys. Renal damage results when a bacterial urinary tract infection is superimposed on the reflux. Chronic pyelonephritis results in a kidney that 4 grossly shows blunting and thickened dilation of the calyces and uneven scarring. The condition may 5 happen unilaterally if due to some congenital anatomic abnormality. The oval in the image here shows a stag horn calculus filling calyces of kidney with chronic pyelonephritis; the cortex and 6 medulla is atrophic and the calyces are dilated. Staghorn calculi (struvite stones) can be seen in 7 association with chronic infection and are classically associated with with urease producing organisms such as proteus species. 8 Urina y t infection Pyelonephritis Vesicoureteral reflux Urease Neurogenic bladder dysfunction Struvite 9 Diabetes mellitus Urinary system Urinary bladder Renal calyx Kidney Nervous system Atrophy 10 Congenital disorder Infection 11 A is not correct. 13 0/o c hose this . 12 Although diabetic patients are susceptible to acute pyelonephritis from increased instrumentation and neurogenic bladder, the kidney shown in 13 the image is not characteristic of acute pyelonephritis. In the acute condition, the kidney surface is freq uently studded with microabscesses, 14 indicating a recent infection. Pyelonephritis Neurogenic bladder dysfunction Kidney Urinary bladder Diabetes mellitus Nervous system Infection • 15 • 16 C is not co rrect. s olo c hose this . There is nothing in this patient's history to indicate acute tubulointerstitial nephritis. Acute tubulointerstitial nephritis is often caused by • 17 hypersensitivity to a drug (eg, methicillin, rifampin, thiazide diuretics, and nonsteroidal anti-inflammatory drugs). It is characterized by fever, • 18 rash, eosinophilia, and renal anomalies (hematuria, increased serum creatinine, and oliguria) . Oliguria Hematuria Interstitial nephritis Rifampicin Creatinine Eosinophilia Thiazide Diuretic Nonsteroidal anti-inflammatory drug Nephritis Serum creatinine • 19 Hypersensitivity Blood plasma Anti-inflammatory Rash Meticillin Serum (blood) Fever • 2 0 D is no t correct . 1 2 0/o chose this . • 21 • Althn11nh hvnPrtPnc;;inn m::t~v hP ::t~ c:inn nf thP rhrnnir rPn::tl f::t~il11rP hPinn r;u I<:;Prl hv rhrnnir nvPinnPnhritic::; hvnPrtPnc:inn ::t~lnnP wn1 drl nnt r;u IC:P thP a s 8 Lock Suspend End Block Item: 14 of 24 ~ . I • M k <:] t> al ~· ~ QIO: 4161 .l. ar Previous Next lab 'lifllues Notes Calculator

1 , , ( r I I 1 A is not correct. 130/o c hose t his. 2 Although d iabetic patients are susceptible to acute pyelonephritis from increased instrumentation and neurogenic bladder, the kidney shown in 3 the image is not characteristic of acute pyelonephritis. In the acute condition, the kidney surface is frequently studded with microabscesses, 4 ind icating a recent infection . Pyelonephritis Neurogenic bladder dysfunction Kidney Urinary bladder Diabetes mellitus Nervous system Infection 5 C is not correct. SO/o c hose t his. 6 There is nothing in this patient's history to ind icate acute tubulointerstitial nephritis. Acute tubulointerstitial nephritis is often caused by 7 hypersensitivity to a d rug (eg, methicillin, rifampin, thiazide d iuretics, and nonsteroidal anti-inflammatory d rugs). It is characterized by fever, 8 rash, eosinophilia, and renal anomalies (hematuria, increased serum creatinine, and oliguria). Oliguria Hematuria Interstitial nephritis Rifampicin Creatinine Eosinophilia Thiazide Diuretic Nonsteroidal anti-inflammatory drug Nephritis Serum creatinine 9 Hypersensitivity Blood plasma Anti-inflammatory Rash Meticillin Serum (blood) Fever 10 0 is not correct. 1 20/o c hose t his. 11 Although hypertension may be a sign of the chronic renal failure being caused by chronic pyelonephritis, hypertension alone would not cause the 12 pathology shown in the image. Pyelonephritis Chronic kidney disease Hypertension Pathology Kidney 13 14 E is not correct. 20/o c hose t his. Minimal change d isease is a g lomerular d isease most frequently seen in pediatric patients. It is a cause of the nephrotic syndrome in a healthy • 15 patient. Most patients respond to steroid therapy; a small fraction are at risk for chronic renal failure years later. • 16 Nephrotic syndrome Minimal change disease Chronic kidney disease Steroid Glomerulus Glomerulus (kidney) Kidney • 17

• 18 Bottom Line : • 19 Ch ron ic d iabetes is associated with neurogenic bladder requiring catheterization, which can lead to chronic pyelonephritis, with kidneys that • 20 exhibit uneven scarring along with blunting and d ilation of the calyces . Pyelonephritis Neurogenic bladder dysfunction Diabetes mellitus Renal calyx Urinary catheterization Urinary bladder Kidney Catheter Nervous system Vasodilation • 21 6 s 0 lock Suspend End Block Item: 14 of 24 ~ 1 • M k -<:J 1>- Jil ~· !:';-~ QIO: 4161 ..L a r Previous Next Labfli!l tues Not es Calculat o r

1 & & FA17 p 570.2 2 Pyelonephritis 3 Acute pyelonephritis infiltrate renal interstitium · . Affect~ cortex with relati,·e sparing of glomeruli/vessels. 4 Presents with fe,·ers, Aank pain (costovertebral angle tenderness), nausea/vomiting, chills. 5 Causes include ascending UTI (E coli is most common), hematogenous spread to kidney. Presents enhancement 6 with vVBCs in urine+/- \VBC casts. CT \\Ould sho" striated parenchymal Ri sk factors include indwelling urinar> catheter, urinary tract obstruction, ves icoureteral reAu;\, 7 diabetes mellitus, pregnancy. 8 Complications include chronic pyelonephritis, renal papillar)' necrosis, perinephric abscess, 9 u roseps1s. 10 Treatment: antibiotics. 11 Chronic The result of recurrent episodes of acute pyelonephritis. Typically requires predisposition to 12 pyelonephritis infection such as vesicourctcral rcAux or chronically obstructing kidney stones. Coarse, asymmetric corticomedullary scarring, blunted calyx. Tubules can contain eosinophilic 13 casts resembling thyroid ti ssue (thyroidiza ti on of kidn ey). 14 Xanthogranulomatous pyelonephritis- rare; gross!)' orange nodules that can mimic tumor • 15 nodules; characterized by widespread kidn ey damage due to granulomatous ti ssue conta ining • 16 foamy macrophages . • 17 ... . 18 • 19 • 20 • 21 • a s 8 Lock Suspend End Block Item: 15 of 24 ~ 1 • M k -<:J 1>- Jil ~· !:';-~ QIO: 5157 ..L a r Previous Next Labfli!llues Not es Calculat o r

1 & & A 3-year-old boy comes to the pediatrician after his mother noticed swelling in his face and legs. His physical examination is significant for 2 periorbital edema and 1 + edema in his lower extremities. Urine dipstick reveals 3 + protein and no blood. 3 4 Examination of tissue from a kidney biopsy under light microscopy would most likely reveal which of the following?

5 : 6 A. Circumferential subendothelial deposits

7 B. Diffuse capillary and basement membrane thickening 8 C. Double-contour basement membrane appearance 9 D. Glomerular crescent format ion 10 11 E. Normal glomeruli

12 F. Pink hyaline nodules 13 14

0 15

0 16

0 17

0 18 • 19

0 2 0 • 21 • a s 8 Lock Suspend End Block Item: 15 of 24 ~ 1 • M k -<:J 1>- Jil ~· !:';-~ QIO: 5157 ..L a r Prev ious Next Labfli!llues Not es Calculat or

& & 1 The correct a ns wer is E. 690/o chose this. 2 This patient most likely has minimal change disease, the most common cause of childhood nephrotic symptoms include peripheral and periorbital edema, massive 3 syndrome. Typical signs and proteinuria, hypoalbuminemia, and hyperlipidemia. Kidney biopsy from a patient with minimal 4 change disease appears normal on light microscopic examination. However, electron microscopy 5 reveals effacement of the foot processes of the podocytes, as evidenced by the red arrows in the I image. Minimal change disease is typically diagnosed by urinary protein excretion >50 mg/kg/d and 6 a serum albumin level <3 g/dl. This disease is easily treated in children (it may be refractory in 7 adults) and responds well to steroids. Neph otic synd orne Hypoalbuminemia Minimal change disease Proteonuria Hyperlipodemia Albumin 8 Serum albumin Protein Human serum albumin Edema Fine-needle aspiration Blood plasma Electron microscope 9 Biopsy Steroid Kidney Podocyte Renal biopsy Microscopy Serum (blood) 10

11

12 A is not correct. 8 0fo chose this. immune complex deposits giving a "wire-loop" appearance are seen in d iffuse lupus nephritis in patients with 13 Circumferential subendothelial systemic lupus erythematosus. Lupus nephritis has presenting signs and symptoms similar to those of minimal change d isease; however, it is not 14 commonly found in children. 15 Lupus nephritis Systemic lupus erythematosus Immune complex Minimal change disease Nephritis Lupus erythematosus Endothelium • 16 B is not correct. 8 0fo c hose this . membrane thickening on light microscopy is typically seen in membranous glomerulonephritis. Membranous • 17 Diffuse capillary and basement glomerulonephritis was once the most common biopsy diagnosis in adults with nephrotic syndrome, but has since been overtaken by focal • 18 segmental glomerulosclerosis. • 19 Nephrotic syndrome Membranous glomerulonephritis Focal segmental glomerulosclerosis Basement membrane Glomerulonephritis Biopsy Ught microscopy Optical microscope Capillary Glomerulosclerosis • 20 C is no t correct . 60fo chose this . • 21 • A ...... _ ...... 1 ~ " ...... 1 ..... &..1 .... _ ...... -...... -..CA ...... : ...... &...... _: ...... -. •• : ...... &...... l;t:...... ; ...... I ...... I ...... L. ..: .. :.,. TL. .... a s 8 Lock Suspend End Block Item: 15 of 24 ~ . I • M k <:] t> al ~· ~ QIO: 5157 .l. ar Previous Next lab 'lifllues Notes Calculator

1 segmental g lomerulosclerosis. Nephrotic syndrome Membranous glomerulonephritis Focal segmental glomerulosclerosis Basement membrane Glomerulonephritis Biopsy light microscopy 2 Optical microscope Capillary Glomerulosclerosis 3 C is not correct. 60/o c hose t his. 4 A "tram -track," or double-contour, appearance of deposits can be seen on electron m icroscopy in membranoproliferative g lomerulonephritis. The 5 tram -track appearance is caused by subendothelial immune deposits and mesangial deposition in the capillary waiL The resu lt is a double­ contour appearance. Although membranoproliferative g lomerulonephritis is seen in children, it is not as common as minimal change d isease. In 6 addition, it normally manifests with hematuria and hypertension, not with edema and proteinuria as seen in this patient. 7 Membranoproliferative glomerulonephritis Hematuria Minimal change disease Proteinuria Glomerulonephritis Hypertension Mesangium Edema Electron microscope 8 Microscopy 9 0 is not correct. 60/o c hose t his. 10 Glomerular crescent formation is typical of rapid ly prog ressive g lomerulonephritis (RPGN}. RPGN is uncommon in children. In addition, it usually manifests with hematuria and hypertension instead of edema and proteinuria. 11 Rapidly progressive glomerulonephritis Hematuria Proteinuria Glomerulonephritis Hypertension Edema Glomerulus Glomerulus (kidney) Crescent

12 F is not correct. 30/o c hose t his. 13 Pink hyaline nodules, also known as Kimmelstiei-Wilson nodules, are characteristic of d iabetic nephropathy. Diabetic nephropathy has presenting 14 signs and symptoms similar to those of minimal change d isease; however, it is not commonly found in children. Diabetic nephropathy Minimal change disease Kidney disease Diabetes mellitus 15 • 16 Bottom Line : • 17 Minimal change d isease is the most common cause of childhood nephrotic syndrome. Typical signs and symptoms include periorbital and • 18 peripheral edema, massive proteinuria, hypoalbuminemia, and hyperlipidemia. Kidney biopsy appears normal on light m icroscopy, although • 19 effacement of podocyte foot processes is visible on electron microscopy. • 20 Podocyte Nephrotic syndrome Hypoalbuminemia Minimal change disease Proteinuria Hyperlipidemia Biopsy Edema Kidney Electron microscope Optical microscope Renal biopsy Fine-needle aspiration light microscopy Microscopy • 21 6 s 0 lock Suspend End Block Item: 15 of 24 ~ 1 • M k -<:J 1>- Jil ~· !:';-~ QIO: 5157 ..L a r Previous Next Labfli!llues Not es Calculat o r

1 & &

2 FA17 p566.1 3 Nephrotic syndrome 'ephrOtic syndrome-massi\e prOteinuria {> 3.:> glday) with hypoalbuminemia, resulting 4 edema, h) perlipidemia. Frothy urine" ith fatly casts. Due to podOC) te damage disrupting 5 glomerular filtration charge barrier. vlay be 1° (eg, direct sclerosis of podocytes) or 2° (systemic [eg, diabetes] secondarily damages podocytes). Associated with hypercoagulable state (eg, 6 process thromboembolism) due to antithrombin { T) Ill loss in urine and t risk of infection (due to loss of 7 immunoglobulins in urine and soft tissue compromise by edema). 8 Se,·ere nephritic syndrome may present with nephrotic S}lldrome features {nephritic-nephrotic 9 syndrome) if damage to C Bl\1 is severe enough to damage charge barrier. 10 Minimal change LM-normal glomeruli (lipid may be seen in \11ost common cause of nephrotic S)l1drome 11 disease (lipoid PCT cells). in children. Often 1° (idiopathic) and ma) be 12 nephrosis) we. triggered by recent infection, immunization, EM -effacement of foot processes fl. immune stimulus. Rarely, may be 2° to 13 lymphoma (eg, cytokine-mediated damage). I0 14 disease has excellent response to corticosteroids. 15 Focal segmental LM-segmental sclerosis and hya li nosis : . Most common cause of nephrotic syndrome in • 16 glomerulosclerosis IF- often 8, but may be (f) for nonspecific focal African Americans and Hispanics. Can be l0 • 17 deposits of Ig r, C3, C l . (idiopathic) or zo to other conditions (eg, I-I IV • 18 E If-effacement of foot process similar to infection, sickle cell disease, heroin abuse, • 19 minimal change disease . massive obesity, interferon treatment, chronic kidney disease due to congenitalmalfom1ations). • 20 1° disease has inconsistent response lo steroids . • 21 • ~ lav progress to chronic renal disease . a s 8 Lock Suspend End Block Item: 15 of 24 ~ 1 • M k -<:J 1>- Jil ~· !:';-~ QIO: 5157 ..L a r Previous Next Labfli!llues Not es Calculat o r

1 & &

2 FA17 p 563.2 3 Glomerular diseases 4 5 6 Nephritic syndrome-due to GBM diSruption Nephrotic syndrome-podocyte disruptiOn Hypertension. "' BUN and aeaumne. obguna. --. charge barner 1mpaared Massive proteinuraa 7 hematuria. RBC casts Ill unne Prote~nuna often (> 3 5g/day) w1th hypoalbum1nem1a. 8 1n the subnephtotiC range(< 3.5 g/day) but 1n hyperhpldemaa. edema May be 1' leg. direct severe cases may be in nephrotic range podocyte damage) or 2" (podocyte damage 9 • Acute poststreptococcalglomerulonephntiS rrom systemiC process leg. diabetesJ) 10 • Rapedly progressave glomerulonephnus • Focal segmental glomeruloscleroses (1" or 2') • lgA nephropathy (Berger disease) • M1n1mal change d1sease 11' or 2'1 11 • Alport syndrome • Membranous nephropathy 11" or 2'1 glomerulonephnlls • AmylOidoSIS (21 12 • Membranopromerabve • Diabetic glomerulonephropathy 12'1 13 14 15 Nephritic-nephrotic syndrome-severe nephritic syndrome • 16 with pro round GBM damage that damages the glomerular • 17 filtration charge barraer ~ nephrollc·range proteinuria(> 3.5 g/day) and concomitant reatures or nephrotic syndrome. Can • 18 occur with any rorm or nephritic syndrome. but IS most commonly seen with • 19 • D1ffuse prohrerallve glomerulonephntis • 20 • Membranoprohrerallve glomerulonephntls • 21 • a s 8 Lock Suspend End Block Item: 15 of 24 ~ 1 • M k -<:J 1>- Jil ~· !:';-~ QIO: 5157 ..L a r Previous Next Labfli!llues Not es Calculat o r

1 & & FA17 p 564.1 2 Nephritic syndrome 1\ephrltic syndrome= Inflammatory process. When glomeruli are inmlved, leads to hematuria 3 and RBC casts in urine. Associated with a~otemia, oliguria, hypertension (due to salt retention), 4 proteinuria. 5 Acute Uvl-glomerul i enlarged and h) percellular fl. lost frequently seen in children. Occurs 6 poststreptococca I IF' - ("starry sky") granular appearance - 2--+ weeks after group A streptococcal 7 glomerulonephritis ("lumpy-bumpy") [l] due to lgG, lg \ I, and C3 infection of pharynx or skin. Resolves 8 deposition along GB I and mesangium. spontaneouslr Type Ill h~ persensiti' it) 9 E~1-subepithelial immune complex (IC) reaction. humps. Presents with peripheral and periorbita I edema, 10 cola-colored urine, hypertension. 11 Positive strep titers/serologies, ~ complement 12 b·els (C3) due to consumption. 13 Rapidly progressive LM and IF' -crescent moon shape B. Crescents Poor prognosis. Rapidly deteriorating renal 14 (crescentic) consist of fibrin and plasma proteins (eg, C3b) function (days to weeks). 15 glomerulonephritis with glomerular parietal cell s, monocytcs, • 16 macro phages. Several disease processes may result in this • 17 pattern, in particular: I . 18 Goodpasture syndrome-type II llematuria /hemoptys is. • 19 hypersensitivity reaction; antibodies to Treatment: emergent plasmapheresis. • 20 GB I and alveolar basement membrane • 21 - linear IF • • r--- _ ___ _J ___ ._ __ : ___ • .: ._L __ L ____:: a: 1\ \ 1 _ -·-' 1"\n-, ,,, ,.... , t_ .t'lr"'l n _ __ _ .: : ______, __ . _ ,,...... , a s 8 Lock Suspend End Block Item: 16 of 24 ~ 1 • M k -<:J 1>- Jil ~· !:';-~ QIO: 3490 ..L a r Previous Next Labfli!l tues Not es Calculat o r

1 & & A 63-year-old man is seen by his doctor after measuring his blood pressure at home and finding it to be 168/100 mm Hg. The patient is 2 concerned because he has had high blood pressure for the past 12 months that has not improved with dietary changes, exercise, or examination is unremarkable except for a bruit heard on auscultation of the abdomen just to the left of the midline. 3 medication. A physical 4 What laboratory finding is most likely to be seen in this patient? 5

6 A. Decreased levels of ADH 7 8 B. Decreased levels of angiotensin II

9 C. Hypernatremia 10 D. Hypokalemia 11 E. Increased levels of angiotensin-converting enzyme 12

13 14

15 • 16

• 17

• 18 • 19

• 20

• 21 • a s 8 Lock Suspend End Block Item: 16 of 24 ~ . I • M k <:] t> al ~· ~ QIO: 3490 .l. ar Previous Next lab 'lifllues Notes Calculator

1 • The correct a nswer is 0. 3 4 0/o c hose t his. 2 The patient most likely has hypertension and hypokalemia as a resu lt of the unilateral stenosis of a renal artery. Renal artery stenosis is usually 3 caused by atherosclerosis or fibromuscular dysplasia, and the affected kidney usually becomes atrophic. Stenosis of this vessel causes turbulent flow, resu lting in the abdominal thrill detected in this patient. The decreased perfusion of the kidney causes juxtaglomerular cells to release ren in, 4 which cleaves angiotensinogen into angiotensin L Angiotensin -converting enzyme (ACE} in alveolar capillaries converts angiotensin I to 5 angiotensin II, which stimulates vasoconstriction, sympathetic activity, tubular reabsorption of sodium and water, secretion of antidiuretic hormone (ADH} from the posterior pituitary, and secretion of aldosterone from the adrenal cortex. ADH increases water absorption from the 6 collecting duct, whereas aldosterone leads to sodium-potassium exchange (sodium out of urine and potassium into urine) and water retention. All 7 of these things combined lead to hypokalemia as well as hypertension. Angiotensinogen Hypokalemia Renal artery stenosis Atherosclerosis Aldosterone Adrenal cortex Vasoconstriction Angiotensin II Renin Angiotensin I Enzyme 8 Juxtaglomerular cell Posterior pituitary Vasopressin Angiotensin Renal artery Autonomic nervous system Hypertension Sympathetic nervous system 9 Collecting duct system Angiotensin-converting enzyme Hormone Sodium Kidney Capillary Renal physiology Urine Pituitary gland Perfusion Potassium Turbulence

10 Stenosis Atrophy 11 A is not correct. ]Ofo c hose t his. 12 This patient has an abdominal bruit near the kidney reg ion, a classic sign of renal artery stenosis. Renal artery stenosis decreases overall kidney perfusion and would thus lead to an increase in antidiuretic hormone (ADH} release from the posterior pituitary. In response to decreased 13 perfusion, the j uxtaglomerular cells of the afferent arteriole release ren in, which cleaves angiotensinogen into angiotensin L In the alveolar 14 capillaries, angiotensin-converting enzyme converts angiotensin I to angiotensin II, which stimulates the release of ADH from the posterior 15 pituitary. Angiotensinogen Arteriole Renal artery stenosis Afferent arterioles Angiotensin-converting enzyme Angiotensin Angiotensin II Angiotensin I Renin 16 Juxtaglomerular cell Vasopressin Renal artery Posterior pituitary Enzyme Kidney Capillary Hormone Pituitary gland Perfusion • 17 B is not correct. 90fo c hose t his . • 18 This patient has an abdominal bruit near the kidney reg ion, a classic sign of renal artery stenosis. Renal artery stenosis would lead to increased • 19 levels of angiotensin IL Decreased perfusion of the kidney causes j uxtaglomerular cells of the afferent arteriole to release ren in . Ren in then cleaves angiotensinogen into angiotensin I, which is later converted into angiotensin II by angiotensin-converting enzyme in alveolar capillaries . • 20 Angiotensinogen Arteriole Renal artery stenosis Afferent arterioles Angiotensin Angiotensin-converting enzyme Angiotensin I Angiotensin II Enzyme

• 21 Juxtaglomerular cell Renal artery Renin Kidney Capillary Perfusion Stenosis • 6 s 0 lock Suspend End Block Item: 16 of 24 ~ . I • M k <:] t> al ~· ~ QIO: 3490 .l. ar Previous Next lab 'lifllues Notes Calculator

1 • capillaries, angiotensin-converting enzyme converts angiotensin I to angiotensin II, which stimulates the release of ADH from the posterior pituitary. 2 Angiotensinogen Arteriole Renal artery stenosis Afferent arterioles Angiotensin-converting enzyme Angiotensin Angiotensin II Angiotensin I Renin 3 Juxtaglomerular cell Vasopressin Renal artery Posterior pituitary Enzyme Kidney Capillary Hormone Pituitary gland Perfusion

4 B is no t correct. 90fo c hose this. 5 This patient has an abdominal bruit near the kidney reg ion, a classic sign of renal artery stenosis. Renal artery stenosis would lead to increased levels of angiotensin IL Decreased perfusion of the kidney causes j uxtaglomerular cells of the afferent arteriole to release ren in . Ren in then 6 cleaves angiotensinogen into angiotensin I, which is later converted into angiotensin II by angiotensin-converting enzyme in alveolar capillaries. 7 Angiotensinogen Arteriole Renal artery stenosis Afferent arterioles Angiotensin Angiotensin-converting enzyme Angiotensin I Angiotensin II Enzyme 8 Juxtaglomerular cell Renal artery Renin Kidney Capillary Perfusion Stenosis 9 C is no t correct. 20 0/o c hose this. 10 Renal artery stenosis ind irectly leads to an increase in aldosterone because of angiotensin II acting on the zona g lomerulosa of the adrenal cortex to increase aldosterone production . Aldosterone will lead to sodium retention, creating a favorable sodium g rad ient for water reabsorption . 11 Because hypernatremia is caused by a free-water deficit, the patient will compensate as long as his access to water is not restricted. Therefore, 12 sodium levels will remain normaL Zona glomerulosa Renal artery stenosis Hypernatremia Aldosterone Adrenal cortex Angiotensin II Angiotensin Renal artery Sodium Na+/K+-ATPase Kidney 13 Adrenal gland Electrochemical gradient 14 E is no t correct. 3 0 0/o c hose this. 15 Renal artery stenosis would not have an effect on the levels of angiotensin-converting enzyme in the lungs; however, more angiotensin I would 16 be available and therefore there would also be an increase in angiotensin IL Renal artery stenosis Angiotensin-converting enzyme Angiotensin Angiotensin II Enzyme Angiotensin I Renal artery Kidney Stenosis lung • 17 • 18 • 19 Bo tto m Line : • 20 Renal artery stenosis can lead to hypertension due to activation of the ren in -angiotensin -aldosterone system . Renal artery stenosis Renal artery Hypertension Renin-angiotensin system Stenosis Kidney • 21 • 6 s 0 lock Suspend End Block Item: 16 of 24 ~ 1 • M k -<:J 1>- Jil ~· !:';-~ QIO: 3490 ..L a r Previous Next Labfli!l tues Not es Calculat o r

1 & & FA17 p 558.1 2 3 Renin-angiotensin-aldosterone system

4 I BP (JG cells) 5 I Na • delivery Acts at angiotensin II - -+ Vasoconstriction - -+ 1 BP (macula densa cells) ) 6 receptor. type 1 (AT1 on vascular smooth muscle 7 t sympathe!JC tone -receptors) 8 Constncts efferent t FF to preserve renal function (GFRI ~--v.::----'' of glomerulus 9 arteriOle - in low-volume states (ie. when RBF ! ) 10 ~ B dyk .. Remn ACE --+ ra lnln Aldosterone t Na• channel insertion and - -+ Creates I breakdown 11 (adrenal cortex) I activity of Na•/K' pump; favorable Na· 12 Angiotensinogen - -+ Angiotensin I - -+ enhances K-and H- gradient for excretion by way of principal Na• and Hp 13 cell K' channels and reabsorption 14 ex-intercalated cell H• ATPases ...--.. AOH 15 {posterior ______,.. I aquaporin insertion in -----+ Hp principal cells reabsorption 16 pituitary)

• 17 ---+ Na'. HC03-. and Hp reabsorption 1 PCT Na '/H ' activity {can permit contraction alkalosis) . 18 Stimulates hypothalamus --+ Thirst • 19 • 20 Renin Secreted by JG cells in response to l renal arterial pressure, t renal sympathetic discharge (P1 • 21 • effect). and l Na.,. deliverv to macula densa cells . a s 8 Lock Suspend End Block 16 24 • M k ~ £!1}>' !!":-~ Item: of ~ 1 a r -<:J I> • QIO: 3490 ..L Previous Next Lab lues Not es Calculat o r

1 & & FA17 p 559.2 2 3 Hormones acting on kidney 4 • na e •c '"'e"' 1e Seaeted in response toi atrial pressure Causes i GFR 5 and i Na· filtration with no compensatory Na· reabsorpbon nephron effect Na' loss and volume loss 6 in distal Net Distal Glomerulus convoluted (k• 7 .... tubule 0 8 Alferent 9 Efferent

10 \..__ K H 11 Secreted 1n response to Angiotensin II .!. blood volume (v.a AT II) and 12 Synthesized in response to .!. BP Causes efferent arteriole i plasma (K'I. causes iNa• constnction-+ f GFR and i FF but with compensatory Na· Ca reabsorption. iK• secret•on. 13 ,r- Mg reabsoo'pbon in proximal and distal nephron. Net effect: Cortex i H' secretion. 14 preservation of renal function If FF) in low-volume state - - proximal with simultaneous Na• reabsorption {both Medulla ADH (v sopr s 1) and distal) to maintain circulating volume. Na' 15 K Secreted in response to 20 i plasma osmolarity and 16 Parathyroid hormone Ascending Umb. .!. blood volume Binds to • 17 Secreted 1n response to receptors on principal cells. 1 loop of Henle .!. plasma (Ca ']. i plasma IP04 >-), causing f number of . 18 {por,.•bl• 10 wllll or.!. plasma 1.25-(0H)z 03. aquaponnsandi H10 Causes ICa1' ] reabsorption IOCn. • 19 i reabsorption .!. !POl l reabsorptiOn IPCll. and • 20 i 1.25-(0Hb 03producoon Collecting duct If Ca'' and P04 absorption from gut • 21 v.a vllamon 0) • a s 8 Lock Suspend End Block Item: 17 of 24 ~ 1 • M k -<:J 1>- Jil ~· !:';-~ QIO: 4693 ..L a r Previous Next Labfli!l tues Not es Calculat o r

1 & & A newborn girl is diagnosed as dysmorphic by a pediatrician in the newborn nursery. On physical examination the girl has a broad neck, 2 wide-spaced nipples, and a systolic ejection murmur. An echocardiogram is performed and demonstrates coarctation of the aorta. The also runs his transducer across the patient's abdomen and notices a renal abnormality associated with this 3 echocardiography technologist patient's syndrome. 4 5 The most likely observed renal abnormality increases this patient's risk for developing which disease? 6 : 7 A. Cervical cancer 8 B. Neuroblastoma 9 C. Polycystic kidney disease 10 11 0. Urinary tract infection

12 E. Uterine cancer 13 14 15 16 • 17 • 18 • 19 • 20 • 21 • a s 8 Lock Suspend End Block Item: 17 of 24 ~ 1 • M k -<:J 1>- Jil ~· !:';-~ QIO: 4693 ..L a r Previous Next Labfli!l tues Not es Calculat o r

1 & & The c o rrect a ns w e r is D. 530/o chose this . 2 This patient has classic findings associated with Turner syndrome, a genetic disorder resulting from a 3 45XO complement of chromosomes. Approximately 30%- 40% of patients will have associated renal 4 anomalies. The most common abnormalities are collecting-system malformations (20%) and horseshoe kidney (10%, shown in the image), followed by malrotation and other positional 5 abnormalities (5%). Patients should undergo renal ultrasonography at the time of diagnosis of 6 Turner syndrome, and if structural abnormalities are identified, they should be monitored yearly for urinary tract infection or when clinical suspicion warrants investigation. 7 Urina y t ct infection Turner syndrome Horseshoe kidney Genetic disorder Urina y system Medical ultrasound 8 Kidney Chromosome Intestinal malrotation Horseshoe Infection 9 10 11 12 13 14

15 A is not co rre ct. 50/o c hose this. 16 Turner syndrome patients do n ot have a specifiic increased risk of developing cervical cancer. Patients t hat are exposed to hu man papillomavirus have an increased risk of cervical cancer. 17 Turner syndrome Human papillomavirus Cervical cancer Cervix Cancer • 18 B is not co rre ct . 11Ofo chose this . • 19 Although it has been suggested that patients with Turner syndrome are at increased risk of developing malignancy, they do not have a specific • 2 0 increased risk of developing neuroblastoma. Neuroblastoma is a feature of neurofibromatosis type 1 and Beckwith-Wiedemann syndrome. syndrome Turner syndrome Neuroblastoma Neurofibromatosis type I Neurof'bromatosis Malignancy cancer • 21 Beckwith-Wiedemann • a s 8 Lock Suspend End Block Item: 17 of 24 ~ . I • M k <:] t> al ~· ~ QIO: 4693 .l. ar Previous Next lab 'lifllues Notes Calculator

1 A is not correct. 50fo c hose t his. 2 Turner syndrome patients do not have a specifiic increased risk of developing cervical cancer. Patients that are exposed to human papillomavirus 3 have an increased risk of cervical cancer. 4 Turner syndrome Human papillomavirus Cervical cancer Cervix Cancer 5 B is not correct. 11 Ofo c hose t his. Although it has been suggested that patients with Turner syndrome are at increased risk of developing malignancy, they do not have a specific 6 increased risk of developing neuroblastoma. Neuroblastoma is a feature of neurofibromatosis type 1 and Beckwith -Wiedemann syndrome. 7 Beckwith-Wiedemann syndrome Turner syndrome Neuroblastoma Neurofibromatosis type I Neurofibromatosis Malignancy Cancer

8 C is not correct. 260/o c hose t his. 9 Po lycystic kidney d isorder can be either an autosomal-dominant or an autosomal-recessive inherited d isorder. The autosomal-dominant d isorder presents with hypertension, renal insufficiency, and renal cysts later in life. The autosomal-recessive version is often fatal in utero or within the 10 first month of life, and often presents with symptoms of oligohydramnios. It is not associated with characteristics of Turner syndrome. 11 Oligohydramnios Turner syndrome Polycystic kidney disease Hypertension Dominance (genetics) Genetic disorder Kidney In utero 12 Focal segmental glomerulosclerosis Cyst 13 E is not correct. 50fo c hose t his. 14 Individuals with Turner syndrome do not have a specific increased risk of developing uterine cancer. Un opposed estrogen secretion (eg, hormone replacement therapy without progesterone) increases the risk of uterine cancer. 15 Turner syndrome Estrogen Progesterone Uterine cancer Hormone replacement therapy (male-to-female) Hormone replacement therapy (menopause) 16 Hormone replacement therapy Hormone Cancer Uterus 17

• 18 Bottom Line : • 19 Turner syndrome is associated with horseshoe kidney in addition to collecting system malformations and positional abnormalities. Due to an • 20 increased chance of collecting system abnormalities, ultrasonography is warranted at the time of d iagnosis . Turner syndrome Horseshoe kidney Medical ultrasound Kidney Horseshoe • 21 6 s 0 lock Suspend End Block Item: 17 of 24 ~ 1 • M k -<:J 1>- Jil ~· !:';-~ QIO: 4693 ..L a r Previous Next Labfli!l tues Not es Calculat o r

1 & & FA17p 549.1 2 Horseshoe kidney Inferi or poles of both kidne) s fuse 3 abnormally fJ. As they ascend from pelvis 4 during fetal development, horseshoe kidneys 5 get trapped under inferior mesenteric 6 artery and remain lo\\ in the abdomen. 7 Kidners function normally. Associated ,,·ith hydronephrosis (eg, ureteropelvic 8 lnfenor mesentenc junction obstruction), renal stones, infection, 9 artery ~ chromosomal aneuploidy syndrome) (eg, 10 Turner syndrome; trisomies 13, 18, 21), and 11 rareh-• renal cancer. 12

13 FA17 p 569.1 14 Nephroblastoma Most common renal malign- Jil ~· !:';-~ QIO: 4747 ..L a r Previous Next Labfli!llues Not es Calculat o r

1 & & A 67-year-old man is brought to the clinic by his children, who are concerned about him passing "blood in the urine" for the past several 2 days. The patient is reluctant to visit a doctor, stating that he does not feel any pain. His children state that their father had been an years ago. He has always been a heavy smoker. The 3 agricultural worker in their native Egypt before immigrating to the United States a few patient's physical examination is normal, and a urinalysis reveals hematuria with numerous epithelial cells per high-powered field. An intravenous 4 pyelogram shows irregular-shaped filling defects above the trigone, with several regions of focal calcifications in the bladder wall. The physician 5 informs the patient that his problems may have been caused by a chronic infection. 6 Infection with which organism is most likely to have contributed to the patient's condition? 7 8 : A. Dracunculus medinensis 9 10 B. Necator americanus

11 C. Schistosoma haematobium 12 D. Schistosoma mansoni 13 E. Taenia solium 14 15 16 17 . 18 . 19 • 20 . 21 • a s 8 Lock Suspend End Block Item: 18 of 24 ~ . I • M k <:] t> al ~· ~ QIO: 4747 .l. ar Previous Next Lab'lifllues Notes Calculator

1 • The correct answer is C. 780/o chose t his. 2 Schistosoma hematobium is a parasitic flatworm found most commonly in North Africa and the Middle East, where it infects farmers and agricultural workers. Freshwater snails serve as the intermediate host, and humans and other mammals serve as the definitive host. In the 3 human, S. haematobium m ig rates to the urinary bladder, where it can be completely asymptomatic, causing chronic inflammation of the bladder 4 wall or, occasionally, persistent hematuria. Ch ron ic bladder wall schistosomiasis is associated with squamous cell carcinoma of the bladder, particularly in ind ividuals with other risk factors (eg, advanced age, smoking) from reg ions with a high prevalence of Schistosoma parasites, such 5 as this patient. Resu lts of the intravenous pyelog ram are consistent with bladder cancer, and the areas of schistosomiasis infection in the bladder 6 wall are seen as areas of focal calcification during cystography. Praziquantel is the d rug of choice to treat acute or ongoing schistosomiasis infection . 7 Flatworm Intravenous pyelogram Schistosomiasis Praziquantel Hematuria Parasitic life cycles Bladder cancer Squamous-cell carcinoma Schistosoma haematobium

8 Urinary bladder Host (biology) Intermediate host Schistosoma Parasitism Intravenous therapy Middle East Inflammation Infection Calcification Asymptomatic 9 Africa Squamous epithelial cell Carcinoma Mammal Cancer

10 A is not correct. 30fo chose t his. 11 Dracunculus medinensis, commonly known as the Gu inea worm, is a parasite commonly found in Africa. The parasitic worm usually resides in subcutaneous tissue of humans, and the tail end of the adult female worm often protrudes from a small ulcer, commonly found in the lower lim b 12 of infected ind ividuals, where offspring are released into the water. Dracunculus infection causes cutaneous nodules and ulceration, but does not 13 cause systemic illness such as bladder carcinoma. Helminths Subcutaneous tissue Dracunculus medinensis Parasitism Urinary bladder Ulcer {dermatology) Bladder cancer Systemic disease 14 15 B is not correct. 30fo chose t his. Necator americanus is a hookworm more commonly found in the Americas, sub-Saharan Africa, and Southeast Asia. Its primary infective 16 population is children, where chronic hookworm infection is associated with malnutrition, mental retardation, abdominal pain, and d iarrhea. It 17 rarely causes chronic infection and never manifests as urinary d isease. Hookworm infection Necator americanus Southeast Asia Diarrhea Sub-Saharan Africa Abdominal pain Dysuria Malnutrition Intellectual disability Americas Africa 18 Infection Asia • 19 0 is not correct. 130/o chose t his . • 20 Schistosoma mansoni, although another flatworm with similar infection patterns and life cycles to S. haematobium, is not associated with bladder • 21 infection or the development of squamous cell carcinoma of the bladder. The S. mansoni parasite is more common in sub-Saharan Africa and • 6 s 0 lock Suspend End Block Item: 18 of 24 ~ . I • M k <:] t> al ~· ~ QIO: 4747 .l. ar Previous Next Lab'lifllues Notes Calculator

1 B is no t corre ct . 30fo c hose t his. 2 Necator americanus is a hookworm more commonly found in the Americas, sub-Saharan Africa, and Southeast Asia. Its primary infective 3 population is children, where chronic hookworm infection is associated with malnutrition, mental retardation, abdominal pain, and d iarrhea. It rarely causes chronic infection and never manifests as urinary d isease. 4 Hookworm infection Necator americanus Southeast Asia Diarrhea Sub-Saharan Africa Abdominal pain Dysuria Malnutrition Intellectual disability Americas Africa 5 Infection Asia

6 0 is no t corre ct . 130/o c hose t his. 7 Schistosoma mansoni, although another flatworm with similar infection patterns and life cycles to S. haematobium, is not associated with bladder infection or the development of squamous cell carcinoma of the bladder. The S. mansoni parasite is more common in sub-Saharan Africa and 8 South America, and is associated with intestinal schistosomiasis, intestinal cancer, liver infection, and Katayama fever. 9 Flatworm Schistosomiasis Schistosoma mansoni Bladder cancer Squamous-cell carcinoma Schistosoma haematobium Schistosoma Colorectal cancer liver 10 Parasitism Sub-Saharan Africa Cancer Urinary bladder Fever South America Urinary tract infection Infection Carcinoma Africa

11 E is no t corre ct . 30fo c hose t his. Taenia solium, also known as pork tapeworm, is a flatworm that infects pigs and humans in Asia, Africa, South America, and parts of North 12 America. These worms have long, slender, flat bod ies composed of units called prog lottids and a single head with numerous hooks and suckers 13 designed to attach itself to the intestinal wall of the host. Ch ron ic infection with tapeworm causes intestinal symptoms; occasionally, m ig ration of 14 worms to the brain can cause neurocysticercosis, which can resu lt in major neurologic problems. Flatworm Neurocysticercosis Cestoda Taenia solium Taenia (genus) Pork Sucker (zoology) Proglottids Brain South America Asia Africa Infection 15 16 Bo tto m Line : 17 Schistosoma haematobium m ig rates to the urinary bladder, where it can be completely asymptomatic while causing chronic inflammation of the 18 bladder wall or, occasionally, persistent hematuria. Ch ron ic bladder wall schistosomiasis is associated with squamous cell carcinoma of the • 19 bladder. Schistosomiasis Schistosoma haematobium Bladder cancer Hematuria Urinary bladder Squamous-cell carcinoma Schistosoma Carcinoma Inflammation • 20 Squamous epithelial cell Asymptomatic • 21 6 s 0 lock Suspend End Block Item: 18 of 24 ~ .I • M k <:] t> al ~· ~ QIO: 4747 .l. ar Previous Next Lab'lifllues Notes Calculator

1 • • FA17 p 154.1 2 Protozoa--others 3 ORGANISM OISEASE TRANSMISSION DIAGNOSIS TREATMENT 4 Visceral infections 5 Trypanosoma Chagas disease-dilated Reduviid bug Trypomastigotc in Bcnznidazole 6 cruzi cardiomyopathy with ("kissing bug") blood smear a or nifurtimox; 7 apical atrophy, megacolon, feces, deposited C ruzing in my megaesophagus; predominantly in a painless bite Benz, with a fur 8 in South America (much like a kiss) coat on 9 Unilateral periorbita 1 swelling 10 (Romaiia sign) characteristic of 11 acute stage 12 Leishmania Visceral leishmaniasis SandAy Macrophages Amphotericin B, 13 donovani (kala-azar)- spiking fevers, containing sodium 14 hepatosplenomegaly, amastigotes rn stibogluconate 15 pancytopenia Cutaneous leishmaniasis-skin 16 ulcers 17 18 • 19 Sexually transmitted infections • 20 Trichomonas Vaginitis-foul-smelling, green ish Sexual (cannot exist Trophozoites Metronidazole for • 21 • vaainalis discharge: itching and burning: outside human (motile) [il on wet oatient and nartner • 6 s 0 lock Suspend End Block Item: 18 of 24 ~ 1 • M k -<:J 1>- Jil ~· !:';-~ QIO: 4747 ..L a r Previous Next Labfli!llues Not es Calculat o r

& & 1 FA17 p 156.2 2 Trematodes (flukes) 3 ORGANISM DISEASE TRANSMISSION TREATMENT 4 Schistosoma Liver and spleen enlargement Snails arc host; cercariae Praziquantel 5 (S mansoni, egg with penetrate skin of humans lateral spine t'J), fibrosis, 6 inflammation, portal 7 hypertension 8 Chronic infection with

9 0 0 S haematobium (egg'' ith [l] o . 10 0 terminal spine · ) can lead to squamous cell carcinoma 11 of the bladder (painless 12 hematuria) and pulmonary 13 hypertension 14 Clonorchis sinensis Bi liary tract inflammation Undcrcooked fi sh Praziquantel

15 -+ pigmented ga llstones 16 Associated with cholangiocarcinoma 17 18 • 19 FA17 p 569.3 • 20 Squamous cell irritation of urinary bladder - squamous metaplasia - dysplasia and squamous cell • 21 Chronic • a s 8 Lock Suspend End Block Item: 19 of 24 ~ 1 • M k -<:J 1>- Jil ~· !:';-~ QIO: 4166 ..L a r Previous Next Labfli!l tues Notes Calculat o r

1 & & A 36-year-old man presents with dyspnea and hemoptysis of recent onset. X-ray of the chest shows diffuse infiltrates. He has a negative 2 sputum culture and is unresponsive to steroid therapy. A kidney biopsy is performed and the findings suggest that an antibody is bound to month later, the man dies of renal failure. 3 type IV collagen . A 4 Which of the following findings would have preceded this patient's renal failure? 5 6 A. 7 8 B. Ketones

9 C. Microscopic hematuria 10 D. Proteinuria ~ 3.5g/day 11 E. Squamous epithelial cells 12 13 14 15 16 17 18 . 19 • 20 . 21 • a s 8 Lock Suspend End Block Item: 19 of 24 ~ 1 • M k -<:J 1>- Jil ~· !:';-~ QIO: 4166 ..L a r Prev ious Next Lab fli!ltues Notes Calculat or

1 & The correct a nswer is C. 71 Ofo chose this. & to 2 The symptoms strongly suggest Goodpasture syndrome. Goodpasture syndrome causes lesions both glomerulus and lung as a result of autoantibodies directed against type IV collagen, a shared 3 protein in the alveolar and glomerular basement membranes. In the kidney it causes a rapidly 4 progressive glomerulonephritis, as the antibodies cause an immune attack on the basement membrane (as demonstrated in the IF image). Microscopic hematuria is a common sign of 5 glomerulonephritis. 6 Hematu 1a Basement membrane Collagen Rapidly progressive glomerulonephritis Type IV collagen Glomerulonephntis Goodpasture syndrome Kidney Autoantibody Protein Antibody Glomerulus (kidney) 7 Glomerulus Pulmonary alveolus Lung 8 9

10 Image© 2010 Patel eta/; licensee BioMed 11 Central Ltd.

12 A i s not correct. 1 Ofo chose this. 13 Bacteriuria would be characteristic of a urinary tract infection (UTI). UT!s generally present with dysuria, frequency, urgency, and suprapubic 14 pain. Diagnosis is achieved with the above signs and symptoms and is supported by bacteriuria or pyuria on urinalysis. Urine cultures are generally not required for d iagnosis but can be used to identify the causative organism and to aid in antibiotic selection. E. coli is by far the most 15 common causative agent (approximately 75-95% ). Of note, UT!s are uncommon in men and this patient has no characteristic symptoms of a UTI. 16 Additionally, he has lung involvement (dyspnea and hemoptysis) making this answer choice less likely. Bacterial infection is not involved in the pathology of Goodpasture disease, therefore bacteriuria should not be seen on urinalysis. 17 Urinary tract infection Dysuria Pyuria Hemoptysis Dyspnea Escherichia coli Bacteriuria Antibiotics Urinalysis Urinary system urine Pathogenic bacteria Pathology 18 Infection

19 B is n ot correct . 1 Ofo ch ose this • • 20 is the result of excessive metabolism of fatty acids. Classically, it is associated with type I diabetics who present with polyuria, who are found to be in diabetic ketoacidosis (DKA). These patients have an absolute insulin deficiency and as result, • 21 polydipsia, and weight loss • nl11rnc:P r~nnnt hP • •tili7Prl ~c:; ;::t f11PI c:;n11rrP F;::!lt thPn hPrnmP<: thP nrPtinmin;\nt hnrlv f11PI ;::f!nrt PYrPc:;c:;ivP fA~ttv ~rirl mPtA~hnlic::m (p;ulc: tn fnrm-=-tinn a s 8 Lock Suspend End Block Item: 19 of 24 ~ . I • M k <:] t> al ~· ~ QIO: 4166 .l. ar Previous Next lab 'lifllues Notes Calculator

1 • Bacteriuria would be characteristic of a urinary tract infection {UTI}. UT!s generally present with dysuria, frequency, urgency, and suprapubic pain. Diagnosis is achieved with the above signs and symptoms and is supported by bacteriuria or pyuria on urinalysis. Urine cultures are 2 generally not required for d iagnosis but can be used to identify the causative organism and to aid in antibiotic selection. E. coli is by far the most 3 common causative agent (approximately 75-95% ). Of note, UT!s are uncommon in men and this patient has no characteristic symptoms of a UTI. 4 Additionally, he has lung involvement {dyspnea and hemoptysis) making this answer choice less likely. Bacterial infection is not involved in the pathology of Goodpasture d isease, therefore bacteriuria should not be seen on urinalysis. 5 Urinary tract infection Dysuria Pyuria Hemoptysis Dyspnea Escherichia coli Bacteriuria Antibiotics Urinalysis Urinary system Urine Pathogenic bacteria Pathology 6 Infection

7 B is not correct. 1 Ofo chose this. 8 Ketonuria is the resu lt of excessive metabolism of fatty acids. Classically, it is associated with type I d iabetics who present with polyuria, polyd ipsia, and weight loss who are found to be in d iabetic ketoacidosis (DKA). These patients have an absolute insulin deficiency and as resu lt, 9 g lucose cannot be utilized as a fuel source. Fat then becomes the predominant body fuel, and excessive fatty acid metabolism leads to formation 10 of ketone bod ies - acetoacetic acid, beta-hydroxybutyric acid, and acetone. Excess ketone bod ies then spill into the urine ind icating the body is using fat as the maj or fuel source. Some other conditions where ketonuria is seen include starvation, hyperthyroidism, and pregnancy 11 (gestational DM}. This patient lacks signs or symptoms associated with the above d iseases and has some pathology involving the lung 12 (hemoptysis and dyspnea) ind icating a d ifferent d isease process is occu rring . Therefore, ketonuria is unlikely to be seen. Diabetic ketoacidosis Acetone Hyperthyroidism Polyuria Acetoacetic acid Ketonuria Hemoptysis Ketone bodies Beta-Hydroxybutyric acid Dyspnea Fatty acid 13 Polydipsia Fatty acid metabolism Ketone Ketoacidosis Insulin Diabetes mellitus Glucose Metabolism Urine Weight loss lung Pathology Fat Pregnancy 14 0 is not correct. 250/o chose this. 15 Heavy proteinuria is characteristic of the nephrotic syndrome, in which 3 .5 g (or more) of protein is lost in the urine per day. Although 16 Goodpasture patients may have some proteinuria, it is generally in the subnephrotic range. 17 Nephrotic syndrome Proteinuria Urine Protein

18 E is not correct. 20/o chose this. Squamous epithelial cells are found in the d istal urethra or external genitalia. They are generally absent on urinalysis and represent 19 contamination or poor sampling when present. Squamous epithelial cells would not be associated with Goodpasture d isease and if present, would 0 20 on ly ind icate contamination. Urethra Epithelium Anatomical terms of location Sex organ Squamous epithelial cell Urinalysis 0 21 • 6 s 0 lock Suspend End Block Item: 19 of 24 ~ . I • M k <:] t> al ~· ~ QIO: 4166 .l. ar Previous Next lab 'lifllues Notes Calculator

.- 1 1 .- 1 1 1 Infection 2 B is no t correct . 1 Ofo c hose t his. 3 Ketonuria is the resu lt of excessive metabolism of fatty acids. Classically, it is associated with type I d iabetics who present with polyuria, 4 polyd ipsia, and weight loss who are found to be in d iabetic ketoacidosis (DKA). These patients have an absolute insulin deficiency and as resu lt, g lucose cannot be utilized as a fuel source. Fat then becomes the predominant body fuel, and excessive fatty acid metabolism leads to formation 5 of ketone bod ies - acetoacetic acid, beta-hyd roxybutyric acid, and acetone. Excess ketone bod ies then spill into the urine ind icating the body is 6 using fat as the maj or fuel source. Some other conditions where ketonuria is seen include starvation, hyperthyroidism, and pregnancy (gestational DM}. This patient lacks signs or symptoms associated with the above d iseases and has some pathology involving the lung 7 (hemoptysis and dyspnea) ind icating a d ifferent d isease process is occu rring . Therefore, ketonuria is unlikely to be seen. 8 Diabetic ketoacidosis Acetone Hyperthyroidism Polyuria Acetoacetic acid Ketonuria Hemoptysis Ketone bodies Beta-Hydroxybutyric acid Dyspnea Fatty acid 9 Polydipsia Fatty acid metabolism Ketone Ketoacidosis Insulin Diabetes mellitus Glucose Metabolism Urine Weight loss lung Pathology Fat Pregnancy 10 0 is no t correct . 250/o c hose t his.

11 Heavy proteinuria is characteristic of the nephrotic syndrome, in which 3 .5 g (or more) of protein is lost in the urine per day. Although Goodpasture patients may have some proteinuria, it is generally in the subnephrotic range. 12 Nephrotic syndrome Proteinuria Urine Protein

13 E is no t correct . 20/o c hose t his. 14 Squamous epithelial cells are found in the d istal urethra or external genitalia. They are generally absent on urinalysis and represent 15 contamination or poor sampling when present. Squamous epithelial cells would not be associated with Goodpasture d isease and if present, would on ly ind icate contamination. 16 Urethra Epithelium Anatomical terms of location Sex organ Squamous epithelial cell Urinalysis 17

18 Botto m Line : 19 Goodpasture syndrome resu lts from anti-type IV collagen antibodies and most commonly manifests its kidney pathology as nephritic syndrome

0 20 - not nephrOtic syndrome. Thus, it is more likely to present with microscopic hematuria. Hematuria Collagen Kidney Antibody Goodpasture syndrome Pathology Microhematuria 0 21 6 s 0 lock Suspend End Block Item: 19 of 24 ~ 1 • M k -<:J 1>- Jil ~· !:';-~ QIO: 4166 ..L a r Previous Next Labfli!l tues Notes Calculat o r

1 & & FA17 p 564.1 2 Nephritic syndrome l\ephrltic syndrome= InAammatory process. When glomeruli are inml\'ed, leads to hematuria 3 and RBC casts in urine. Associated with a~o t emia, oliguria, hypertension (due to salt retention), 4 proteinuria. 5 Acute L.M -glomeruli enlarged and h) percellular rJ. lost frequently seen in children. Occurs 6 poststre ptococca I IF- (.. s tarry sky") granular appearance - 2-4 weeks after group A streptococcal 7 glo me rulone phritis ("lumpy-bumpy") [lJ due to lgC, lg\ I, and C3 infection of pharynx or skin. Resolves 8 deposition along GB I and mesangium. spontaneous!). Type Ill h) persensiti' it) 9 E~1-subepithelial immune complex (IC) reaction. humps. Presents with peripheral and periorbital edema, 10 cola-colored urine, hypertension. 11 Positi,·e strep titers/serologies, l complement 12 b ·cls (C3) due to consumption. 13 Rapidly progressive LM and IF- crescent moon shape [j. Crescents Poor prognosis. Rapidly deteriorating renal 14 (crescentic) consist of fibrin and plasma proteins (eg, C3b) function (days to weeks). 15 glomerulo ne ph rit is with glomerular parietal cel ls, monocytes, macro phages. 16 Several d isease processes may result in th is 17 pallern, in particular: 18 • Goodpasture syndrome-type II llemaluria/hemoptysis. 19 hrpersensiti\'ity reaction; antibodies to Treatment: emergent plasmapheresis. • 20 CBl\1 and alveolar basement membrane -+ linear IF . 21 • a s 8 Lock Suspend End Block Item: 20 of 24 ~ 1 • M k -<:J 1>- Jil ~· !:';-~ QIO: 4163 ..L a r Previous Next Labfli!l tues Not es Calculat o r

1 & & A 22-year-old woman complains of increased urinary frequency and significantly increased urine volume. A 24-hour urine collection 2 demonstrates urine osmolarity of 240 mOsm/L and specific gravity of 1.005 in a total volume of 4 L. The concentration does not increase on . 3 administration of desmopressin 4 Which of the following is the most likely diagnosis? 5 6 A. Central diabetes insipidus 7 8 B. Diabetes mellitus

9 C. Nephrogenic diabetes insipidus 10 D. Primary polydipsia 11 E. Syndrome of inappropriate secretion of ADH 12 13 14 15 16 17 18 19 • 20 . 21 • a s 8 Lock Suspend End Block Item: 20 of 24 ~ 1 • M k -<:J 1>- Jil ~· !:';-~ QIO: 4163 ..L a r Previous Next Lab fli!ltues Notes Calculator

1 & & Th e correct an sw er i s C. 820/o ch o se this . 2 The low urine osmolarity suggests that the kidney is unable to concentrate urine. 3 Normally, final adjustments to urine osmolarity are made in the collecting duct, where Diagnosis of diabetes insipidus and polydipsia 1 aquaporins (water channels) are inserted in response to the action of vasopressin. This Nephrogenic Primary 4 Test 1central Dl allows water to leave the tubule for the hyperosmolar interstitium, thus preventing 01 polydipsia 5 water losses. Patients whose kidneys have decreased or absent responses to Random plaSma vasopressin have nephrogenic diabetes insipidus (DI), characterized by increased ~ 6 oomola~ urinary volume and frequency, and urinary osmolarity <300 mOsm/ l, as indicated in Randomunne - "'~ "'~ ~ 7 the table. Nephrogenic DI may be distinguished from a central DI by administration of osmolalty desmopressin, a vasopressin analog. In cent ral DI, desmopressin should increase Urine osmolality dUnng 8 No change No change urine osmolarity and decrease urine volume. water deprivation 9 Desmop e .in Osmotic concentration Diabetes insipidus Nephrogenoc diabetes insipidus - Urine osmolality aner No change "' IVOOAVP 10 Vasopressin Aquaporin Collecting duct system Urine Diabetes mellitus Kidney Interstitial fluid I PlasmaADH "'~ Normal to 1' "'~ Nephron I 11

12 A is not correct. 90/o c hose t his . 13 Central diabetes insipidus (DI) occurs when the posterior pituitary does not release vasopressin. This may occur due to trauma, brain tumors, or infections. However, in central DI the kidney would respond to desmopressin by producing more concentrated urine and having decreased urine 14 output. 15 Desmopressin Diabetes insipidus Vasopressin Neurogenic diabetes insipidus Posterior pituitary Diabetes mellitus Kidney Urine Pituitary gland Oliguria Neoplasm 16 Brain 17 B is not correct. 30/o ch ose this . to desmopressin. Polyuria in diabetes 18 Diabetes mellitus may result in increased urinary frequency and polydipsia, but the kidney would respond mellitus is due to the hyperglycemia exceeding the kidney's ability to resorb glucose; this causes a state of osmotic diuresis. The response to 19 desmopressin still occurs because there is no renal pathology present. 20 Desmopre~sm Polyur'a Diabetes mellitus Osmotic diuresis Polydipsia Hyperglycemoa Kidney Glucose Diuretic Frequent urination Osmosis Forced diuresis Diuresis • 21 D is not correct . 30/o ch ose this • • a s 8 Lock Suspend End Block Item: 20 of 24 ~ . I • M k <:] t> al ~· ~ QIO: 4163 .l. ar Previous Next lab 'lifllues Notes Calculator

1 A is not correct. 90fo c hose t his. 2 Central d iabetes insipidus (DI) occu rs when the posterior pituitary does not release vasopressin. This may occu r due to trauma, brain tumors, or 3 infections. However, in central DI the kidney would respond to desmopressin by producing more concentrated urine and having decreased urine output. 4 Desmopressin Diabetes insipidus Vasopressin Neurogenic diabetes insipidus Posterior pituitary Diabetes mellitus Kidney Urine Pituitary gland Oliguria Neoplasm 5 Brain

6 B is not correct. 30fo c hose t his. 7 Diabetes mellitus may resu lt in increased urinary frequency and polydipsia, but the kidney would respond to desmopressin . Po lyuria in d iabetes 8 mellitus is due to the hyperglycemia exceeding the kidney's ability to resorb g lucose; this causes a state of osmotic d iuresis. The response to desmopressin still occu rs because there is no renal pathology present. 9 Desmopressin Polyuria Diabetes mellitus Osmotic diuresis Polydipsia Hyperglycemia Kidney Glucose Diuretic Frequent urination Osmosis Forced diuresis Diuresis

10 0 is not correct. 30fo c hose t his. 11 Primary polydipsia is a condition of excessive thirst. Although the urine osmolarity would be lowered by increased fluid intake, the kidney would respond to administration of desmopressin . 12 Desmopressin Polydipsia Osmotic concentration Primary polydipsia Kidney Urine 13 E is not correct. 30fo c hose t his. 14 The syndrome of inappropriate secretion of ADH is characterized by high levels of vasopressin. The urine is highly concentrated, and 15 hyponatremia may resu lt from retention of water. Hyponatremia Vasopressin Urine Water retention (medicine) Secretion Syndrome of inappropriate antidiuretic hormone secretion 16 17 18 Bottom Line : 19 In nephrogenic d iabetes insipidus, the kidney is unable to concentrate urine, lead ing to increased urinary frequency and volume with low osmolarity that does not improve with desmopressin . 20 Desmopressin Diabetes insipidus Nephrogenic diabetes insipidus Osmotic concentration Diabetes mellitus Kidney Urine Frequent urination • 21 6 s 0 lock Suspend End Block Item: 20 of 24 ~ 1 • M k -<:J 1>- Jil ~· !:';-~ QIO: 4163 ..L a r Previous Next Labfli!l tues Not es Calculat o r • •• ,..,a ...., y I 1 FIRST AID FACTS 2 3 FA17 p 334.1 4 Diabetes insipidus Characterized by intense thirst and polyuria with inability to concentrate urine due to lack of AOIJ 5 (central) or failure of response to circulating AD II (nephrogenic). 6 Central 01 Nephrogenic 01 7 ETIOLOGY Pituitary tumor, autoimmune, trauma, surgery, Hereditary (ADH receptor mutation), 2° 8 ischemic encephalopathy, idiopathic to hypercalcemia, hypokalemia, lithium, demeclocycline (ADH antagonist) 9 FINDINGS ! DH ·ormal or t ADH b·els 10 Urine specific gra\'itr < 1.006 Urine specific gra,•ity < 1.006 11 Serum osmolality> 290 mOsm/kg Serum osmolality> 290 mOsm/kg 12 Hyperosmotic volume contraction Hyperosmotic ,·olume contraction 13 WATER DEPRIVATION TEST' >50% t in urine osmolality only after l\ linimal change in urine osmolality, even after 14 ad ministration of ADH amrlog administration of ADH analog 15 TREATMENT Desmopressin acetate HCTZ, indomethacin, amiloride 16 Hyd ration Hydration, dietary salt rest ric tion, avoidance of 17 offending agent 18 a o water intake for 2-3 hr followed by hou rly measurements of urine volume and osmolarity and plasma Na+ concentration 19 aud osmolarity. AD II analog (desmopressin acetate) is administered if serum o~mo l ality > 295-300 mOsm/kg, plasma Na+ ~ H5, or urine osmolality does not rise despite a rising plasma osmolality. 20 . 21 • a s 8 Lock Suspend End Block Item: 21 of 24 ~ 1 • M k -<:J 1>- Jil ~· !:';-~ QIO: 4742 ..L a r Previous Next Labfli!llues Not es Calculat o r

1 & & A 72-year-old retired man recently diagnosed with heart disease arrives at the emergency depa•·tment complaining of acute-onset 2 respiratory difficulty and severe circumoral, periorbital, and facial edema. The patient denies any pruritus or skin rashes. On questioning, he to control his high blood pressure 3 says his primary care physician recently put him on a regimen of atorvastatin, metoprolol, and captopril and cholesterol. The physician suspects that the symptoms the patient is currently experiencing are related to one of the patient's medications. 4

5 The accumulation of what substance is most likely causing the patient's symptoms? 6 : 7 A. Bradykinin 8 B. Histamine 9 10 C. Prostacyclin 11 0. Prostaglandin E2

12 E. Serotonin 13 14 15 16 17 18 19 20 . 21 • a s 8 Lock Suspend End Block Item: 21 of 24 ~ . I • M k <:] t> al ~· ~ QIO: 4742 .l. ar Previous Next Lab'lifllues Notes Calculator

1 The correct answer is A. 7 30/o chose t his. 2 The patient's symptoms are of angioedema, a well-known adverse effect of angiotensin-converting enzyme (ACE} inhibitors. These d rugs are generally prescribed to patients with hypertension. Angioedema is caused by the secondary activity of ACE on the degradation of kin ins, 3 includ ing, most commonly, bradykinin. The blockage of ACE, and the commensurate accumulation of high levels of bradykinin, account for 4 increased vessel permeability and subsequent edema in the face, lips, mouth, and subglottic tissues, typically without pruritus. Intestinal edema can also lead to gastrointestinal symptoms of d iarrhea, vomiting, and abdominal pain in some patients. Older patients with a prior history of 5 d rug -induced hypersensitivity and environmental allergies are particularly susceptible to ACE inhibitor -induced angioedema. 6 Bradykinin Angiotensin-converting enzyme ACE inhibitor Enzyme Edema Angioedema Diarrhea Adverse effect Hypertension Itch Hypersensitivity Allergy 7 Abdominal pain Vomiting Human gastrointestinal tract Gastrointestinal tract

8 B is not correct. 120/o chose t his. 9 Histam ine is a biogenic amine that has a variety of functions, includ ing inflammation, smooth muscle and vascular d ilatation, and neurotransmission. Histam ine released by mast cells is primarily responsible for the hypersensitivity reaction, although in this case histamine 10 levels are not affected by the use of ACE inhibitors. This patient's symtpoms- circumoral, periorbital, and facial edema not associated with 11 pruritus or skin rash - in the setting of ACE inhibitor usage, points toward angioedema instead of a histamine-mediated allergic reaction. ACE inhibitor Biogenic amine Histamine Amine Angioedema Itch Neurotransmission Allergy Mast cell Smooth muscle tissue Edema Inflammation Hypersensitivity 12 Blood vessel Muscle Enzyme inhibitor 13 C is not correct. 5 0/o chose t his. 14 Prostacyclin (or prostagland in !2} is an arachidonic acid derivative, produced by the vascular endothelial cells from PGH2. Its major function is to 15 prevent platelet aggregation during coagulation. Prostacyclin is also a potent vasodilator, although its metabolism is not affected by ACE, and thus prostacyclin levels are not affected by the use of ACE inhibitors. 16 Arachidonic acid Prostacyclin Prostaglandin Vasodilation Platelet ACE inhibitor Platelet aggregation Endothelium Coagulation Metabolism Blood vessel 17 0 is not correct. S O/o chose t his. 18 Prostagland in E2 (PGE2) is an arachidonic acid derivative that controls smooth muscle contraction, d ilatation, and constriction of blood vessels, as 19 well as the modulation of inflammation. It is also im plicated in the induction of fever. However, PG E2 has no relationship to ACE inhibitors and the kinin -related effects of ACE. 20 Arachidonic acid Prostaglandin Smooth muscle tissue ACE inhibitor Inflammation Muscle contraction Vasoconstriction Fever Muscle 21 F is not c.orrP.c.t. J O/o c.hosP. t his. 6 s 0 lock Suspend End Block Item: 21 of 24 ~ . I • M k <:] t> al ~· ~ QIO: 4742 .l. ar Previous Next Lab'lifllues Notes Calculator

1 • neurotransmission. Histam ine released by mast cells is primarily responsible for the hypersensitivity reaction, although in this case histamine levels are not affected by the use of ACE inhibitors. This patient's symtpoms- circumoral, periorbital, and facial edema not associated with 2 pruritus or skin rash - in the setting of ACE inhibitor usage, points toward angioedema instead of a histamine-mediated allergic reaction . 3 ACE inhibitor Biogenic amine Histamine Amine Angioedema Itch Neurotransmission Allergy Mast cell Smooth muscle tissue Edema Inflammation Hypersensitivity 4 Blood vessel Muscle Enzyme inhibitor 5 C is not correct. 5 0/o chose t his. 6 Prostacyclin (or prostagland in l2} is an arachidonic acid derivative, produced by the vascular endothelial cells from PGH2. Its maj or function is to prevent platelet aggregation during coagulation. Prostacyclin is also a potent vasodilator, although its metabolism is not affected by ACE, and 7 thus prostacyclin levels are not affected by the use of ACE inhibitors. 8 Arachidonic acid Prostacyclin Prostaglandin Vasodilation Platelet ACE inhibitor Platelet aggregation Endothelium Coagulation Metabolism Blood vessel 9 0 is not correct. S O/o chose t his. 10 Prostagland in E2 (PGE2) is an arachidonic acid derivative that controls smooth muscle contraction, d ilatation, and constriction of blood vessels, as well as the modulation of inflammation . It is also im plicated in the induction of fever. However, PG E2 has no relationship to ACE inhibitors and the 11 kinin -related effects of ACE. 12 Arachidonic acid Prostaglandin Smooth muscle tissue ACE inhibitor Inflammation Muscle contraction Vasoconstriction Fever Muscle 13 E is not correct . 2 0/o chose t his. 14 Serotonin is primarily a neurotransmitter produced by the central nervous system and certain peripheral neurons (enteric neurons). It is not involved in vascular d ilatation and inflammation, and has no relationship to ACE inhibitors. 15 Serotonin Central nervous system Neurotransmitter Nervous system Enteric nervous system ACE inhibitor Neuron Inflammation Gastrointestinal tract 16 Human gastrointestinal tract Blood vessel 17

18 Bottom Line: 19 The blockage of angiotensin-converting enzyme and the accumulation of bradykinin account for increased vessel permeability and subsequent 20 edema in the face, lips, mouth, and subglottic tissues, typically without pruritus. Bradykinin Angiotensin-converting enzyme Enzyme Itch Edema 21 • 6 s 0 lock Suspend End Block Item: 21 of 24 ~ 1 • M k -<:J 1>- Jil ~· !:';-~ QIO: 4742 ..L a r Previous Next Labfli!llues Not es Calculat o r

1 & &

2 FA17p577.1 3 Angiotensin­ Captopril, enalapril, lisinopril, ramipril. 4 converting enzyme inhibitors 5 MECHANISM - ~ AT II - ~ CFR b) prc,enting 6 Inhibit ACE constriction of efferent arterioles. f renin due 7 to loss of negati,·e feedback. Inhibition of CE 8 also prevents inactivation of brad) kinin, a 9 potent vasodilator. 10 CLINICAL USE Hypertension, IIF U mortality), proteinuria, In chronic kidney disease (eg, diabe ti c 11 diabetic nephropathy. Pre,entunfmorablc nephropathy), l intraglomerular pressure, 12 heart remodeling as a result of chronic slowing CBYl thickening. hypertension. 13 ADVERSE EFFECTS IIU. 14 Cough, Angioedema (due to f bradykinin; Captopril's CATC contraindicated in Cl esterase inhibit·or 15 deficiency), Teratogen (fe tal renal 16 malfo rmations), f Creatinine U CFR), 17 l lyperka lemia, and llypotension . Used wi th 18 caution in bilateral renal artery stenosis 19 because ACE inhibitors will further ~ C FR - renal failure. 20 21 • a s 8 Lock Suspend End Block 21 24 • M k ~ £!1}>' !!":-~ Item: of ~ 1 a r -<:J I> • QIO: 4742 ..L Previous Next Lab lues Not es Calculat o r

& & 1 FA17 p 391 .2 2 Coagulation and kinin pathways 3 Collagen, •••••••••••••••••• ••••••••••••••••· HMWK 4 basement membrane, ,/ Kal~k j Vasod'laf activated platelets •• ••• retn · ----- ~1 •• ·'"' ' ton 5 . ... ., .. : ./ BraA.A.il'lln -~~:: ••••• ~ i Permeabi~ty 6 Contact I # ~ ~ ~~ . ~ ' ~ ~ ~. 7 xu- Xlla •• i Patn 8 XI ...!...XIa 9 IX--Lixa Tissue factor 1 VIII 10 "-...... _ • - with vWF ANTICOAGULANTS: lla (thrombin) Tissue factc vu---*:----+ VIla-...... :::"".- --, • heparin (greatest efficacy) (extnnstc) 11 - LMWH (dalteparin. enoxaparinl pathway X,.--....Xa 12 • dtrect thrombin inhibitors (argatroban. - j• - v bivaltrudin. dabigatran) 13 ANTICOAGULANTS: factor Xa - LMWH (greatest efficacy) II ,.--....lla ~ 14 Prothrombin Thrombin · heparin Plasminogen 15 · direct Xa inhibitors (apixaban. rivaroxaban) ,.--....~ •. THROMBOLYTICS: • fondaparinux Jtf alteplase, reteplase. 16 Ia tPA ~ streptokinase. tenecteplase Ftbrinogen Fibrin monomers 17 ~ Aminocaproic aod 18 Hemophilia A: deficiency of factor VIII O- Jil ~· !:';-~ QIO: 4167 ..L a r Previous Next Labfli!llues Not es Calculat o r

A A 4 A 25-year-old sexually active woman presents to the emergency room for worsening dysuria and urina•·y frequency. The patient states the 5 symptoms began a day ago after she had protected sex with her boyfriend. The patient has a past medical history significant for type 2 diabetes, polycystic ovarian syndrome and recurrent urinary tract infections. Over the past two years she has been treated six times for 6 cystitis and once for pyelonephritis. Vital signs upon entrance to the emergency department are as follows: temperature of 98. 7°F, blood pressure of 7 122/82 mm Hg, heart rate of 88/min, and a respiratory rate of 16/min. The physical exam is significant for suprapubic tenderness.

8 her infections? 9 Which of the following is characteristic of the microorganism that most commonly causes

10 : A. It grows most efficiently at 42°C. 11

12 B. It grows well in bile and demonst rates variable hemolysis

13 C. It is a hemolytic 14 D. It is only seen under dark-field microscopy 15 and will ferment maltose 16 E. It is oxidase positive

17 F. It will ferment lactose 18 19

20

21 • 22

• 23 • 24 • a s 8 Lock Suspend End Block Item: 22 of 24 ~ 1 • M k -<:J 1>- Jil ~· !:';-~ QIO: 4167 ..L a r Previous Next Labfli!llues Not es Calculat o r

A A 4 The correct a ns w e r i s F. 670/o chose this . in young, sexually active women is Escherichia 5 The most common cause of urinary tract infections coli. It is typically transmitted by fecal contamination of the urethra. E. coli is a gram-negative rod 6 that ferments lactose, forming pink colonies on MacConkey agar, as shown in the image. infection 7 Eschenchia col Gram-negative bacteria MacConkey agar urethra Lactose Ur'nary system Ur'nary tract Feces Agar 8 9 10 11 12 Image courtesy of Wikimedia Commons 13 14 A is not c orrect. 40/o chose t his. Campylobacter jejuni, a cause of food poisoning that is commonly associated with poultry, is known for its high optimal growth temperature of 15 42°C. Escherichia coli does not grow well in such extreme conditions. 16 Campylobacter jejuni Escherichia coli Campylobacter Foodborne illness Poultry

17 B is not co rrect. 6 0/o c hose this. 18 Enterococcus grows well in bile as well as in hig h-salt enviroments and can be a -, ~-,or y -hemolytic. Enterococcal UT!s are usually nosocomial and are often associated with urinary catheters. 19 Enterococcus Urinary catheterization Catheter Hospital-acquired infection Bile 20 C is no t correct . ] Ofo chose this . 21 Hemolysis of RBCs on a culture plate is a way to distinguish between streptococcal species. a-Hemolysis is partial lysing of the RBCs and is 22 characteristic of Streptococcus pneumoniae and the viridans streptococci. S. pneumoniae causes bacterial pneumonia, whereas the viridans streptococci are often implicated in subacute bacterial endocarditis following dental procedures . • 23 Streptococcu~ pneumomae Hemolysis Bacterial pneumonia Endocarditis Vindans streptococc Pneumonia Streptococcus • 24 Subacute bactertal endocarditis Lysis a s 8 Lock Suspend End Block Item: 22 of 24 ~ 1 • M k -<:J 1>- Jil ~· !:';-~ QIO: 4167 ..L a r Prev ious Next Labfli!llues Not es Calculat or

- ~ . '' . -. .. A ...... A 4 5 A is n o t correct . 4 0/o ch ose this . temperature of 6 Campylobacter jejuni, a cause of food poisoning that is commonly associated with poultry, is known for its high optimal growth 42° C. Escherichia coli does not grow well in such extreme conditions. 7 Campylobacter jejuni Escher1chia coli campylobacter Foodborne lness Poultry

8 B is n o t correct . 6 0fo chose this. 9 Enterococcus grows well in bile as well as in high- salt enviroments and can be o-, ~-,or y-hemolytic. Enterococcal UTis are usually nosocomial and are often associated with urinary catheters. 10 Enterococc s Urinary catheterization Catheter Hospital-acquired infection Bile 11 C is not correct. 7 0fo ch ose t h is. 12 Hemolysis of RBCs on a culture plate is a way to distinguish between streptococcal species. a-Hemolysis is partial lysing of the RBCs and is 13 characteristic of Streptococcus pneumoniae and t he viridans streptococci. S. pneumoniae causes bacterial pneumonia, whereas the viridans bacterial endocarditis following dental procedures. 14 streptococci are often implicated in subacute Streptococcus pneumoniae Hemolysis Bacterial pneumonia Endocarditis Viridans streptococci Pneumonia Streptococcus Infective endocarditis 15 Subacute bacterial endocarditis Lysis 16 D is not co rrect. 4 0/o c hose t his . 17 Dark-field microscopy is req uired to visualize spirochetes, such as Treponema pallidum, the cause of syphilis. These bacteria do not cause urinary 18 tract infections. Treponema pallidum Dark field microscopy Syphilis Microscopy Spiral bacteria Bacteria Spirochaete Treponema Urinary tract infection 19 E is no t correct. 1 2 0/o ch ose this . 20 Maltose fermentation is characteristic of the g ram-negative cocci Neisseria meningitidis. N. meningitidis is an oxidase-positive organism that 21 causes meningitis in teens and adults, although there is a meningitis vaccine that protects against ce•·tain strains. The other gram-negative 22 coccus, Neisseria gonorrhoeae, is a maltose n o n fermenter. Neisseria meningitidis Neisseria gonorrhoeae Coccus Maltose Gram-negative bacteria Meningitis Neisseria Fermentation Organism Vaccine Industrial fermentation • 23 Fermentation In food processing • 24 a s 8 Lock Suspend End Block Item: 22 of 24 ~ . I • M k <:] t> al ~· ~ QIO: 4167 .l. ar Previous Next lab 'lifllues Notes Calculator

4 B is not correct. 60/o c hose t his.

5 Enterococcus grows well in bile as well as in high-salt enviroments and can be a-, ~ - , o r y-hemolytic. Enterococcal UT!s are usually nosocomial 6 and are often associated with urinary catheters. Enterococcus Urinary catheterization Catheter Hospital-acquired infection Bile 7 C is not correct. ]Ofo c hose t his. 8 Hemolysis of RBCs on a culture plate is a way to d istinguish between streptococcal species. a-Hemolysis is partial lysing of the RBCs and is 9 characteristic of Streptococcus pneumoniae and the viridans streptococci. S. pneumoniae causes bacterial pneumonia, whereas the viridans 10 streptococci are often implicated in subacute bacterial endocard itis following dental procedures. Streptococcus pneumoniae Hemolysis Bacterial pneumonia Endocarditis Viridans streptococci Pneumonia Streptococcus Infective endocarditis 11 Subacute bacterial endocarditis lysis 12 0 is not correct. 4 0fo c hose t his. 13 Dark-field microscopy is required to visualize spirochetes, such as Treponema pallidum, the cause of syphilis. These bacteria do not cause urinary 14 tract infections. Treponema pallidum Dark field microscopy Syphilis Microscopy Spiral bacteria Bacteria Spirochaete Treponema Urinary tract infection 15 E is not correct. 1 20/o c hose t his. 16 Maltose fermentation is characteristic of the g ram -negative cocci Neisseria meningitidis. N. meningitidis is an oxidase-positive organism that 17 causes meningitis in teens and adults, although there is a meningitis vaccine that protects against certain strains. The other g ram -negative 18 coccus, Neisseria gonorrhoeae, is a maltose no nfermenter. Neisseria meningitidis Neisseria gonorrhoeae Coccus Maltose Gram-negative bacteria Meningitis Neisseria Fermentation Organism Vaccine Industrial fermentation 19 Fermentation in food processing 20 21 Bottom Line : 22 E. coli, which ferments lactose, is the most common cause of urinary tract infections in sexually active young women . • 23 Escherichia coli lactose Urinary system Urinary tract infection • 24 6 s 0 lock Suspend End Block Item: 22 of 24 ~ 1 • M k -<:J 1>- Jil ~· !:';-~ QIO: 4167 ..L a r Previous Next Labfli!llues Not es Calculat o r

A A 4 FA17 p 137.2 5 Gram-negative lab algorithm 6 7 8 9 Diplococci Coccobacilfi Convna-shaped rods 10 11 12 Aerobtc Haemophilus influenzH Oxidase '+' (reqwes factors Vand Xl 13 Maltose utilization Pasteurella Brucella 14 Bo~trlla pmussis Grows in 42•c Grows in alkaline media Produces urease Francisella tularens1s 15 N 9CJIIOffi!MH Clmpylob«ttr jtjuni Vibrio cholerae Htlicob«ttr pylori N mtningitidis 16 Moraxtlla 17 18 19 Bacilli 20 I 21 lactose fermentation 22 -e---'-----1~ . 23 Oxida.se Fast Slow . 24 • I ( a s 8 Lock Suspend End Block Item: 22 of 24 ~ .I • M k <:] t> al ~· ~ QIO: 4167 .l. ar Previous Next lab 'lifllues Notes Calculator • • 4 FA17 p 138.1 5 Lactose-fermenting Fermentation of lactose ... pink colonies Lactose is key. 6 enteric bacteria on MacConkcy agar. Examples include Test with MacConKEE'S agar. 7 Klebsiella, E coli, Enterobacter, and EMB agar-lactose fermenters grow as purple/ black colonies. E coli grows colonies with a 8 Serratia (weak fenncntcr). E coli produces ~-galactosidase, which breaks down lactose green sheen. 9 into glucose and galactose. 10 11 FA11 p 177.2 12 UTI bugs 13 SPECIES FEATURES COMMENTS 14 Escherichia coli Leading cause of UTI. Colonies show green Diagnostic markers: 15 metall ic sheen on EMB agar. <±> = evidence of WBC 16 Staphylococcus 2nd leading cause of UTI in sexually active activit)'· 17 saprophyticus women. <±> itrite test = reduction of urinary nitrates by bacterial species (eg, E coli). 18 Klebsiella pneumoniae 3rd leading cause of UTJ. Large mucoid capsule <±> Urease test = urease-producing bugs (eg, and viscous colonies. 19 S saprofJhyticus, Proteus, Klebsiella). 20 Serratia marcescens Some strains produce a red pigment; often 21 nosocomial and drug res istant. 22 Enterococcus Often nosocomial and drug res istant. • 23 Proteus mirabilis Nlotil ity causes "swarming" on agar; produces urease; associated with struvitc stones . • 24 • • 6 s 0 lock Suspend End Block Item: 23 of 24 ~ 1 • M k -<:J 1>- Jil ~· !:';-~ QIO: 4744 ..L a r Previous Next Labfli!llues Not es Calculat o r

A A 4 A 22-year-old college student is rushed to the emergency department by her roommate, who found her lying unconscious on her bed. The 5 roommate is unsure of the patient's past m edical history, but notes that the patient has been suffering from excruciatingly painful menstrual cramps for several days for which she was taking pain medication. A careful examination reveals no signs of trauma. Her temperature is 6 38.2°C {100.8°F), respiratory rate is 37/min, blood pressure is 145/59 mm Hg, and heart rate is 110/min. Laboratory studies show: 7 Hematocrit: 34% 8 Hemoglobin: 12.9 g/dl 9 Na+: 145 mEq/L K+: 3.2 mEq/L 10 AST: 20 IU/L 11 ALT: 30 IU/L cl-: 112 mEq/L 12 HC03-: 13 mEq/L 13 Blood urea nitrogen: 14 mg/dl 14 Creatinine: 0.9 mg/dl Glucose: 137 mg/dl 15 Arterial blood gas analysis shows a pH of 7.32, partial pressure of carbon dioxide of 27 mm Hg, and partial pressure of oxygen of 99 mm Hg. 16 17 What is the most likely cause of this patient's presentation? 18 19 : A. Acetaminophen toxicity 20 21 B. Diabetic ketoacidosis

22 C. Ethanol toxicity • 23 D. Menorrhagia • 24 a s 8 Lock Suspend End Block Item: 23 of 24 ~ Ma rk -<:J I> £!1?' ~~~ QI0: 4744 ,. • ..L Prev i-ous Next Lab ~lu es Not es Calculat o r 4 C. Ethanol toxicity .,. 5 Like Dislike D. Menorrhagia ·- 6 E. Opiate toxicity 7 8 F. Salicylate toxicity 9 10 11 The c orrect a ns wer is F. 61 Ofo chose this. 12 The arterial blood gas tests in this patient reveal that she is suffering from metabolic acidosis (as indicated by the low pH and bicarbonate). 13 Additionally, this patient has an anion gap of 20 (normal range is 8-16; anion gap = Na+- [cl- + Hco3- ]), and by definition a patient with an increased anion gap has an anion gap metabolic acidosis. The differential diagnosis for high anion gap metabolic acidosis includes Methanol, 14 Uremia, Diabetic ketoacidosis, Propylene glycol, I ron tablets or I soniazid, Lactic acidosis, Ethylene glycol/ethanol toxicity, and Salicylate toxicity, 15 which together make up the mnemonic MUDPILES. Salicylates in particular also cause a respiratory alkalosis by promoting hyperventilation (this is not solely compensatory, but also a direct effect of salicylates), creating a mixed anion gap metabolic acidosis and respiratory alkalosis. This will 16 result in a decreased PC02. 17 Respiratory alkalosis Arterial blood gas Metabolic acidosis High anion gap metabolic acidosis Hyperventilation Ethanol Anion gap PH Ketoacidosis Bicarbonate 18 Acidosis Salicylic acid Blood gas Alkalosis Mnemonic D ifferential diagnosis Anion Metabolism 19 A is no t correct. 170/o c hose this. 20 Acetaminophen (paracetamol) toxicity results in signs and symptoms of acute hepatitis and may result in fulminant hepatic failure. Many individuals with acetaminophen toxicity may have no symptoms at all during the first 24 hours after overdose. After massive overdoses, 21 metabolic acidosis is a possibility. However, typically acetaminophen toxicity manifests with transaminitis after 2- 3 days, which can be detected 22 by measuring the liver enzymes aspartate aminotransferase and alanine aminotransferase, which are normal in this patient. Paracetamol Alanine transaminase Aspartate transaminase Metabolic acidosis Hepatotoxicity Uver Paracetamol toxicity Hepatitis Uver failure Acute liver failure 23 Alanine Aspartic acid Acidosis Toxicity Enzyme Fulminant Drug overdose Metabolism Uver function tests Transaminase Acute hepatitis • 24 a s 8 Lock Suspend End Block Item: 23 of 24 ~ . I • M k <:] t> al ~· ~ QIO: 4744 .l. ar Previous Next Lab'lifllues Notes Calculator • 4 The correct ans w e r is F. 61 Ofo c hose t his. 5 The arterial blood gas tests in this patient reveal that she is suffering from metabolic acidosis (as ind icated by the low pH and bicarbonate). Additionally, this patient has an anion gap of 20 (normal range is 8 -16; anion gap = Na+ - [CI- + HCo - )), and by definition a patient with an 6 3 increased anion gap has an anion gap metabolic acidosis. The d ifferential d iag nosis for hig h anion gap metabolic acidosis includes Methanol, 7 U remia, D iabetic ketoacidosis, Propylene g lycol, I ron tablets or I soniazid, Lactic acidosis, Ethylene g lycol/ethanol toxicity, and Salicylate toxicity, 8 which together make up the mnemonic MUOPILES. Salicylates in particular also cause a respiratory alkalosis by promoting hyperventilation (this is not solely compensatory, but also a d irect effect of salicylates), creating a mixed anion gap metabolic acidosis and respiratory alkalosis. This will 9 result in a decreased PC02. 10 Respiratory alkalosis Arterial blood gas Metabolic acidosis High anion gap metabolic acidosis Hyperventilation Ethanol Anion gap PH Ketoacidosis Bicarbonate Acidosis Salicylic acid Blood gas Alkalosis Mnemonic Differential diagnosis Anion Metabolism 11 A is no t correct . 1 70/o c hose t his. 12 Acetaminophen (paracetamol) toxicity results in sig ns and symptoms of acute hepatitis and may result in fulminant hepatic failure. Many 13 ind ivid uals with acetaminophen toxicity may have no symptoms at all d uring the first 24 hours after overdose. After massive overdoses, 14 metabolic acidosis is a possibility. However, typically acetaminophen toxicity manifests with transaminitis after 2-3 days, which can be detected by measuring the liver enzymes aspartate aminotransferase and alanine aminotransferase, which are normal in this patient. 15 Paracetamol Alanine transaminase Aspartate transaminase Metabolic acidosis Hepatotoxicity liver Paracetamol toxicity Hepatitis liver failure Acute liver failure

16 Alanine Aspartic acid Acidosis Toxicity Enzyme Fulminant Drug overdose Metabolism liver function tests Transaminase Acute hepatitis

17 B is no t correct . 100/o c hose t his. 18 Diabetic ketoacidosis is one of the causes of hig h anion gap metabolic acidosis. However, the g lucose levels req uired to achieve a ketoacidotic 19 state are substantially hig her than those recorded in this patient. In addition, the K+ plasma concentration ([ K+] ) is often hig h d ue to extracellular shifting . However, it important to note the d istinction the total body K+ is commonly low in these patients despite the elevated [ K+] . 20 Treatment of DKA often results in a d rastic red uction of [ K+] as insulin d rives K+ into the cells. 21 High anion gap metabolic acidosis Diabetic ketoacidosis Metabolic acidosis Anion gap Ketoacidosis Blood plasma Acidosis Insulin Diabetes mellitus Anion Glucose Metabolism 22 C is no t correct . 4 0/o c hose t his. 23 Ethanol toxicity is always a possibility in this college-age patient, and ethanol toxicity can cause an anion gap metabolic acidosis. However, this • 24 ~ n.::.tiP nt'c:: hic::tn nt n f t.::ll<-in n P¥r P c::c::ivP rnPrl ir.::ltin n n n intc:: m n n :. t n c::.::.lirvl.::.tP t n¥iritv A rlrl itin n.::.llv Pth.::.n n l intn¥ir.::ltin n ic:: n ftP n .::.rrn mn.::.niPrl hv 6 s 0 lock Suspend End Block Item: 23 of 24 ~ . I • M k <:] t> al ~· ~ QIO: 4744 .l. ar Previous Next lab'lifllues Notes Calculator • 4 extracellular shifting. However, it important to note the d istinction the total body K+ is commonly low in these patients despite the elevated [ K+] . Treatment of DKA often resu lts in a d rastic reduction of [ K+] as insulin d rives K+ into the cells. 5 High anion gap metabolic acidosis Diabetic ketoacidosis Metabolic acidosis Anion gap Ketoacidosis Blood plasma Acidosis Insulin Diabetes mellitus Anion Glucose 6 Metabolism

7 C is no t correct . 4 0/o c hose t his. 8 Ethanol toxicity is always a possibility in this college-age patient, and ethanol toxicity can cause an anion gap metabolic acidosis. However, this patient's history of taking excessive medication points more to salicylate toxicity. Additionally, ethanol intoxication is often accompanied by 9 respiratory depression, hypotension, and hypoglycema- none of which is seen in this patient. 10 Alcohol Metabolic acidosis Hypoventilation Ethanol Hypotension Anion gap Anion Salicylic acid Acidosis Toxicity Alcohol intoxication Depression (mood) 11 Metabolism Aspirin poisoning Substance intoxication Major depressive disorder 12 0 is no t correct . 30/o c hose t his. Menstrual bleeding does not cause metabolic acidosis. Although it may have ind irectly contributed to this patient taking high doses of pain 13 medication, it is not the immed iate cause of this patient's problem. In addition, this patient's hemoglobin and hematocrit are within normal limits. 14 Metabolic acidosis Hematocrit Hemoglobin Acidosis Menstrual cycle Menstruation Analgesic Metabolism

15 E is no t correct . 50/o c hose t his. 16 Opiate toxicity manifests with respiratory depression, miosis, hypotension, central nervous system depression, acute mental status changes, 17 arrhythmias, and seizures. This patient presents with hyperpnea and hypertension, making opiate intoxication unlikely. Miosis Hypoventilation Opiate Central nervous system Hypotension Hypertension Epileptic seizure Central nervous system depression Opioid overdose Toxicity 18 Cardiac arrhythmia Major depressive disorder Depression (mood) Nervous system Altered state of consciousness 19 20 Bo tto m Line : 21 Common causes of anion gap metabolic acidosis include methanol, uremia, d iabetic ketoacidosis, paraformaldehyde/isoniazid lactic acidosis, 22 ethylene g lycol/ethanol toxicity, and salicylate toxicity. 23 Diabetic ketoacidosis Uremia Methanol Metabolic acidosis Anion gap lactic acidosis Ketoacidosis Salicylic acid Acidosis Aspirin poisoning Anion Diabetes mellitus Metabolism Toxicity • 24 ~ 6 s 0 lock Suspend End Block 23 24 • M k ~ £!1}>' !!":-~ Item: of ~ 1 a r -<:J I> • QIO: 4744 ..L Prev ious Next Lab lues Not es Calculat o r

A A 4 FA17 p 561 .2 5 Acidosis and alkalosis 6 l pH 7 Check arteria 8 pH< 7.35 pH> 745 9 Acidemia Alkalemia 10 Pco > 44 mm Hg HC~- < 20 mEq/L 11 1 12 RespiratOf}' Respiratory 13 MetaboUc acidos1s Metabolic alkalOSIS acidosis alkalosis 14 15 anion gap Hypowntit.tion Check Hyptrvtntit.tion H• loss/HC01- excess 16 =Na • - (CI- +HCO ") Airway obstruction I Hysteria Loop diuretics 17 Acute lung disease Hypoxemia (eg, high altitude) Vomiting Chronic lung disease Salicylates (early) Antacid use 18 Opioids, sedatives Tumor Hyperaldosteronism Weakening of respiratory Pulmonary embolism 19 muscles 20 21 > 12 mEq/L 8-12 mEq/L Pco • 22 I 4S 40mmHg 23 40 Resporatory 1An ion gap Normal anion gap ::J ac1dosls . 24 3S • UIIOIIII ~C· llADIUCC· ~ a - s 8 Lock Suspend End Block Item: 23 of 24 ~ 1 • M k -<:J 1>- Jil ~· !:';-~ QIO: 4744 ..L a r Previous Next Labfli!llues Not es Calculat o r • 4 FA17 p239.1 5 Specific toxicity TOXIN TREATMENT 6 treatments Acetaminophen 1-acetylcysteine (replenishes glutathione) 7 AChE inhibitors, organophosphates Atropine> pralidoxime 8 Antimuscarinic, anticholinergic agents Physostigmine, control hyperthermia 9 Aisenic Dimercaprol, succimer 10 Benzodiazepines Flumazenil 11 ~-blockers Atropine, glucagon 12 Carbon monoxide 100% 0 , hyperbaric 0 13 2 2 Copper Penicillamine, trientine (Copper p en ny) 14 Cyanide Nitrile+ thiosulfate, hydroxocobalamin 15 16 Digitalis (digoxin) Anti-dig Fab fragments 17 Heparin Protamine sulfate 18 Iron Defc roxam ine, deferasirox, defcriprone 19 Lead EDTA, dimercaprol, succimer, penicillamine 20 l\lercury Dimercaprol, succimer 21 Methanol, ethylene glycol (a ntifree..:e) Fomepizole >ethanol, dialysis 22 l\ lethemoglobin 1\ Iet hylene blue, vitamin C 23 O piO ids NalO xO ne • 24 a s 8 Lock Suspend End Block Item: 24 of 24 ~ 1 • M k -<:J 1>- Jil ~· !:';-~ QIO: 4168 ..L a r Previous Next Labfli!l tues Not es Calculat o r

A A 4 A 72-year-old man presents to his physician with abdominal pain and distention. On inte•·view, he reveals that for the last year he had a 5 problem with starting a urine stream, but he has been unable to urinate well for the past 2 days. The patient has a past medical history of diabetes, hypertension, and coronary artery disease. The patient is a heavy smoker since the age of 24. The patient remembers that his 6 mother had a type of renal disease, and his father died at the age of 85 from prostate cancer. The patient is worried that cancer may run in his 7 family because his uncle also died due to bladder cancer. The patient's serum creatinine is 2.4 mg/dl. When a foley catheter is placed, urinary flow begins immediately. 8 9 Which of the following is the most likely diagnosis? 10 : 11 A. Benign prostatic hyperplasia 12 B. Bladder cancer 13 14 C. Diabetic nephropathy

15 D. Prostate cancer 16 E. Renal cell carcinoma 17

18 19

20

21

22

23 • 24 • a s 8 Lock Suspend End Block Item: 24 of 24 ~ . I • M k <:] t> al ~· ~ QIO: 4168 .l. ar Previous Next lab 'lifllues Notes Calculator • 4 The correct answer is A. 690/o chose this. 5 This man likely has ben ign prostatic hyperplasia {BPH}, a common condition in older males. It is caused by stimulation of the prostate by 6 d ihyd rotestosterone {DHT}. DH T is formed from testosterone by Sa-reductase. BPH may cause increased urinary frequency or urgency, d ifficulty urinating, and in extreme cases, anuria. This may resu lt in bladder d istention and symptoms of acute renal failure, which may be relieved by 7 d raining the bladder. Diabetes and smoking can contribute to a more complicated or atypical presentation of BPH . 8 Benign prostatic hyperplasia Anuria Dihydrotestosterone Testosterone Acute kidney injury Diabetes mellitus Hyperplasia Frequent urination Prostate Urinary bladder Prostate cancer Benign tumor Kidney 9

10 B is not correct. 90/o chose this. Bladder cancer typically manifests with painless hematuria and is associated with obstructive voiding symptoms such as intermittent urine 11 stream, nocturia, and/or feeling of incomplete voiding. Irritative voiding symptoms such as dysuria, increased frequency or urgency, and urge 12 incontinence may also be present. The most common cause of pain with bladder cancer is often due to metastasis and may commonly manifest as abdominal, back, and/or flank pain. Bladder cancer is more commonly d iagnosed in older ind ividuals with an average age at d iagnosis being 69. 13 Risk for bladder cancer has also been linked to professions such as painters, textile workers, and leather works. Exposure to polycyclic aromatic 14 hydrocarbons has been associated with bladder cancer. Dysuria Hematuria Nocturia Bladder cancer Urinary incontinence Metastasis Urinary bladder Abdominal pain Overactive bladder Polycyclic aromatic hydrocarbon 15 Cancer Fecal incontinence Irritation Aromaticity 16 C is not correct. 60/o chose this. 17 Diabetic nephropathy would not present with anuria. There are usually no symptoms in the early stages of d iabetic nephropathy even though 18 kidney damage and may be present. If d iabetic nephropathy progresses ind ividuals may begin to see edema in the feet, weakness, 19 nausea, and vomiting. Patients may need less insulin or antidiabetic medications because the kidney is unable to clear these medications from the body. Laboratory data may show elevated levels of blood urea nitrogen and serum creatinine. 20 Anuria Diabetic nephropathy Albuminuria Blood urea nitrogen Urea Creatinine Kidney Nausea Diabetes mellitus Edema Insulin Kidney disease Blood plasma 21 Anti-diabetic medication Serum creatinine Vomiting Nitrogen Serum (blood) Nephrotoxicity

22 0 is not correct. 120/o chose this. 23 Prostate cancer typically does not manifest with acute urinary retention or urinary symptoms- it is often asymptomatic. This is because most prostate cancers arise in zones of the prostate (the posterior lobes) that do not compress the prostatic urethra. The d iag ram shown here depicts 24 ~ ...... -.-...... & ...... - ...... - ...... _ .... : ...... -: ... 1... .-.-.-.1 -.-...... - ...... - ...... :.- , . _ ...... _ ...... _ .... & .... -.-. 6 s 0 lock Suspend End Block Item: 24 of 24 ~ . I • M k <:] t> al ~· ~ QIO: 4168 .l. ar Previous Next lab 'lifllues Notes Calculator • Risk for bladder cancer has also been linked to professions such as painters, textile workers, and leather works. Exposure to polycyclic aromatic • 4 hydrocarbons has been associated with bladder cancer. 5 Dysuria Hematuria Nocturia Bladder cancer Urinary incontinence Metastasis Urinary bladder Abdominal pain Overactive bladder Polycyclic aromatic hydrocarbon 6 Cancer Fecal incontinence Irritation Aromaticity 7 C is no t correct . 60/o c hose t his. 8 Diabetic nephropathy would not present with anuria. There are usually no symptoms in the early stages of d iabetic nephropathy even though kidney damage and albuminuria may be present. If d iabetic nephropathy progresses ind ividuals may begin to see edema in the feet, weakness, 9 nausea, and vomiting. Patients may need less insulin or antidiabetic medications because the kidney is unable to clear these medications from the 10 body. Laboratory data may show elevated levels of blood urea nitrogen and serum creatinine. Anuria Diabetic nephropathy Albuminuria Blood urea nitrogen Urea Creatinine Kidney Nausea Diabetes mellitus Edema Insulin Kidney disease Blood plasma 11 Anti-diabetic medication Serum creatinine Vomiting Nitrogen Serum (blood) Nephrotoxicity 12 0 is no t correct . 1 20/o c hose t his. 13 Prostate cancer typically does not manifest with acute urinary retention or urinary symptoms- it is often asymptomatic. This is because most 14 prostate cancers arise in zones of the prostate {the posterior lobes) that do not compress the prostatic urethra. The d iag ram shown here depicts the zones of the prostate and shows that most prostate cancers involve the peripheral zone and do not compress the prostatic urethra. Therefore 15 benign prostatic hyperplasia is more likely to cause this patient's obstructive symptoms. 16 Benign prostatic hyperplasia Urethra Urinary retention Prostate cancer Hyperplasia Cancer Prostatic urethra Prostate Benignity

17 E is no t correct . 4 0/o c hose t his. 18 Renal cell cancers also frequently present with hematuria but are frequently asymptomatic, although they can also be associated with flank pain and/or an abdominal lump. 19 Hematuria Abdominal pain Asymptomatic Kidney 20 21 Botto m Line : 22 Ben ign prostatic hyperplasia may cause increased urinary frequency or urgency, d ifficulty urinating, and in extreme cases, anuria. 23 Benign prostatic hyperplasia Anuria Frequent urination Hyperplasia 24 ~ 6 s 0 lock Suspend End Block Item: 24 of 24 ~ 1 • M k -<:J 1>- Jil ~· !:';-~ QIO: 4168 ..L a r Previous Next Labfli!l tues Not es Calculat o r

A A 4 FA17 p619.2 5 Benign prostatic Common in men> 50 years old. Characterized Anlet101lobe 6 hyperplasia by smooth, elastic, firm nodular enlargement ptOStatiC hyperplasia 7 (hyperplasia not h) pertrophr) of periurethral 8 (lateral and middle) lobes, \\hich compress the Lateral lobe ...... _,;--- 9 urethra into a \'ertical slit. Not premalignant. Often presents with t frequency of urinal ion, 'I 10 ' , ' I nocturia, difficulty starting and stopping urine Middle lobt ~ , • 11 ... ~~ stream, dysuria. ~ l ay lead to distention and ..- - ~------, 12 hrpertrophy of bladder, h) droneph rosis, UTh. Posteoor lobe ------• ~-;ostate cancer 13 t free prostate-specific antigen (PSA). 14 Treatment: a 1-antagonists (terazosin , tamsulosin), which cause relaxation of 15 smooth muscle; )a -reductase inhibitors (eg, 16 finasteride); PDE-5 inhibitors (eg, tadalafil); 17 surgical resection (eg, TURP, ablat ion). 18

19 FA17 p 619.4 20 Prostatic Common in men> 50 yea rs old . Arises most often from posterior lobe (peripheral zone) of prostate 21 adenocarcinoma gland and is most frequently diagnosed by t PSA and subsequent needle core biopsies. Prostatic 22 acid phosphatase (PAP) and PSA are useful tumor markers (f total PSA, with ~ fraction of free back pain 23 PSA). Osteoblastic metastases in bone may de\·elop in late stages, as indicated by lower and t serum ALP and PSA. 24 • a s 8 Lock Suspend End Block